Sunteți pe pagina 1din 101

ETERNAL GARDENS MEMORIAL G.R. No.

166245
PARK CORPORATION,
Petitioner,
Present:
CARPIO MORALES,
- versus - Acting Chairperson,
TINGA,
VELASCO, JR.,
CHICO-NAZARIO, and
BRION, JJ.
THE PHILIPPINE AMERICAN Promulgated:
LIFE INSURANCE COMPANY,
Respondent. April 9, 2008
x-----------------------------------------------------------------------------------------x

DECISION

VELASCO, JR., J.:

The Case

Central to this Petition for Review on Certiorari under Rule 45 which seeks to reverse and set aside the November 26,
2004 Decision[1] of the Court of Appeals (CA) in CA-G.R. CV No. 57810 is the query: May the inaction of the insurer on
the insurance application be considered as approval of the application?

The Facts

On December 10, 1980, respondent Philippine American Life Insurance Company (Philamlife) entered into an agreement
denominated as Creditor Group Life Policy No. P-1920[2] with petitioner Eternal Gardens Memorial Park Corporation
(Eternal). Under the policy, the clients of Eternal who purchased burial lots from it on installment basis would be insured
by Philamlife. The amount of insurance coverage depended upon the existing balance of the purchased burial lots. The
policy was to be effective for a period of one year, renewable on a yearly basis.

The relevant provisions of the policy are:

ELIGIBILITY.

Any Lot Purchaser of the Assured who is at least 18 but not more than 65 years of age, is indebted to the
Assured for the unpaid balance of his loan with the Assured, and is accepted for Life Insurance coverage
by the Company on its effective date is eligible for insurance under the Policy.

EVIDENCE OF INSURABILITY.

No medical examination shall be required for amounts of insurance up to P50,000.00. However, a


declaration of good health shall be required for all Lot Purchasers as part of the application. The
Company reserves the right to require further evidence of insurability satisfactory to the Company in
respect of the following:
1. Any amount of insurance in excess of P50,000.00.
2. Any lot purchaser who is more than 55 years of age.

LIFE INSURANCE BENEFIT.

The Life Insurance coverage of any Lot Purchaser at any time shall be the amount of the unpaid balance
of his loan (including arrears up to but not exceeding 2 months) as reported by the Assured to the
Company or the sum of P100,000.00, whichever is smaller. Such benefit shall be paid to the Assured if
the Lot Purchaser dies while insured under the Policy.

EFFECTIVE DATE OF BENEFIT.

The insurance of any eligible Lot Purchaser shall be effective on the date he contracts a loan with the
Assured. However, there shall be no insurance if the application of the Lot Purchaser is not approved by
the Company.[3]

Eternal was required under the policy to submit to Philamlife a list of all new lot purchasers, together with a copy of the
application of each purchaser, and the amounts of the respective unpaid balances of all insured lot purchasers. In relation
to the instant petition, Eternal complied by submitting a letter dated December 29, 1982, [4] containing a list of insurable
balances of its lot buyers for October 1982. One of those included in the list as new business was a certain John Chuang.
His balance of payments was PhP 100,000. On August 2, 1984, Chuang died.

Eternal sent a letter dated August 20, 1984[5] to Philamlife, which served as an insurance claim for Chuangs death.
Attached to the claim were the following documents: (1) Chuangs Certificate of Death; (2) Identification Certificate
stating that Chuang is a naturalized Filipino Citizen; (3) Certificate of Claimant; (4) Certificate of Attending Physician;
and (5) Assureds Certificate.

In reply, Philamlife wrote Eternal a letter on November 12, 1984,[6] requiring Eternal to submit the following
documents relative to its insurance claim for Chuangs death: (1) Certificate of Claimant (with form attached); (2)
Assureds Certificate (with form attached); (3) Application for Insurance accomplished and signed by the insured, Chuang,
while still living; and (4) Statement of Account showing the unpaid balance of Chuang before his death.

Eternal transmitted the required documents through a letter dated November 14, 1984,[7] which was received by
Philamlife on November 15, 1984.

After more than a year, Philamlife had not furnished Eternal with any reply to the latters insurance claim. This
prompted Eternal to demand from Philamlife the payment of the claim for PhP 100,000 on April 25, 1986.[8]

In response to Eternals demand, Philamlife denied Eternals insurance claim in a letter dated May 20, 1986,[9] a
portion of which reads:

The deceased was 59 years old when he entered into Contract #9558 and 9529
with Eternal Gardens Memorial Park in October 1982 for the total maximum insurable amount of
P100,000.00 each. No application for Group Insurance was submitted in our office prior to his death
on August 2, 1984.

In accordance with our Creditors Group Life Policy No. P-1920, under Evidence of Insurability provision,
a declaration of good health shall be required for all Lot Purchasers as party of the application. We cite
further the provision on Effective Date of Coverage under the policy which states that there shall be no
insurance if the application is not approved by the Company. Since no application had been submitted by
the Insured/Assured, prior to his death, for our approval but was submitted instead on November 15,
1984, after his death, Mr. John Uy Chuang was not covered under the Policy. We wish to point out
that Eternal Gardens being the Assured was a party to the Contract and was therefore aware of these
pertinent provisions.

With regard to our acceptance of premiums, these do not connote our approval per se of the insurance
coverage but are held by us in trust for the payor until the prerequisites for insurance coverage shall have
been met. We will however, return all the premiums which have been paid in behalf of John Uy Chuang.

Consequently, Eternal filed a case before the Makati City Regional Trial Court (RTC) for a sum of money against
Philamlife, docketed as Civil Case No. 14736. The trial court decided in favor of Eternal, the dispositive portion of which
reads:

WHEREFORE, premises considered, judgment is hereby rendered in favor of Plaintiff ETERNAL,


against Defendant PHILAMLIFE, ordering the Defendant PHILAMLIFE, to pay the sum of P100,000.00,
representing the proceeds of the Policy of John Uy Chuang, plus legal rate of interest, until fully paid;
and, to pay the sum of P10,000.00 as attorneys fees.

SO ORDERED.

The RTC found that Eternal submitted Chuangs application for insurance which he accomplished before his
death, as testified to by Eternals witness and evidenced by the letter dated December 29, 1982, stating, among others:
Encl: Phil-Am Life Insurance Application Forms & Cert.[10] It further ruled that due to Philamlifes inaction from the
submission of the requirements of the group insurance on December 29, 1982 to Chuangs death on August 2, 1984, as
well as Philamlifes acceptance of the premiums during the same period, Philamlife was deemed to have approved
Chuangs application. The RTC said that since the contract is a group life insurance, once proof of death is submitted,
payment must follow.

Philamlife appealed to the CA, which ruled, thus:

WHEREFORE, the decision of the Regional Trial Court of Makati in Civil Case No. 57810
is REVERSED and SET ASIDE, and the complaint is DISMISSED. No costs.

SO ORDERED.[11]

The CA based its Decision on the factual finding that Chuangs application was not enclosed in Eternals letter
dated December 29, 1982. It further ruled that the non-accomplishment of the submitted application form violated Section
26 of the Insurance Code. Thus, the CA concluded, there being no application form, Chuang was not covered by
Philamlifes insurance.
Hence, we have this petition with the following grounds:

The Honorable Court of Appeals has decided a question of substance, not therefore determined by
this Honorable Court, or has decided it in a way not in accord with law or with the applicable
jurisprudence, in holding that:

I. The application for insurance was not duly submitted to respondent PhilamLife before the
death of John Chuang;

II. There was no valid insurance coverage; and

III. Reversing and setting aside the Decision of the Regional Trial Court dated May 29, 1996.

The Courts Ruling

As a general rule, this Court is not a trier of facts and will not re-examine factual issues raised before the CA and
first level courts, considering their findings of facts are conclusive and binding on this Court. However, such rule is
subject to exceptions, as enunciated in Sampayan v. Court of Appeals:

(1) when the findings are grounded entirely on speculation, surmises or conjectures; (2) when the
inference made is manifestly mistaken, absurd or impossible; (3) when there is grave abuse of discretion;
(4) when the judgment is based on a misapprehension of facts; (5) when the findings of facts are
conflicting; (6) when in making its findings the [CA] went beyond the issues of the case, or its findings
are contrary to the admissions of both the appellant and the appellee; (7) when the findings [of the CA]
are contrary to the trial court; (8) when the findings are conclusions without citation of specific
evidence on which they are based; (9) when the facts set forth in the petition as well as in the petitioners
main and reply briefs are not disputed by the respondent; (10) when the findings of fact are premised on
the supposed absence of evidence and contradicted by the evidence on record; and (11) when the Court of
Appeals manifestly overlooked certain relevant facts not disputed by the parties, which, if properly
considered, would justify a different conclusion.[12] (Emphasis supplied.)

In the instant case, the factual findings of the RTC were reversed by the CA; thus, this Court may review them.

Eternal claims that the evidence that it presented before the trial court supports its contention that it submitted a copy of
the insurance application of Chuang before his death. In Eternals letter dated December 29, 1982, a list of insurable
interests of buyers for October 1982 was attached, including Chuang in the list of new businesses. Eternal added it was
noted at the bottom of said letter that the corresponding Phil-Am Life Insurance Application Forms & Cert. were enclosed
in the letter that was apparently received by Philamlife on January 15, 1983. Finally, Eternal alleged that it provided a
copy of the insurance application which was signed by Chuang himself and executed before his death.

On the other hand, Philamlife claims that the evidence presented by Eternal is insufficient, arguing that Eternal must
present evidence showing that Philamlife received a copy of Chuangs insurance application.

The evidence on record supports Eternals position.


The fact of the matter is, the letter dated December 29, 1982, which Philamlife stamped as received, states that the
insurance forms for the attached list of burial lot buyers were attached to the letter. Such stamp of receipt has the effect of
acknowledging receipt of the letter together with the attachments. Such receipt is an admission by Philamlife against its
own interest.[13] The burden of evidence has shifted to Philamlife, which must prove that the letter did not contain
Chuangs insurance application. However, Philamlife failed to do so; thus, Philamlife is deemed to have received Chuangs
insurance application.

To reiterate, it was Philamlifes bounden duty to make sure that before a transmittal letter is stamped as received, the
contents of the letter are correct and accounted for.

Philamlifes allegation that Eternals witnesses ran out of credibility and reliability due to inconsistencies is groundless. The
trial court is in the best position to determine the reliability and credibility of the witnesses, because it has the opportunity
to observe firsthand the witnesses demeanor, conduct, and attitude. Findings of the trial court on such matters are binding
and conclusive on the appellate court, unless some facts or circumstances of weight and substance have been overlooked,
misapprehended, or misinterpreted,[14] that, if considered, might affect the result of the case.[15]

An examination of the testimonies of the witnesses mentioned by Philamlife, however, reveals no overlooked facts of
substance and value.

Philamlife primarily claims that Eternal did not even know where the original insurance application of Chuang was, as
shown by the testimony of Edilberto Mendoza:
Atty. Arevalo:

Q Where is the original of the application form which is required in case of new coverage?

[Mendoza:]

A It is [a] standard operating procedure for the new client to fill up two copies of this form and the
original of this is submitted to Philamlife together with the monthly remittances and the second copy is
remained or retained with the marketing department of Eternal Gardens.

Atty. Miranda:

We move to strike out the answer as it is not responsive as counsel is merely asking for the location and
does not [ask] for the number of copy.

Atty. Arevalo:

Q Where is the original?

[Mendoza:]

A As far as I remember I do not know where the original but when I submitted with that payment together
with the new clients all the originals I see to it before I sign the transmittal letter the originals are attached
therein.[16]
In other words, the witness admitted not knowing where the original insurance application was, but believed that
the application was transmitted to Philamlife as an attachment to a transmittal letter.

As to the seeming inconsistencies between the testimony of Manuel Cortez on whether one or two insurance
application forms were accomplished and the testimony of Mendoza on who actually filled out the application form, these
are minor inconsistencies that do not affect the credibility of the witnesses. Thus, we ruled in People v. Paredesthat minor
inconsistencies are too trivial to affect the credibility of witnesses, and these may even serve to strengthen their credibility
as these negate any suspicion that the testimonies have been rehearsed.[17]

We reiterated the above ruling in Merencillo v. People:

Minor discrepancies or inconsistencies do not impair the essential integrity of the prosecutions
evidence as a whole or reflect on the witnesses honesty. The test is whether the testimonies agree on
essential facts and whether the respective versions corroborate and substantially coincide with each other
so as to make a consistent and coherent whole.[18]
In the present case, the number of copies of the insurance application that Chuang executed is not at issue, neither is
whether the insurance application presented by Eternal has been falsified. Thus, the inconsistencies pointed out by
Philamlife are minor and do not affect the credibility of Eternals witnesses.

However, the question arises as to whether Philamlife assumed the risk of loss without approving the application.

This question must be answered in the affirmative.

As earlier stated, Philamlife and Eternal entered into an agreement denominated as Creditor Group Life Policy
No. P-1920 dated December 10, 1980. In the policy, it is provided that:

EFFECTIVE DATE OF BENEFIT.

The insurance of any eligible Lot Purchaser shall be effective on the date he contracts a loan with
the Assured. However, there shall be no insurance if the application of the Lot Purchaser is not approved
by the Company.

An examination of the above provision would show ambiguity between its two sentences. The first sentence
appears to state that the insurance coverage of the clients of Eternal already became effective upon contracting a loan with
Eternal while the second sentence appears to require Philamlife to approve the insurance contract before the same can
become effective.

It must be remembered that an insurance contract is a contract of adhesion which must be construed liberally in
favor of the insured and strictly against the insurer in order to safeguard the latters interest. Thus, in Malayan Insurance
Corporation v. Court of Appeals, this Court held that:

Indemnity and liability insurance policies are construed in accordance with the general rule of
resolving any ambiguity therein in favor of the insured, where the contract or policy is prepared by the
insurer. A contract of insurance, being a contract of adhesion, par excellence, any ambiguity therein
should be resolved against the insurer; in other words, it should be construed liberally in favor of the
insured and strictly against the insurer. Limitations of liability should be regarded with extreme jealousy
and must be construed in such a way as to preclude the insurer from noncompliance with its
obligations.[19] (Emphasis supplied.)

In the more recent case of Philamcare Health Systems, Inc. v. Court of Appeals, we reiterated the above ruling,
stating that:

When the terms of insurance contract contain limitations on liability, courts should construe them
in such a way as to preclude the insurer from non-compliance with his obligation. Being a contract of
adhesion, the terms of an insurance contract are to be construed strictly against the party which prepared
the contract, the insurer. By reason of the exclusive control of the insurance company over the terms and
phraseology of the insurance contract, ambiguity must be strictly interpreted against the insurer and
liberally in favor of the insured, especially to avoid forfeiture.[20]

Clearly, the vague contractual provision, in Creditor Group Life Policy No. P-1920 dated December 10, 1980,
must be construed in favor of the insured and in favor of the effectivity of the insurance contract.

On the other hand, the seemingly conflicting provisions must be harmonized to mean that upon a partys purchase
of a memorial lot on installment from Eternal, an insurance contract covering the lot purchaser is created and the same is
effective, valid, and binding until terminated by Philamlife by disapproving the insurance application. The second
sentence of Creditor Group Life Policy No. P-1920 on the Effective Date of Benefit is in the nature of a resolutory
condition which would lead to the cessation of the insurance contract. Moreover, the mere inaction of the insurer on the
insurance application must not work to prejudice the insured; it cannot be interpreted as a termination of the insurance
contract. The termination of the insurance contract by the insurer must be explicit and unambiguous.

As a final note, to characterize the insurer and the insured as contracting parties on equal footing is inaccurate at
best. Insurance contracts are wholly prepared by the insurer with vast amounts of experience in the industry purposefully
used to its advantage. More often than not, insurance contracts are contracts of adhesion containing technical terms and
conditions of the industry, confusing if at all understandable to laypersons, that are imposed on those who wish to avail of
insurance. As such, insurance contracts are imbued with public interest that must be considered whenever the rights and
obligations of the insurer and the insured are to be delineated. Hence, in order to protect the interest of insurance
applicants, insurance companies must be obligated to act with haste upon insurance applications, to either deny or approve
the same, or otherwise be bound to honor the application as a valid, binding, and effective insurance contract.[21]

WHEREFORE, we GRANT the petition. The November 26, 2004 CA Decision in CA-G.R. CV No. 57810
is REVERSED and SET ASIDE. The May 29, 1996 Decision of the Makati City RTC, Branch 138 is MODIFIED.
Philamlife is hereby ORDERED:

(1) To pay Eternal the amount of PhP 100,000 representing the proceeds of the Life Insurance Policy of Chuang;
(2) To pay Eternal legal interest at the rate of six percent (6%) per annum of PhP 100,000 from the time of extra-judicial
demand by Eternal until Philamlifes receipt of the May 29, 1996 RTC Decision on June 17, 1996;
(3) To pay Eternal legal interest at the rate of twelve percent (12%) per annum of PhP 100,000 from June 17, 1996 until
full payment of this award; and
(4) To pay Eternal attorneys fees in the amount of PhP 10,000.

No costs.
DBP POOL OF ACCREDITED G.R. NO. 147039
INSURANCE COMPANIES,
Petitioner, Present:

PANGANIBAN, C.J.
(Chairman)
YNARES-SANTIAGO,
- versus - AUSTRIA-MARTINEZ,
CALLEJO, SR., and
CHICO-NAZARIO, JJ.

RADIO MINDANAO NETWORK,


INC., Promulgated:
Respondent. January 27, 2006
x - - - - - - - - - - - - - - - - - - - - - - - - - - - - - - - - - - - - - - - - - - - - - - - - - - -x

DECISION

AUSTRIA-MARTINEZ, J.:

This refers to the petition for certiorari under Rule 45 of the Rules of Court seeking the review of the
Decision[1] dated November 16, 2000 of the Court of Appeals (CA) in CA-G.R. CV No. 56351, the dispositive portion of
which reads:

Wherefore, premises considered, the appealed Decision of


the Regional Trial Court of Makati City, Branch 138 in Civil Case No. 90-602 is hereby AFFIRMED
with MODIFICATION in that the interest rate is hereby reduced to 6% per annum.

Costs against the defendants-appellants.

SO ORDERED.[2]

The assailed decision originated from Civil Case No. 90-602 filed by Radio Mindanao Network, Inc. (respondent) against
DBP Pool of Accredited Insurance Companies (petitioner) and Provident Insurance Corporation (Provident) for recovery
of insurance benefits. Respondent owns several broadcasting stations all over the country. Provident covered respondents
transmitter equipment and generating set for the amount of P13,550,000.00 under Fire Insurance Policy No. 30354, while
petitioner covered respondents transmitter, furniture, fixture and other transmitter facilities for the amount
of P5,883,650.00 under Fire Insurance Policy No. F-66860.

In the evening of July 27, 1988, respondents radio station located in SSS Building, Bacolod City, was razed by fire
causing damage in the amount of P1,044,040.00. Respondent sought recovery under the two insurance policies but the
claims were denied on the ground that the cause of loss was an excepted risk excluded under condition no. 6 (c) and (d), to
wit:
6. This insurance does not cover any loss or damage occasioned by or through or in consequence, directly
or indirectly, of any of the following consequences, namely:

(c) War, invasion, act of foreign enemy, hostilities, or warlike operations (whether war be declared or
not), civil war.

(d) Mutiny, riot, military or popular rising, insurrection, rebellion, revolution, military or usurped
power.[3]

The insurance companies maintained that the evidence showed that the fire was caused by members of the Communist
Party of the Philippines/New Peoples Army (CPP/NPA); and consequently, denied the claims. Hence, respondent was
constrained to file Civil Case No. 90-602 against petitioner and Provident.
After trial on the merits, the Regional Trial Court of Makati, Branch 138, rendered a decision in favor of respondent. The
dispositive portion of the decision reads:

IN VIEW THEREOF, judgment is rendered in favor of plaintiff. Defendant Provident Insurance


Corporation is directed to pay plaintiff the amount of P450,000.00 representing the value of the destroyed
property insured under its Fire Insurance Policy plus 12% legal interest from March 2, 1990 the date of
the filing of the Complaint. Defendant DBP Pool Accredited Insurance Companies is likewise ordered to
pay plaintiff the sum of P602,600.00 representing the value of the destroyed property under its Fire
Insurance Policy plus 12% legal interest from March 2, 1990.

SO ORDERED.[4]

Both insurance companies appealed from the trial courts decision but the CA affirmed the decision, with the modification
that the applicable interest rate was reduced to 6% per annum. A motion for reconsideration was filed by petitioner DBP
which was denied by the CA per its Resolution dated January 30, 2001.[5]
Hence, herein petition by DBP Pool of Accredited Insurance Companies,[6] with the following assignment of errors:

Assignment of Errors

THE HONORABLE COURT OF APPEALS ERRED WHEN IT HELD THAT THERE WERE NO
SUFFICIENT EVIDENCE SHOWING THAT THE APPROXIMATELY TENTY [sic] (20) ARMED
MEN WHO CUSED [sic] THE FIRE AT RESPONDENTS RMN PROPERTY AT BACOLOD CITY
WERE MEMBERS OF THE CPP-NPA.

THE HONORABLE COURT OF APPEALS ERRED WHEN IT ADJUDGED THAT RESPONDENT


RMN CANNOT BEHELD [sic] FOR DAMAGES AND ATTORNEYS FEES FOR INSTITUTING THE
PRESENT ACTION AGAINST THE PETITIONER UNDER ARTICLES 21, 2208, 2229 AND 2232 OF
THE CIVIL CODE OF THE PHILIPPINES.[7]
Petitioner assails the factual finding of both the trial court and the CA that its evidence failed to support its allegation that
the loss was caused by an excepted risk, i.e., members of the CPP/NPA caused the fire. In upholding respondents claim
for indemnity, the trial court found that:
The only evidence which the Court can consider to determine if the fire was due to the intentional act
committed by the members of the New Peoples Army (NPA), are the testimony [sic] of witnesses Lt. Col.
Nicolas Torres and SPO3 Leonardo Rochar who were admittedly not present when the fire
occurred. Their testimony [sic] was [sic] limited to the fact that an investigation was conducted and in the
course of the investigation they were informed by bystanders that heavily armed men entered the
transmitter house, poured gasoline in (sic) it and then lighted it. After that, they went out shouting
Mabuhay ang NPA (TSN, p. 12., August 2, 1995). The persons whom they investigated and actually saw
the burning of the station were not presented as witnesses.The documentary evidence particularly Exhibits
5 and 5-C do not satisfactorily prove that the author of the burning were members of the NPA. Exhibit 5-
B which is a letter released by the NPA merely mentions some dissatisfaction with the activities of some
people in the media in Bacolod. There was no mention there of any threat on media facilities.[8]

The CA went over the evidence on record and sustained the findings of the trial court, to wit:

To recapitulate, defendants-appellants presented the following to support its claim, to wit: police blotter
of the burning of DYHB, certification of the Negros Occidental Integrated National Police, Bacolod City
regarding the incident, letter of alleged NPA members Celso Magsilang claiming responsibility for the
burning of DYHB, fire investigation report dated July 29, 1988, and the testimonies of Lt. Col. Nicolas
Torres and SFO III Leonardo Rochas. We examined carefully the report on the police blotter of the
burning of DYHB, the certification issued by the Integrated National Police of Bacolod City and the fire
investigation report prepared by SFO III Rochas and there We found that none of them categorically
stated that the twenty (20) armed men which burned DYHB were members of the CPP/NPA. The said
documents simply stated that the said armed men were believed to be or suspected of being members of
the said group.Even SFO III Rochas admitted that he was not sure that the said armed men were members
of the CPP-NPA, thus:

In fact the only person who seems to be so sure that that the CPP-NPA had a hand in the burning
of DYHB was Lt. Col. Nicolas Torres. However, though We found him to be persuasive in his testimony
regarding how he came to arrive at his opinion, We cannot nevertheless admit his testimony as conclusive
proof that the CPP-NPA was really involved in the incident considering that he admitted that he did not
personally see the armed men even as he tried to pursue them. Note that when Lt. Col. Torres was
presented as witness, he was presented as an ordinary witness only and not an expert witness. Hence, his
opinion on the identity or membership of the armed men with the CPP-NPA is not admissible in
evidence.

Anent the letter of a certain Celso Magsilang, who claims to be a member of NPA-NIROC, being an
admission of person which is not a party to the present action, is likewise inadmissible in evidence under
Section 22, Rule 130 of the Rules of Court. The reason being that an admission is competent only when
the declarant, or someone identified in legal interest with him, is a party to the action.[9]

The Court will not disturb these factual findings absent compelling or exceptional reasons. It should be stressed
that a review by certiorari under Rule 45 is a matter of discretion. Under this mode of review, the jurisdiction of the Court
is limited to reviewing only errors of law, not of fact.[10]

Moreover, when supported by substantial evidence, findings of fact of the trial court as affirmed by the CA are
conclusive and binding on the parties,[11] which this Court will not review unless there are exceptional circumstances.
There are no exceptional circumstances in this case that would have impelled the Court to depart from the factual findings
of both the trial court and the CA.

Both the trial court and the CA were correct in ruling that petitioner failed to prove that the loss was caused by an
excepted risk.

Petitioner argues that private respondent is responsible for proving that the cause of the damage/loss is covered by
the insurance policy, as stipulated in the insurance policy, to wit:

Any loss or damage happening during the existence of abnormal conditions (whether physical or
otherwise) which are occasioned by or through in consequence directly or indirectly, of any of the said
occurrences shall be deemed to be loss or damage which is not covered by the insurance, except to the
extent that the Insured shall prove that such loss or damage happened independently of the existence of
such abnormal conditions.

In any action, suit or other proceeding where the Companies allege that by reason of the
provisions of this condition any loss or damage is not covered by this insurance, the burden of proving
that such loss or damage is covered shall be upon the Insured.[12]

An insurance contract, being a contract of adhesion, should be so interpreted as to carry out the purpose for which
the parties entered into the contract which is to insure against risks of loss or damage to the goods. Limitations of liability
should be regarded with extreme jealousy and must be construed in such a way as to preclude the insurer from
noncompliance with its obligations.[13]

The burden of proof contemplated by the aforesaid provision actually refers to the burden of evidence (burden of
going forward).[14] As applied in this case, it refers to the duty of the insured to show that the loss or damage is covered by
the policy. The foregoing clause notwithstanding, the burden of proof still rests upon petitioner to prove that the damage
or loss was caused by an excepted risk in order to escape any liability under the contract.

Burden of proof is the duty of any party to present evidence to establish his claim or defense by the amount of
evidence required by law, which is preponderance of evidence in civil cases. The party, whether plaintiff or defendant,
who asserts the affirmative of the issue has the burden of proof to obtain a favorable judgment. For the plaintiff, the
burden of proof never parts.[15] For the defendant, an affirmative defense is one which is not a denial of an essential
ingredient in the plaintiffs cause of action, but one which, if established, will be a good defense i.e. an avoidance of the
claim.[16]
Particularly, in insurance cases, where a risk is excepted by the terms of a policy which insures against other
perils or hazards, loss from such a risk constitutes a defense which the insurer may urge, since it has not assumed that risk,
and from this it follows that an insurer seeking to defeat a claim because of an exception or limitation in the policy
has the burden of proving that the loss comes within the purview of the exception or limitation set up. If a proof is
made of a loss apparently within a contract of insurance, the burden is upon the insurer to prove that the loss arose from a
cause of loss which is excepted or for which it is not liable, or from a cause which limits its liability. [17]

Consequently, it is sufficient for private respondent to prove the fact of damage or loss. Once respondent makes
out a prima facie case in its favor, the duty or the burden of evidence shifts to petitioner to controvert respondents prima
facie case.[18] In this case, since petitioner alleged an excepted risk, then the burden of evidence shifted to petitioner to
prove such exception. It is only when petitioner has sufficiently proven that the damage or loss was caused by an excepted
risk does the burden of evidence shift back to respondent who is then under a duty of producing evidence to show why
such excepted risk does not release petitioner from any liability. Unfortunately for petitioner, it failed to discharge its
primordial burden of proving that the damage or loss was caused by an excepted risk.

Petitioner however, insists that the evidence on record established the identity of the author of the damage. It
argues that the trial court and the CA erred in not appreciating the reports of witnesses Lt. Col Torres and SFO II Rochar
that the bystanders they interviewed claimed that the perpetrators were members of the CPP/NPA as an exception to the
hearsay rule as part of res gestae.

A witness can testify only to those facts which he knows of his personal knowledge, which means those facts
which are derived from his perception.[19] A witness may not testify as to what he merely learned from others either
because he was told or read or heard the same. Such testimony is considered hearsay and may not be received as proof of
the truth of what he has learned. The hearsay rule is based upon serious concerns about the trustworthiness and reliability
of hearsay evidence inasmuch as such evidence are not given under oath or solemn affirmation and, more importantly,
have not been subjected to cross-examination by opposing counsel to test the perception, memory, veracity and
articulateness of the out-of-court declarant or actor upon whose reliability on which the worth of the out-of-court
statement depends.[20]

Res gestae, as an exception to the hearsay rule, refers to those exclamations and statements made by either the
participants, victims, or spectators to a crime immediately before, during, or after the commission of the crime, when the
circumstances are such that the statements were made as a spontaneous reaction or utterance inspired by the excitement of
the occasion and there was no opportunity for the declarant to deliberate and to fabricate a false statement. The rule in res
gestae applies when the declarant himself did not testify and provided that the testimony of the witness who heard the
declarant complies with the following requisites: (1) that the principal act, the res gestae, be a startling occurrence; (2) the
statements were made before the declarant had the time to contrive or devise a falsehood; and (3) that the statements must
concern the occurrence in question and its immediate attending circumstances.[21]

The Court is not convinced to accept the declarations as part of res gestae. While it may concede that these
statements were made by the bystanders during a startling occurrence, it cannot be said however, that these utterances
were made spontaneously by the bystanders and before they had the time to contrive or devise a falsehood. Both SFO
III Rochar and Lt. Col. Torres received the bystanders statements while they were making their investigations during and
after the fire. It is reasonable to assume that when these statements were noted down, the bystanders already had enough
time and opportunity to mill around, talk to one another and exchange information, not to mention theories and
speculations, as is the usual experience in disquieting situations where hysteria is likely to take place. It cannot therefore
be ascertained whether these utterances were the products of truth. That the utterances may be mere idle talk is not
remote.

At best, the testimonies of SFO III Rochar and Lt. Col. Torres that these statements were made may be considered
as independently relevant statements gathered in the course of their investigation, and are admissible not as to the veracity
thereof but to the fact that they had been thus uttered.[22]

Furthermore, admissibility of evidence should not be equated with its weight and sufficiency. [23] Admissibility of
evidence depends on its relevance and competence, while the weight of evidence pertains to evidence already admitted
and its tendency to convince and persuade.[24] Even assuming that the declaration of the bystanders that it was the
members of the CPP/NPA who caused the fire may be admitted as evidence, it does not follow that such declarations are
sufficient proof. These declarations should be calibrated vis--vis the other evidence on record. And the trial court aptly
noted that there is a need for additional convincing proof, viz.:

The Court finds the foregoing to be insufficient to establish that the cause of the fire was the intentional
burning of the radio facilities by the rebels or an act of insurrection, rebellion or usurped power. Evidence
that persons who burned the radio facilities shouted Mabuhay ang NPA does not furnish logical
conclusion that they are member [sic] of the NPA or that their act was an act of rebellion or
insurrection. Additional convincing proof need be submitted. Defendants failed to discharge their
responsibility to present adequate proof that the loss was due to a risk excluded.[25]

While the documentary evidence presented by petitioner, i.e., (1) the police blotter; (2) the certification from the
Bacolod Police Station; and (3) the Fire Investigation Report may be considered exceptions to the hearsay rule, being
entries in official records, nevertheless, as noted by the CA, none of these documents categorically stated that the
perpetrators were members of the CPP/NPA.[26] Rather, it was stated in the police blotter that: a group of persons
accompanied by one (1) woman all believed to be CPP/NPA more or less 20 persons suspected to be CPP/NPA,[27] while
the certification from the Bacolod Police station stated that some 20 or more armed men believed to be members of the
New Peoples Army NPA,[28] and the fire investigation report concluded that (I)t is therefore believed by this Investigating
Team that the cause of the fire is intentional, and the armed men suspected to be members of the CPP/NPA where (sic)
the ones responsible [29] All these documents show that indeed, the suspected executor of the fire were believed to be
members of the CPP/NPA. But suspicion alone is not sufficient, preponderance of evidence being the quantum of proof.

All told, the Court finds no reason to grant the present petition.

WHEREFORE, the petition is DISMISSED. The Court of Appeals Decision dated November 16, 2000 and
Resolution dated January 30, 2001 rendered in CA-G.R. CV No. 56351 are AFFIRMED in toto.
G.R. No. L-47593 December 29, 1943

THE INSULAR LIFE ASSURANCE CO., LTD., petitioner,


vs.
SERAFIN D. FELICIANO ET AL., respondents.

OZAETA, J.:

In a four-to-three decision promulgated on September 13, 1941, 1 this Court affirmed the judgment of the Court
of Appeals in favor of the respondents and against the petitioner for the sum of P25,000, representing the
value of two insurance policies issued by the petitioner on the life of Evaristo Feliciano. A motion to reconsider
and set aside said decision has been filed by the petitioner, and both parties have submitted exhaustive and
luminous written arguments in support of their respective contentions.

The facts of the case are set forth in the majority and dissenting opinions heretofore handed down by this
Court, the salient points of which may be briefly restated as follows:

Evaristo Feliciano, who died on September 29, 1935, was suffering with advanced pulmonary tuberculosis
when he signed his applications for insurance with the petitioner on October 12, 1934. On that same date
Doctor Trepp, who had taken X-ray pictures of his lungs, informed the respondent Dr. Serafin D. Feliciano,
brother of Evaristo, that the latter "was already in a very serious ad practically hopeless condition."
Nevertheless the question contained in the application "Have you ever suffered from any ailment or disease
of the lungs, pleurisy, pneumonia or asthma?" appears to have been answered , "No" And above the
signature of the applicant, following the answers to the various questions propounded to him, is the following
printed statement:1awphil.net

I declare on behalf of myself and of any person who shall have or claim any interest in any policy
issued hereunder, that each of the above answers is full, complete and true, and that to the best of my
knowledge and belief I am a proper subject for life insurance. (Exhibit K.)

The false answer above referred to, as well as the others, was written by the Company's soliciting agent
Romulo M. David, in collusion with the medical examiner Dr. Gregorio Valdez, for the purpose of securing the
Company's approval of the application so that the policy to be issued thereon might be credited to said agent in
connection with the inter-provincial contest which the Company was then holding among its soliciting agents to
boost the sales of its policies. Agent David bribed Medical Examiner Valdez with money which the former
borrowed from the applicant's mother by way of advanced payment on the premium, according to the finding of
the Court of Appeals. Said court also found that before the insured signed the application he, as well as the
members of his family, told the agent and the medical examiner that he had been sick and coughing for some
time and that he had gone three times to the Santol Sanatorium and had X-ray pictures of his lungs taken; but
that in spite of such information the agent and the medical examiner told them that the applicant was a fit
subject for insurance.

Each of the policies sued upon contains the following stipulations:

This policy and the application herefor constitute the entire contract between the parties hereto. . . .
Only the President, or the Manager, acting jointly with the Secretary or Assistant Secretary (and then
only in writing signed by them) have power in behalf of the Company to issue permits, or to modify this
or any contract, or to extend the same time for making any premium payment, and the Company shall
not be bound by any promise or representation heretofore or hereafter given by any person other than
the above-named officials, and by them only in writing and signed conjointly as stated.

The application contains, among others, the following statements:


18. I [the applicant] hereby declare that all the above statements and answers as well as all those
that I may make to the Company's Medical Examiner in continuation of this application, to be complete,
true and correct to the best of my knowledge and belief, and I hereby agree as follows:

1. That his declaration, with the answers to be given by me to the Medical Examiner, shall be the basis
of the policy and form part of same.

xxx xxx xxx

3. That the said policy shall not take effect until the first premium has been paid and the policy has
been delivered to and accepted by me, while I am in good health.

4. That the agent taking this application has no authority to make, modify or discharge contracts, or to
waive any of the Company's rights or requirements.

5. My acceptance of any policy issued on this application will constitute a ratification by me of any
corrections in or additions to this application made by the Company in the space provided "For Home
Office Corrections or Additions Only." I agree that photographic copy of this applications as corrected or
added to shall constitute sufficient notice to me of the changes made. (Emphasis added.)

The petitioner insists that upon the facts of the case the policies in question are null and void ab initio and that
all that the respondents are entitled to is the refund of the premiums paid thereon. After a careful re-
examination of the facts and the law, we are persuaded that petitioner's contention is correct. To the reasons
adduced in the dissenting opinion heretofore published, we only desire to add the following considerations:

When Evaristo Feliciano, the applicant for insurance, signed the application in blank and authorized the
soliciting agent and/or medical examiner of the Company to write the answers for him, he made them his own
agents for that purpose, and he was responsible for their acts in that connection. If they falsified the answers
for him, he could not evade the responsibility for he falsification. He was not supposed to sign the application in
blank. He knew that the answers to the questions therein contained would be "the basis of the policy," and for
that every reason he was required with his signature to vouch for truth thereof.

Moreover, from the facts of the case we cannot escape the conclusion that the insured acted in connivance
with the soliciting agent and the medical examiner of the Company in accepting the policies in question. Above
the signature of the applicant is the printed statement or representation: " . . . I am a proper subject for life
insurance." In another sheet of the same application and above another signature of the applicant was also
printed this statement: "That the said policy shall not take effect until he first premium has been paid and the
policy as been delivered to and accepted by me, while I am in good health." When the applicant signed the
application he was "having difficulty in breathing, . . . with a very high fever." He had gone three times to the
Santol Sanatorium and had X-ray pictures taken of his lungs. He therefore knew that he was not "a proper
subject for life insurance." When he accepted the policy, he knew that he was not in good health.
Nevertheless, he not only accepted the first policy of P20,000 but then and there applied for and later accepted
another policy of P5,000.

We cannot bring ourselves to believe that the insured did not take the trouble to read the answers contained in
the photostatic copy of the application attached to and made a part of the policy before he accepted it and paid
the premium thereon. He must have notice that the answers to the questions therein asked concerning his
clinical history were false, and yet he accepted the first policy and applied for another. In any event, he
obligated himself to read the policy when he subscribed to this statement: "My acceptance of any policy issued
on this application will constitute a ratification by me of any corrections in or additions to this application made
by the Company . . ." By accepting the policy he became charged with knowledge of its contents, whether he
actually read it or not. He could not ostrich-like hide his head from it in order to avoid his part of the bargain
and at the same time claim the benefit thereof. He knew, or was chargeable with knowledge, from the very
terms of the two policies sued upon (one of which is printed in English and the other in Spanish) that the
soliciting agent and the medical examiner had no power to bind the Company by any verbal promise or oral
representation. The insured, therefore, had no right to rely and we cannot believe he relied in good faith
upon the oral representation. The insured, therefore, had no right to rely and we cannot believe he relied in
good faith upon the oral representation of said agent and medical examiner that he (the applicant) was a fit
subject for insurance notwithstanding that he had been and was still suffering with advanced pulmonary
tuberculosis.

From all the facts and circumstances of this case, we are constrained to conclude that the insured was a
coparticipant, and coresponsible with Agent David and Medical Examiner Valdez, in the fraudulent
procurement of the policies in question and that by reason thereof said policies are void ab initio.

Wheretofore, the motion for reconsideration is sustained and the judgment of the Court of Appeals is hereby
reversed. Let another judgment be entered in favor of the respondents and against the petitioner for the refund
of the premiums amounting to P1,389, with legal interest thereon from the date of the complaint, and without
any finding as to costs.

Moran, Paras and Bocobo, JJ., concur.

Separate Opinions

YULO, C.J., concurring:

I can find no quarrel with the legal considerations and conclusions set forth in the original decision promulgated
by this Court. As general rules of law they find full support not only in reason and in logic, but also in simple
human sense of justice. More so, modern and complicated practices attendant to the ever growing trade in life
insurance demand the strictest accountability by insurance companies for acts of their authorized agents. In
this way only may the State afford reasonable protection to the unwary public from abuse by such
organizations as may be found to be of questionable moral standards.

But a careful consideration of the evidentiary facts as set forth in the decision of the Court of Appeals leads me
to conclude that the ends of justice would not be serve by the application to the present case of the rules so
enunciated. Rather, to serve the ends of justice the case of the respondents should be removed from the
protection of such rules.

The subject of the insurance policies under consideration is the life of the assured. It is contended by his
beneficiaries that they took these policies on the basis of a life expectancy of a person gravely stricken with
tuberculosis. They have consistently made protestations that they had so informed the agents of the insurance
company. But the policies were issued upon the life of the assured, as a perfectly normal and healthy person.
The error is vital and goes to the very existence of the contract itself. Who is responsible for the error?

The direct cause, of course, is the false recitals in the application for insurance. While it is true that it was the
agents of the insurance company who filled out such application, yet it was the assured who, by signing the
application in blank, made it possible for the said agents to procure the issuance of the policies on the basis of
false information, in order to suit their own purposes. Upon the admitted facts, I am of the opinion that in justice
and in equity, the responsibility for the falsifications made by the insurance agents in the preparation of the
insurance application should be laid at the door of the assured and his beneficiaries.

I vote with the majority in granting the motion for reconsideration and in reversing the decision under review.
HONTIVEROS, J., dissenting:

The reasons given in the dissenting opinion in this case, as published in the Official Gazette of October 4, 1941
(pp. 2847 to 2855), supplemented by those in the resolution of the majority on the motion for reconsideration,
do not seem to me sufficient to overthrow the decision rendered by the Court of First Instance, confirmed by
the Court of Appeals, and sustained by this Supreme court in its decision of September 18, 1941. The alleged
connivance between the insured Evaristo Feliciano, the agent Romulo M. David, and the medical examiner Dr.
Gregorio Valdez not only does not clearly appear of record, but on the contrary is denied in the finding of facts
of the court a quo and of the Court of Appeals which cannot be reviewed or altered by this Court.

The mere fact that the insured signed at the bottom of the application for insurance when some of its lines
intended for answers to certain questions were still in blank, answers which according to the evidence and to
the findings of the two inferior courts he had grounds to believe will be made in accordance with the
information which he and his family had given to agent David and to Dr. Valdez, does not convert these two
persons into agents of the insured in a way as to make the latter responsible for the acts of the former. That
the photostatic copies of said forms which are attached to the policies object of this case are almost illegible, is
a fact which should be taken into account, together with the other fact that Evaristo Feliciano does not know
English, the language in which those documents are written. In support of this dissenting opinion, the following
authorities may be cited:

The mere failure of the insured to inform himself of the insertion of false answers in the application
which has been filled out by the agent of the insurer does not convict him of lack of good faith. (Vol. 5,
Cooley's Briefs on Insurance, 2nd Ed., p. 4136, and many cases cited.)

The insured is not chargeable with such negligence as will render him liable for false answers inserted
by the agent merely because he signed the application in blank and trusted the agent to fill out by the
agent, without reading it. (Id., p. 4136, and many cases cited.)

An illiterate person or one who does not understand the English language (as is the case with Evaristo
Feliciano) is not guilty of inexcusable negligence in failing to read the application or having it read to
him, nor can it be said that such person deliberately made a false statement because he did not read
over the application. (81 ALR 865, 866, W. 117 ALR 796.)

Nor can it be said that the assured, who has fully, frankly, truthfully, and in good faith answered all the
required questions, is guilty of negligence in signing, without reading, the application which is thereupon
prepared by the agent. He is justified in assuming that the agent, has, with equal good faith, truthfully
recorded the answers give. He may well say to the Company: 'You accredited this man to me as your
representative, and I signed the application thus prepared by him, relying upon the character which you
gave him, when you commissioned him to come to me as your agent. If he acted dishonestly in the
matter, you, and not I, must suffer the consequences . . .! (Germania Life Ins. Co. vs. Lunkeheimer
[1931] Ind., 538; 26 N. E., 1052)

In such case the acceptance of the policy, with this application attached, does not require the insured to
institute an investigation into its provisions, or the conditions upon which is was issued, to ascertain
whether the agent has acted in good faith, since, under such circumstances, the insured may rely upon
the presumption that he has been honestly dealt with the insurer. (Otto vs. Hartford Ins. Co., 38 Minn.,
423).

Besides, the principles that the insured is not bound to know the contents of the application, and may
rely on the agent's assurances that his answers have been correctly written will, of course, apply with
special force where the insured is illiterate and unable to read, or is ignorant of the language. (Vol. 5,
Cooley's Briefs on Insurance, 2nd Ed. p. 4138, cases cited.)
And also where the photostatic copies of the application embodied in the policy are practically illegible,
the insured is not bound to know the contents of the application. (New York Ins. Co. vs. Holpem D.C.
57 Fed. 2nd, 200).

According to the great weight of authority, if an agent of the insurer, after obtaining from an applicant for
insurance a correct and truthful answer to interrogations contained in the application for insurance,
without knowledge of the applicant fills in false answers, either fraudulently or otherwise, the insurer
cannot assert the falsity of such answers as a defense to the liability on the policy and this is generally
without regard to the subject matter of the answers or the nature of the agent's duties or limitations on
his authority, at least if not brought to the attention of the applicant. It is equally well settled that if a
correct representation is made in a written application, or the insurance agent issuing the policy is
appraised of the true facts concerning the matter in question, as for instance the title to the insured
premises, but the agent inserts an incorrect statement in the policy, the insurer cannot rely upon the
error in avoidance of its liability". Home Ins. Co. vs. Mendenhall, 154 Ill., 452, 45 NE., 1078, 36 LRA.,
374; Phoenix Ins. Co. vs. Tucker, 92 Ill., 64, 34 Am Rep., 106; Commercial Ins. Co. vs. Spanknoble, 52
Ill., 53, 4 Am. Report, 582; Young vs. Hartford F. Ins. Co. 45 Iowa, 377, 24 Am. Rep., 754; Welsh vs.
London Assur. 151 Pa., 607, 25 A, 142, 21 Am St. Rep., 726 (Taken from Am Juris. on Insurance
Vol. 29, par. 843).

An insured may be justified in signing an application in blank at the request of the insurer's agent, who
agrees to fill it in from data furnished by the insured or from an old application. In fact, an insurer cannot
urge the falsity of representations contained in the policy issued, or in the application, where such
representations were inserted therein, either by the company or its agent, after the application was
signed, without the knowledge or consent of the insured, who has made no such representations.
(Couch on Insurance, Vol. 4, par. 842 b.)

I believe that the motion for reconsideration presented in this case should be denied, not only because of the
weighty reasons relied upon in the decision which it attacks, but also because a dangerous precedent would
otherwise be established, for, with the destruction of the confidence which the public has hitherto reposed in
the duly accredited agents of insurance companies and in their examining physicians, this branch of the
economic life of the people will have to be unfavorably affected.
[G.R. No. 125678. March 18, 2002]

PHILAMCARE HEALTH SYSTEMS, INC., petitioner, vs. COURT OF APPEALS and JULITA
TRINOS, respondents.

DECISION
YNARES-SANTIAGO, J.:

Ernani Trinos, deceased husband of respondent Julita Trinos, applied for a health care coverage with petitioner
Philamcare Health Systems, Inc. In the standard application form, he answered no to the following question:

Have you or any of your family members ever consulted or been treated for high blood pressure, heart trouble, diabetes,
cancer, liver disease, asthma or peptic ulcer? (If Yes, give details).[1]

The application was approved for a period of one year from March 1, 1988 to March 1, 1989. Accordingly, he was
issued Health Care Agreement No. P010194. Under the agreement, respondents husband was entitled to avail of
hospitalization benefits, whether ordinary or emergency, listed therein. He was also entitled to avail of out-patient benefits
such as annual physical examinations, preventive health care and other out-patient services.
Upon the termination of the agreement, the same was extended for another year from March 1, 1989 to March 1,
1990, then from March 1, 1990 to June 1, 1990. The amount of coverage was increased to a maximum sum of P75,000.00
per disability.[2]
During the period of his coverage, Ernani suffered a heart attack and was confined at the Manila Medical Center
(MMC) for one month beginning March 9, 1990. While her husband was in the hospital, respondent tried to claim the
benefits under the health care agreement. However, petitioner denied her claim saying that the Health Care Agreement
was void. According to petitioner, there was a concealment regarding Ernanis medical history. Doctors at the MMC
allegedly discovered at the time of Ernanis confinement that he was hypertensive, diabetic and asthmatic, contrary to his
answer in the application form. Thus, respondent paid the hospitalization expenses herself, amounting to about
P76,000.00.
After her husband was discharged from the MMC, he was attended by a physical therapist at home. Later, he was
admitted at the Chinese General Hospital. Due to financial difficulties, however, respondent brought her husband home
again. In the morning of April 13, 1990, Ernani had fever and was feeling very weak. Respondent was constrained to
bring him back to the Chinese General Hospital where he died on the same day.
On July 24, 1990, respondent instituted with the Regional Trial Court of Manila, Branch 44, an action for damages
against petitioner and its president, Dr. Benito Reverente, which was docketed as Civil Case No. 90-53795. She asked for
reimbursement of her expenses plus moral damages and attorneys fees. After trial, the lower court ruled against
petitioners, viz:

WHEREFORE, in view of the forgoing, the Court renders judgment in favor of the plaintiff Julita Trinos, ordering:

1. Defendants to pay and reimburse the medical and hospital coverage of the late Ernani Trinos in the amount of
P76,000.00 plus interest, until the amount is fully paid to plaintiff who paid the same;

2. Defendants to pay the reduced amount of moral damages of P10,000.00 to plaintiff;

3. Defendants to pay the reduced amount of P10,000.00 as exemplary damages to plaintiff;

4. Defendants to pay attorneys fees of P20,000.00, plus costs of suit.

SO ORDERED.[3]
On appeal, the Court of Appeals affirmed the decision of the trial court but deleted all awards for damages and
absolved petitioner Reverente.[4] Petitioners motion for reconsideration was denied.[5]Hence, petitioner brought the instant
petition for review, raising the primary argument that a health care agreement is not an insurance contract; hence the
incontestability clause under the Insurance Code[6]does not apply.
Petitioner argues that the agreement grants living benefits, such as medical check-ups and hospitalization which a
member may immediately enjoy so long as he is alive upon effectivity of the agreement until its expiration one-year
thereafter. Petitioner also points out that only medical and hospitalization benefits are given under the agreement without
any indemnification, unlike in an insurance contract where the insured is indemnified for his loss. Moreover, since Health
Care Agreements are only for a period of one year, as compared to insurance contracts which last longer, [7] petitioner
argues that the incontestability clause does not apply, as the same requires an effectivity period of at least two
years. Petitioner further argues that it is not an insurance company, which is governed by the Insurance Commission, but a
Health Maintenance Organization under the authority of the Department of Health.
Section 2 (1) of the Insurance Code defines a contract of insurance as an agreement whereby one undertakes for a
consideration to indemnify another against loss, damage or liability arising from an unknown or contingent event. An
insurance contract exists where the following elements concur:
1. The insured has an insurable interest;
2. The insured is subject to a risk of loss by the happening of the designated peril;
3. The insurer assumes the risk;
4. Such assumption of risk is part of a general scheme to distribute actual losses among a large group of persons
bearing a similar risk; and
5. In consideration of the insurers promise, the insured pays a premium.[8]
Section 3 of the Insurance Code states that any contingent or unknown event, whether past or future, which may
damnify a person having an insurable interest against him, may be insured against. Every person has an insurable interest
in the life and health of himself. Section 10 provides:

Every person has an insurable interest in the life and health:

(1) of himself, of his spouse and of his children;

(2) of any person on whom he depends wholly or in part for education or support, or in whom he has a
pecuniary interest;

(3) of any person under a legal obligation to him for the payment of money, respecting property or service, of
which death or illness might delay or prevent the performance; and

(4) of any person upon whose life any estate or interest vested in him depends.

In the case at bar, the insurable interest of respondents husband in obtaining the health care agreement was his own
health. The health care agreement was in the nature of non-life insurance, which is primarily a contract of
indemnity.[9] Once the member incurs hospital, medical or any other expense arising from sickness, injury or other
stipulated contingent, the health care provider must pay for the same to the extent agreed upon under the contract.
Petitioner argues that respondents husband concealed a material fact in his application. It appears that in the
application for health coverage, petitioners required respondents husband to sign an express authorization for any person,
organization or entity that has any record or knowledge of his health to furnish any and all information relative to any
hospitalization, consultation, treatment or any other medical advice or examination.[10] Specifically, the Health Care
Agreement signed by respondents husband states:

We hereby declare and agree that all statement and answers contained herein and in any addendum annexed to this
application are full, complete and true and bind all parties in interest under the Agreement herein applied for, that there
shall be no contract of health care coverage unless and until an Agreement is issued on this application and the full
Membership Fee according to the mode of payment applied for is actually paid during the lifetime and good health of
proposed Members; that no information acquired by any Representative of PhilamCare shall be binding upon PhilamCare
unless set out in writing in the application; that any physician is, by these presents, expressly authorized to disclose or
give testimony at anytime relative to any information acquired by him in his professional capacity upon any question
affecting the eligibility for health care coverage of the Proposed Members and that the acceptance of any Agreement
issued on this application shall be a ratification of any correction in or addition to this application as stated in the space for
Home Office Endorsement.[11] (Underscoring ours)

In addition to the above condition, petitioner additionally required the applicant for authorization to inquire about the
applicants medical history, thus:

I hereby authorize any person, organization, or entity that has any record or knowledge of my health and/or that of
__________ to give to the PhilamCare Health Systems, Inc. any and all information relative to any hospitalization,
consultation, treatment or any other medical advice or examination. This authorization is in connection with the
application for health care coverage only. A photographic copy of this authorization shall be as valid as the
original.[12] (Underscoring ours)

Petitioner cannot rely on the stipulation regarding Invalidation of agreement which reads:

Failure to disclose or misrepresentation of any material information by the member in the application or medical
examination, whether intentional or unintentional, shall automatically invalidate the Agreement from the very beginning
and liability of Philamcare shall be limited to return of all Membership Fees paid. An undisclosed or misrepresented
information is deemed material if its revelation would have resulted in the declination of the applicant by Philamcare or
the assessment of a higher Membership Fee for the benefit or benefits applied for.[13]

The answer assailed by petitioner was in response to the question relating to the medical history of the applicant. This
largely depends on opinion rather than fact, especially coming from respondents husband who was not a medical
doctor. Where matters of opinion or judgment are called for, answers made in good faith and without intent to deceive
will not avoid a policy even though they are untrue.[14]Thus,

(A)lthough false, a representation of the expectation, intention, belief, opinion, or judgment of the insured will not avoid
the policy if there is no actual fraud in inducing the acceptance of the risk, or its acceptance at a lower rate of premium,
and this is likewise the rule although the statement is material to the risk, if the statement is obviously of the foregoing
character, since in such case the insurer is not justified in relying upon such statement, but is obligated to make further
inquiry. There is a clear distinction between such a case and one in which the insured is fraudulently and intentionally
states to be true, as a matter of expectation or belief, that which he then knows, to be actually untrue, or the impossibility
of which is shown by the facts within his knowledge, since in such case the intent to deceive the insurer is obvious and
amounts to actual fraud.[15] (Underscoring ours)

The fraudulent intent on the part of the insured must be established to warrant rescission of the insurance
contract.[16] Concealment as a defense for the health care provider or insurer to avoid liability is an affirmative defense and
the duty to establish such defense by satisfactory and convincing evidence rests upon the provider or insurer. In any case,
with or without the authority to investigate, petitioner is liable for claims made under the contract. Having assumed a
responsibility under the agreement, petitioner is bound to answer the same to the extent agreed upon. In the end, the
liability of the health care provider attaches once the member is hospitalized for the disease or injury covered by the
agreement or whenever he avails of the covered benefits which he has prepaid.
Under Section 27 of the Insurance Code, a concealment entitles the injured party to rescind a contract of
insurance. The right to rescind should be exercised previous to the commencement of an action on the contract. [17] In this
case, no rescission was made. Besides, the cancellation of health care agreements as in insurance policies require the
concurrence of the following conditions:

1. Prior notice of cancellation to insured;

2. Notice must be based on the occurrence after effective date of the policy of one or more of the grounds mentioned;

3. Must be in writing, mailed or delivered to the insured at the address shown in the policy;
4. Must state the grounds relied upon provided in Section 64 of the Insurance Code and upon request of insured, to furnish
facts on which cancellation is based.[18]

None of the above pre-conditions was fulfilled in this case. When the terms of insurance contract contain limitations
on liability, courts should construe them in such a way as to preclude the insurer from non-compliance with his
obligation.[19] Being a contract of adhesion, the terms of an insurance contract are to be construed strictly against the party
which prepared the contract the insurer.[20] By reason of the exclusive control of the insurance company over the terms
and phraseology of the insurance contract, ambiguity must be strictly interpreted against the insurer and liberally in favor
of the insured, especially to avoid forfeiture.[21] This is equally applicable to Health Care Agreements. The phraseology
used in medical or hospital service contracts, such as the one at bar, must be liberally construed in favor of the subscriber,
and if doubtful or reasonably susceptible of two interpretations the construction conferring coverage is to be adopted, and
exclusionary clauses of doubtful import should be strictly construed against the provider.[22]
Anent the incontestability of the membership of respondents husband, we quote with approval the following findings
of the trial court:

(U)nder the title Claim procedures of expenses, the defendant Philamcare Health Systems Inc. had twelve months from
the date of issuance of the Agreement within which to contest the membership of the patient if he had previous ailment of
asthma, and six months from the issuance of the agreement if the patient was sick of diabetes or hypertension. The periods
having expired, the defense of concealment or misrepresentation no longer lie.[23]

Finally, petitioner alleges that respondent was not the legal wife of the deceased member considering that at the time
of their marriage, the deceased was previously married to another woman who was still alive. The health care agreement
is in the nature of a contract of indemnity. Hence, payment should be made to the party who incurred the expenses. It is
not controverted that respondent paid all the hospital and medical expenses. She is therefore entitled to
reimbursement. The records adequately prove the expenses incurred by respondent for the deceaseds hospitalization,
medication and the professional fees of the attending physicians.[24]
WHEREFORE, in view of the foregoing, the petition is DENIED. The assailed decision of the Court of Appeals
dated December 14, 1995 is AFFIRMED.
G.R. No. 105135 June 22, 1995

SUNLIFE ASSURANCE COMPANY OF CANADA, petitioner,


vs.
The Hon. COURT OF APPEALS and Spouses ROLANDO and BERNARDA BACANI, respondents.

QUIASON, J.:

This is a petition for review for certiorari under Rule 45 of the Revised Rules of Court to reverse and set aside
the Decision dated February 21, 1992 of the Court of Appeals in CA-G.R. CV No. 29068, and its Resolution
dated April 22, 1992, denying reconsideration thereof.

We grant the petition.

On April 15, 1986, Robert John B. Bacani procured a life insurance contract for himself from petitioner. He was
issued Policy No. 3-903-766-X valued at P100,000.00, with double indemnity in case of accidental death. The
designated beneficiary was his mother, respondent Bernarda Bacani.

On June 26, 1987, the insured died in a plane crash. Respondent Bernarda Bacani filed a claim with petitioner,
seeking the benefits of the insurance policy taken by her son. Petitioner conducted an investigation and its
findings prompted it to reject the claim.

In its letter, petitioner informed respondent Bernarda Bacani, that the insured did not disclose material facts
relevant to the issuance of the policy, thus rendering the contract of insurance voidable. A check representing
the total premiums paid in the amount of P10,172.00 was attached to said letter.

Petitioner claimed that the insured gave false statements in his application when he answered the following
questions:

5. Within the past 5 years have you:

a) consulted any doctor or other health practitioner?

b) submitted to:

EGG?
X-rays?
blood tests?
other tests?

c) attended or been admitted to any hospital or other medical facility?

6. Have you ever had or sought advice for:

xxx xxx xxx

b) urine, kidney or bladder disorder? (Rollo, p. 53)


The deceased answered question No. 5(a) in the affirmative but limited his answer to a consultation with a
certain Dr. Reinaldo D. Raymundo of the Chinese General Hospital on February 1986, for cough and flu
complications. The other questions were answered in the negative (Rollo, p. 53).

Petitioner discovered that two weeks prior to his application for insurance, the insured was examined and
confined at the Lung Center of the Philippines, where he was diagnosed for renal failure. During his
confinement, the deceased was subjected to urinalysis, ultra-sonography and hematology tests.

On November 17, 1988, respondent Bernarda Bacani and her husband, respondent Rolando Bacani, filed an
action for specific performance against petitioner with the Regional Trial Court, Branch 191, Valenzuela, Metro
Manila. Petitioner filed its answer with counterclaim and a list of exhibits consisting of medical records
furnished by the Lung Center of the Philippines.

On January 14, 1990, private respondents filed a "Proposed Stipulation with Prayer for Summary Judgment"
where they manifested that they "have no evidence to refute the documentary evidence of
concealment/misrepresentation by the decedent of his health condition (Rollo, p. 62).

Petitioner filed its Request for Admissions relative to the authenticity and due execution of several documents
as well as allegations regarding the health of the insured. Private respondents failed to oppose said request or
reply thereto, thereby rendering an admission of the matters alleged.

Petitioner then moved for a summary judgment and the trial court decided in favor of private respondents. The
dispositive portion of the decision is reproduced as follows:

WHEREFORE, judgment is hereby rendered in favor of the plaintiffs and against the defendant,
condemning the latter to pay the former the amount of One Hundred Thousand Pesos
(P100,000.00) the face value of insured's Insurance Policy No. 3903766, and the Accidental
Death Benefit in the amount of One Hundred Thousand Pesos (P100,000.00) and further sum
of P5,000.00 in the concept of reasonable attorney's fees and costs of suit.

Defendant's counterclaim is hereby Dismissed (Rollo, pp. 43-44).

In ruling for private respondents, the trial court concluded that the facts concealed by the insured were made in
good faith and under a belief that they need not be disclosed. Moreover, it held that the health history of the
insured was immaterial since the insurance policy was "non-medical".

Petitioner appealed to the Court of Appeals, which affirmed the decision of the trial court. The appellate court
ruled that petitioner cannot avoid its obligation by claiming concealment because the cause of death was
unrelated to the facts concealed by the insured. It also sustained the finding of the trial court that matters
relating to the health history of the insured were irrelevant since petitioner waived the medical examination
prior to the approval and issuance of the insurance policy. Moreover, the appellate court agreed with the trial
court that the policy was "non-medical" (Rollo, pp. 4-5).

Petitioner's motion for reconsideration was denied; hence, this petition.

II

We reverse the decision of the Court of Appeals.

The rule that factual findings of the lower court and the appellate court are binding on this Court is not absolute
and admits of exceptions, such as when the judgment is based on a misappreciation of the facts (Geronimo v.
Court of Appeals, 224 SCRA 494 [1993]).
In weighing the evidence presented, the trial court concluded that indeed there was concealment and
misrepresentation, however, the same was made in "good faith" and the facts concealed or misrepresented
were irrelevant since the policy was "non-medical". We disagree.

Section 26 of The Insurance Code is explicit in requiring a party to a contract of insurance to communicate to
the other, in good faith, all facts within his knowledge which are material to the contract and as to which he
makes no warranty, and which the other has no means of ascertaining. Said Section provides:

A neglect to communicate that which a party knows and ought to communicate, is called
concealment.

Materiality is to be determined not by the event, but solely by the probable and reasonable influence of the
facts upon the party to whom communication is due, in forming his estimate of the disadvantages of the
proposed contract or in making his inquiries (The Insurance Code, Sec. 31).

The terms of the contract are clear. The insured is specifically required to disclose to the insurer matters
relating to his health.

The information which the insured failed to disclose were material and relevant to the approval and issuance of
the insurance policy. The matters concealed would have definitely affected petitioner's action on his
application, either by approving it with the corresponding adjustment for a higher premium or rejecting the
same. Moreover, a disclosure may have warranted a medical examination of the insured by petitioner in order
for it to reasonably assess the risk involved in accepting the application.

In Vda. de Canilang v. Court of Appeals, 223 SCRA 443 (1993), we held that materiality of the information
withheld does not depend on the state of mind of the insured. Neither does it depend on the actual or physical
events which ensue.

Thus, "goad faith" is no defense in concealment. The insured's failure to disclose the fact that he was
hospitalized for two weeks prior to filing his application for insurance, raises grave doubts about his bonafides.
It appears that such concealment was deliberate on his part.

The argument, that petitioner's waiver of the medical examination of the insured debunks the materiality of the
facts concealed, is untenable. We reiterate our ruling in Saturnino v. Philippine American Life Insurance
Company, 7 SCRA 316 (1963), that " . . . the waiver of a medical examination [in a non-medical insurance
contract] renders even more material the information required of the applicant concerning previous condition of
health and diseases suffered, for such information necessarily constitutes an important factor which the insurer
takes into consideration in deciding whether to issue the policy or not . . . "

Moreover, such argument of private respondents would make Section 27 of the Insurance Code, which allows
the injured party to rescind a contract of insurance where there is concealment, ineffective (See Vda. de
Canilang v. Court of Appeals, supra).

Anent the finding that the facts concealed had no bearing to the cause of death of the insured, it is well settled
that the insured need not die of the disease he had failed to disclose to the insurer. It is sufficient that his non-
disclosure misled the insurer in forming his estimates of the risks of the proposed insurance policy or in making
inquiries (Henson v. The Philippine American Life Insurance Co., 56 O.G. No. 48 [1960]).

We, therefore, rule that petitioner properly exercised its right to rescind the contract of insurance by reason of
the concealment employed by the insured. It must be emphasized that rescission was exercised within the two-
year contestability period as recognized in Section 48 of The Insurance Code.

WHEREFORE, the petition is GRANTED and the Decision of the Court of Appeals is REVERSED and SET
ASIDE.
G.R. No. 92492 June 17, 1993

THELMA VDA. DE CANILANG, petitioner,


vs.
HON. COURT OF APPEALS and GREAT PACIFIC LIFE ASSURANCE CORPORATION, respondents.

Simeon C. Sato for petitioner.

FELICIANO, J.:

On 18 June 1982, Jaime Canilang consulted Dr. Wilfredo B. Claudio and was diagnosed as suffering from
"sinus tachycardia." The doctor prescribed the following fro him: Trazepam, a tranquilizer; and Aptin, a beta-
blocker drug. Mr. Canilang consulted the same doctor again on 3 August 1982 and this time was found to have
"acute bronchitis."

On next day, 4 August 1982, Jaime Canilang applied for a "non-medical" insurance policy with respondent
Great Pacific Life Assurance Company ("Great Pacific") naming his wife, Thelma Canilang, as his
beneficiary.1 Jaime Canilang was issued ordinary life insurance Policy No. 345163, with the face value of
P19,700, effective as of 9 August 1982.

On 5 August 1983, Jaime Canilang died of "congestive heart failure," "anemia," and "chronic
anemia."2 Petitioner, widow and beneficiary of the insured, filed a claim with Great Pacific which the insurer
denied on 5 December 1983 upon the ground that the insured had concealed material information from it.

Petitioner then filed a complaint against Great Pacific with the Insurance Commission for recovery of the
insurance proceeds. During the hearing called by the Insurance Commissioner, petitioner testified that she was
not aware of any serious illness suffered by her late husband3 and that, as far as she knew, her husband had
died because of a kidney disorder.4 A deposition given by Dr. Wilfredo Claudio was presented by petitioner.
There Dr. Claudio stated that he was the family physician of the deceased Jaime Canilang5 and that he had
previously treated him for "sinus tachycardia" and "acute bronchitis."6 Great Pacific for its part presented Dr.
Esperanza Quismorio, a physician
and a medical underwriter working for Great Pacific.7 She testified that the deceased's insurance application
had been approved on the basis of his medical declaration.8 She explained that as a rule, medical
examinations are required only in cases where the applicant has indicated in his application for insurance
coverage that he has previously undergone medical consultation and hospitalization.9

In a decision dated 5 November 1985, Insurance Commissioner Armando Ansaldo ordered Great Pacific to
pay P19,700 plus legal interest and P2,000.00 as attorney's fees after holding that:

1. the ailment of Jaime Canilang was not so serious that, even if it had been disclosed, it would
not have affected Great Pacific's decision to insure him;

2. Great Pacific had waived its right to inquire into the health condition of the applicant by the
issuance of the policy despite the lack of answers to "some of the pertinent questions" in the
insurance application;

3. there was no intentional concealment on the part of the insured Jaime Canilang as he had
thought that he was merely suffering from a minor ailment and simple cold; 10 and

4. Batas Pambansa Blg. 847 which voids an insurance contract, whether or not concealment
was intentionally made, was not applicable to Canilang's case as that law became effective only
on 1 June 1985.

On appeal by Great Pacific, the Court of Appeals reversed and set aside the decision of the Insurance
Commissioner and dismissed Thelma Canilang's complaint and Great Pacific's counterclaim. The Court of
Appealed found that the use of the word "intentionally" by the Insurance Commissioner in defining and
resolving the issue agreed upon by the parties at pre-trial before the Insurance Commissioner was not
supported by the evidence; that the issue agreed upon by the parties had been whether the deceased insured,
Jaime Canilang, made a material concealment as the state of his health at the time of the filing of insurance
application, justifying respondent's denial of the claim. The Court of Appeals also found that the failure of
Jaime Canilang to disclose previous medical consultation and treatment constituted material information which
should have been communicated to Great Pacific to enable the latter to make proper inquiries. The Court of
Appeals finally held that the Ng Gan Zee case which had involved misrepresentation was not applicable in
respect of the case at bar which involves concealment.

Petitioner Thelma Canilang is now before this Court on a Petition for Review on Certiorari alleging that:

1. . . . the Honorable Court of Appeals, speaking with due respect, erred in not holding that the
issue in the case agreed upon between the parties before the Insurance Commission is whether
or not Jaime Canilang "intentionally" made material concealment in stating his state of health;

2. . . . at any rate, the non-disclosure of certain facts about his previous health conditions does
not amount to fraud and private respondent is deemed to have waived inquiry thereto. 11

The medical declaration which was set out in the application for insurance executed by Jaime Canilang read as
follows:

MEDICAL DECLARATION

I hereby declare that:

(1) I have not been confined in any hospital, sanitarium or infirmary, nor receive any medical or
surgical advice/attention within the last five (5) years.

(2) I have never been treated nor consulted a physician for a heart condition, high blood
pressure, cancer, diabetes, lung, kidney, stomach disorder, or any other physical impairment.

(3) I am, to the best of my knowledge, in good health.

EXCEPTIONS:

____________________________________________________________________________
____

GENERAL DECLARATION

I hereby declare that all the foregoing answers and statements are complete, true and correct. I
hereby agree that if there be any fraud or misrepresentation in the above statements material to
the risk, the INSURANCE COMPANY upon discovery within two (2) years from the effective
date of insurance shall have the right to declare such insurance null and void. That the liabilities
of the Company under the said Policy/TA/Certificate shall accrue and begin only from the date
of commencement of risk stated in the Policy/TA/Certificate, provided that the first premium is
paid and the Policy/TA/Certificate is delivered to, and accepted by me in person, when I am in
actual good health.

Signed at Manila his 4th day of August, 1992.

Illegibl
e




Signat
ure of
Applic
ant. 12

We note that in addition to the negative statements made by Mr. Canilang in paragraph 1 and 2 of the medical
declaration, he failed to disclose in the appropriate space, under the caption "Exceptions," that he had twice
consulted Dr. Wilfredo B. Claudio who had found him to be suffering from "sinus tachycardia" and "acute
bronchitis."

The relevant statutory provisions as they stood at the time Great Pacific issued the contract of insurance and at
the time Jaime Canilang died, are set out in P.D. No. 1460, also known as the Insurance Code of 1978, which
went into effect on 11 June 1978. These provisions read as follows:

Sec. 26. A neglect to communicate that which a party knows and ought to communicate, is
called a concealment.

xxx xxx xxx

Sec. 28. Each party to a contract of insurance must communicate to the other, in good faith, all
factors within his knowledge which are material to the contract and as to which he makes no
warranty, and which the other has not the means of ascertaining. (Emphasis supplied)

Under the foregoing provisions, the information concealed must be information which the concealing party
knew and "ought to [have] communicate[d]," that is to say, information which was "material to the contract."
The test of materiality is contained in Section 31 of the Insurance Code of 1978 which reads:

Sec. 31. Materially is to be determined not by the event, but solely by the probable and
reasonable influence of the facts upon the party to whom the communication is due, in forming
his estimate of the disadvantages of the proposed contract, or in making his inquiries.
(Emphasis supplied)

"Sinus tachycardia" is considered present "when the heart rate exceeds 100 beats per minute." 13 The
symptoms of this condition include pounding in the chest and sometimes faintness and weakness of the
person affected. The following elaboration was offered by Great Pacific and set out by the Court of Appeals in
its Decision:

Sinus tachycardia is defined as sinus-initiated; heart rate faster than 100 beats per minute.
(Harrison' s Principles of Internal Medicine, 8th ed. [1978], p. 1193.) It is, among others, a
common reaction to heart disease, including myocardial infarction, and heart failure per se.
(Henry J.L. Marriot, M.D., Electrocardiography, 6th ed., [1977], p. 127.) The medication
prescribed by Dr. Claudio for treatment of Canilang's ailment on June 18, 1982, indicates the
condition that said physician was trying to manage. Thus, he prescribed Trazepam, (Philippine
Index of Medical Specialties (PIMS), Vol. 14, No. 3, Dec. 1985, p. 112) which is anti-anxiety,
anti-convulsant, muscle-relaxant; and Aptin, (Idem, p. 36) a cardiac drug, for palpitations and
nervous heart. Such treatment could have been a very material information to the insurer in
determining the action to be take on Canilang's application for life insurance coverage. 14

We agree with the Court of Appeals that the information which Jaime Canilang failed to disclose was material
to the ability of Great Pacific to estimate the probable risk he presented as a subject of life insurance. Had
Canilang disclosed his visits to his doctor, the diagnosis made and medicines prescribed by such doctor, in the
insurance application, it may be reasonably assumed that Great Pacific would have made further inquiries and
would have probably refused to issue a non-medical insurance policy or, at the very least, required a higher
premium for the same coverage. 15 The materiality of the information withheld by Great Pacific did not depend
upon the state of mind of Jaime Canilang. A man's state of mind or subjective belief is not capable of proof in
our judicial process, except through proof of external acts or failure to act from which inferences as to his
subjective belief may be reasonably drawn. Neither does materiality depend upon the actual or physical events
which ensue. Materiality relates rather to the "probable and reasonable influence of the facts" upon the party to
whom the communication should have been made, in assessing the risk involved in making or omitting to
make further inquiries and in accepting the application for insurance; that "probable and reasonable influence
of the facts" concealed must, of course, be determined objectively, by the judge ultimately.

The insurance Great Pacific applied for was a "non-medical" insurance policy. In Saturnino v. Philippine-
American Life Insurance Company, 16 this Court held that:

. . . if anything, the waiver of medical examination [in a non-medical insurance contract] renders
even more material the information required of the applicant concerning previous condition of
health and diseases suffered, for such information necessarily constitutes an important factor
which the insurer takes into consideration in deciding whether to issue the policy or not . . .
. 17 (Emphasis supplied)

The Insurance Commissioner had also ruled that the failure of Great Pacific to convey certain information to
the insurer was not "intentional" in nature, for the reason that Jaime Canilang believed that he was suffering
from minor ailment like a common cold. Section 27 of the Insurance Code of 1978 as it existed from 1974 up to
1985, that is, throughout the time range material for present purposes, provided that:

Sec. 27. A concealment entitles the injured party to rescind a contract of insurance.

The preceding statute, Act No. 2427, as it stood from 1914 up to 1974, had provided:

Sec. 26. A concealment, whether intentional or unintentional, entitles the injured party to rescind
a contract of insurance. (Emphasis supplied)

Upon the other hand, in 1985, the Insurance Code of 1978 was amended by
B.P. Blg. 874. This subsequent statute modified Section 27 of the Insurance Code of 1978 so as to read as
follows:

Sec. 27. A concealment whether intentional or unintentional entitles the injured party to rescind
a contract of insurance. (Emphasis supplied)

The unspoken theory of the Insurance Commissioner appears to have been that by deleting the phrase
"intentional or unintentional," the Insurance Code of 1978 (prior to its amendment by B.P. Blg. 874) intended to
limit the kinds of concealment which generate a right to rescind on the part of the injured party to "intentional
concealments." This argument is not persuasive. As a simple matter of grammar, it may be noted that
"intentional" and "unintentional" cancel each other out. The net result therefore of the phrase "whether
intentional or unitentional" is precisely to leave unqualified the term "concealment." Thus, Section 27 of the
Insurance Code of 1978 is properly read as referring to "any concealment" without regard to whether such
concealment is intentional or unintentional. The phrase "whether intentional or unintentional" was in fact
superfluous. The deletion of the phrase "whether intentional or unintentional" could not have had the effect of
imposing an affirmative requirement that a concealment must be intentional if it is to entitle the injured party to
rescind a contract of insurance. The restoration in 1985 by B.P. Blg. 874 of the phrase "whether intentional or
unintentional" merely underscored the fact that all throughout (from 1914 to 1985), the statute did not require
proof that concealment must be "intentional" in order to authorize rescission by the injured party.

In any case, in the case at bar, the nature of the facts not conveyed to the insurer was such that the failure to
communicate must have been intentional rather than merely inadvertent. For Jaime Canilang could not have
been unaware that his heart beat would at times rise to high and alarming levels and that he had consulted a
doctor twice in the two (2) months before applying for non-medical insurance. Indeed, the last medical
consultation took place just the day before the insurance application was filed. In all probability, Jaime
Canilang went to visit his doctor precisely because of the discomfort and concern brought about by his
experiencing "sinus tachycardia."

We find it difficult to take seriously the argument that Great Pacific had waived inquiry into the concealment by
issuing the insurance policy notwithstanding Canilang's failure to set out answers to some of the questions in
the insurance application. Such failure precisely constituted concealment on the part of Canilang. Petitioner's
argument, if accepted, would obviously erase Section 27 from the Insurance Code of 1978.

It remains only to note that the Court of Appeals finding that the parties had not agreed in the pretrial before
the Insurance Commission that the relevant issue was whether or not Jaime Canilang
had intentionally concealed material information from the insurer, was supported by the evidence of
record, i.e., the Pre-trial Order itself dated 17 October 1984 and the Minutes of the Pre-trial Conference dated
15 October 1984, which "readily shows that the word "intentional" does not appear in the statement or
definition of the issue in the said Order and Minutes." 18

WHEREFORE, the Petition for Review is DENIED for lack of merit and the Decision of the Court of Appeals
dated 16 October 1989 in C.A.-G.R. SP No. 08696 is hereby AFFIRMED. No pronouncement as to the costs.
PRUDENTIAL GUARANTEE and G.R. No. 151890
ASSURANCE INC.,

Petitioner,

- versus -

TRANS-ASIA SHIPPING LINES, INC.,

Respondent.
x- - - - - - - - - - - - - - - - - - - - - - - - - x

TRANS-ASIA SHIPPING LINES, G.R. No. 151991


INC.,

Petitioner, Present:
PANGANIBAN, C.J.
Chairperson,
YNARES-SANTIAGO,
AUSTRIA-MARTINEZ,
CALLEJO, SR., and
CHICO-NAZARIO, JJ.
- versus

Promulgated:

PRUDENTIAL GUARANTEE and June 20, 2006


ASSURANCE INC.,

Respondent.

x----------------------------------------------------------------------------------------x

DECISION

CHICO-NAZARIO, J:

This is a consolidation of two separate Petitions for Review on Certiorari filed by petitioner Prudential Guarantee
and Assurance, Inc. (PRUDENTIAL) in G.R. No. 151890 and Trans-Asia Shipping Lines, Inc. (TRANS-ASIA) in G.R.
No. 151991, assailing the Decision[1] dated 6 November 2001 of the Court of Appeals in CA G.R. CV No. 68278, which
reversed the Judgment[2] dated 6 June 2000 of the Regional Trial Court (RTC), Branch 13, Cebu City in Civil Case No.
CEB-20709. The 29 January 2002Resolution[3] of the Court of Appeals, denying PRUDENTIALs Motion for
Reconsideration and TRANS-ASIAs Partial Motion for Reconsideration of the 6 November 2001Decision, is likewise
sought to be annulled and set aside.

The Facts

The material antecedents as found by the court a quo and adopted by the appellate court are as follows:

Plaintiff [TRANS-ASIA] is the owner of the vessel M/V Asia Korea. In consideration of payment of
premiums, defendant [PRUDENTIAL] insured M/V Asia Korea for loss/damage of the hull and
machinery arising from perils, inter alia, of fire and explosion for the sum of P40 Million, beginning
[from] the period [of] July 1, 1993 up to July 1, 1994. This is evidenced by Marine Policy No.
MH93/1363 (Exhibits A to A-11). On October 25, 1993, while the policy was in force, a fire broke out
while [M/V Asia Korea was] undergoing repairs at the port of Cebu. On October 26, 1993 plaintiff
[TRANS-ASIA] filed its notice of claim for damage sustained by the vessel. This is evidenced by a
letter/formal claim of even date (Exhibit B). Plaintiff [TRANS-ASIA] reserved its right to subsequently
notify defendant [PRUDENTIAL] as to the full amount of the claim upon final survey and determination
by average adjuster Richard Hogg International (Phil.) of the damage sustained by reason of fire. An
adjusters report on the fire in question was submitted by Richard Hogg International together with the U-
Marine Surveyor Report (Exhibits 4 to 4-115).

On May 29, 1995[,] plaintiff [TRANS-ASIA] executed a document denominated Loan and Trust receipt,
a portion of which read (sic):

Received from Prudential Guarantee and Assurance, Inc., the sum of PESOS THREE
MILLION ONLY (P3,000,000.00) as a loan without interest under Policy No. MH
93/1353 [sic], repayable only in the event and to the extent that any net recovery is made
by Trans-Asia Shipping Corporation, from any person or persons, corporation or
corporations, or other parties, on account of loss by any casualty for which they may be
liable occasioned by the 25 October 1993: Fire on Board. (Exhibit 4)

In a letter dated 21 April 1997 defendant [PRUDENTIAL] denied plaintiffs claim (Exhibit 5). The letter
reads:

After a careful review and evaluation of your claim arising from the above-captioned
incident, it has been ascertained that you are in breach of policy conditions, among them
WARRANTED VESSEL CLASSED AND CLASS MAINTAINED. Accordingly, we regret
to advise that your claim is not compensable and hereby DENIED.

This was followed by defendants letter dated 21 July 1997 requesting the return or
payment of the P3,000,000.00 within a period of ten (10) days from receipt of the letter
(Exhibit 6).[4]

Following this development, on 13 August 1997, TRANS-ASIA filed a Complaint[5] for Sum of Money against
PRUDENTIAL with the RTC of Cebu City, docketed as Civil Case No. CEB-20709, wherein TRANS-ASIA sought the
amount of P8,395,072.26 from PRUDENTIAL, alleging that the same represents the balance of the indemnity due upon
the insurance policy in the total amount of P11,395,072.26. TRANS-ASIA similarly sought interest at 42% per annum
citing Section 243[6] of Presidential Decreee No. 1460, otherwise known as the Insurance Code, as amended.
In its Answer,[7] PRUDENTIAL denied the material allegations of the Complaint and interposed the defense that
TRANS-ASIA breached insurance policy conditions, in particular: WARRANTED VESSEL CLASSED AND CLASS
MAINTAINED. PRUDENTIAL further alleged that it acted as facts and law require and incurred no liability to TRANS-
ASIA; that TRANS-ASIA has no cause of action; and, that its claim has been effectively waived and/or abandoned, or it
is estopped from pursuing the same. By way of a counterclaim, PRUDENTIAL sought a refund of P3,000,000.00, which
it allegedly advanced to TRANS-ASIA by way of a loan without interest and without prejudice to the final evaluation of
the claim, including the amounts of P500,000.00, for survey fees and P200,000.00, representing attorneys fees.

The Ruling of the Trial Court

On 6 June 2000, the court a quo rendered Judgment[8] finding for (therein defendant) PRUDENTIAL. It ruled that
a determination of the parties liabilities hinged on whether TRANS-ASIA violated and breached the policy conditions on
WARRANTED VESSEL CLASSED AND CLASS MAINTAINED. It interpreted the provision to mean that TRANS-
ASIA is required to maintain the vessel at a certain class at all times pertinent during the life of the policy. According to
the court a quo, TRANS-ASIA failed to prove compliance of the terms of the warranty, the violation thereof entitled
PRUDENTIAL, the insured party, to rescind the contract.[9]

Further, citing Section 107[10] of the Insurance Code, the court a quo ratiocinated that the concealment made by
TRANS-ASIA that the vessel was not adequately maintained to preserve its class was a material concealment sufficient to
avoid the policy and, thus, entitled the injured party to rescind the contract. The court a quo found merit in
PRUDENTIALs contention that there was nothing in the adjustment of the particular average submitted by the adjuster
that would show that TRANS-ASIA was not in breach of the policy. Ruling on the denominated loan and trust receipt, the
court a quo said that in substance and in form, the same is a receipt for a loan. It held that if TRANS-ASIA intended to
receive the amount of P3,000,000.00 as advance payment, it should have so clearly stated as such.

The court a quo did not award PRUDENTIALs claim for P500,000.00, representing expert survey fees on the
ground of lack of sufficient basis in support thereof. Neither did it award attorneys fees on the rationalization that the
instant case does not fall under the exceptions stated in Article 2208 [11] of the Civil Code. However, the court a
quo granted PRUDENTIALs counterclaim stating that there is factual and legal basis for TRANS-ASIA to return the
amount of P3,000,000.00 by way of loan without interest.

The decretal portion of the Judgment of the RTC reads:

WHEREFORE, judgment is hereby rendered DISMISSING the complaint for its failure to prove
a cause of action.

On defendants counterclaim, plaintiff is directed to return the sum of P3,000,000.00 representing


the loan extended to it by the defendant, within a period of ten (10) days from and after this judgment
shall have become final and executory.[12]
The Ruling of the Court of Appeals

On appeal by TRANS-ASIA, the Court of Appeals, in its assailed Decision of 6 November 2001, reversed the 6
June 2000 Judgment of the RTC.

On the issue of TRANS-ASIAs alleged breach of warranty of the policy condition CLASSED AND CLASS
MAINTAINED, the Court of Appeals ruled that PRUDENTIAL, as the party asserting the non-compensability of the loss
had the burden of proof to show that TRANS-ASIA breached the warranty, which burden it failed to
discharge. PRUDENTIAL cannot rely on the lack of certification to the effect that TRANS-ASIA was CLASSED AND
CLASS MAINTAINED as its sole basis for reaching the conclusion that the warranty was breached. The Court of
Appeals opined that the lack of a certification does not necessarily mean that the warranty was breached by TRANS-
ASIA. Instead, the Court of Appeals considered PRUDENTIALs admission that at the time the insurance contract was
entered into between the parties, the vessel was properly classed by Bureau Veritas, a classification society recognized by
the industry. The Court of Appeals similarly gave weight to the fact that it was the responsibility of Richards Hogg
International (Phils.) Inc., the average adjuster hired by PRUDENTIAL, to secure a copy of such certification to support
its conclusion that mere absence of a certification does not warrant denial of TRANS-ASIAs claim under the insurance
policy.

In the same token, the Court of Appeals found the subject warranty allegedly breached by TRANS-ASIA to be a
rider which, while contained in the policy, was inserted by PRUDENTIAL without the intervention of TRANS-ASIA. As
such, it partakes of a nature of a contract dadhesion which should be construed against PRUDENTIAL, the party which
drafted the contract. Likewise, according to the Court of Appeals, PRUDENTIALs renewal of the insurance policy
from noon of 1 July 1994 to noon of 1 July 1995, and then again, until noon of 1 July 1996 must be deemed a waiver by
PRUDENTIAL of any breach of warranty committed by TRANS-ASIA.

Further, the Court of Appeals, contrary to the ruling of the court a quo, interpreted the transaction between
PRUDENTIAL and TRANS-ASIA as one of subrogation, instead of a loan. The Court of Appeals concluded that
TRANS-ASIA has no obligation to pay back the amount of P3,000.000.00 to PRUDENTIAL based on its finding that the
aforesaid amount was PRUDENTIALs partial payment to TRANS-ASIAs claim under the policy. Finally, the Court of
Appeals denied TRANS-ASIAs prayer for attorneys fees, but held TRANS-ASIA entitled to double interest on the policy
for the duration of the delay of payment of the unpaid balance, citing Section 244[13] of the Insurance Code.

Finding for therein appellant TRANS-ASIA, the Court of Appeals ruled in this wise:
WHEREFORE, the foregoing consideration, We find for Appellant. The instant appeal is
ALLOWED and the Judgment appealed from REVERSED. The P3,000,000.00 initially paid by appellee
Prudential Guarantee Assurance Incorporated to appellant Trans-Asia and covered by a Loan and Trust
Receipt dated 29 May 1995 is HELD to be in partial settlement of the loss suffered by appellant and
covered by Marine Policy No. MH93/1363 issued by appellee. Further, appellee is hereby ORDERED to
pay appellant the additional amount of P8,395,072.26 representing the balance of the loss suffered by the
latter as recommended by the average adjuster Richard Hogg International (Philippines) in its Report,
with double interest starting from the time Richard Hoggs Survey Report was completed, or on 13 August
1996, until the same is fully paid.
All other claims and counterclaims are hereby DISMISSED.

All costs against appellee.[14]

Not satisfied with the judgment, PRUDENTIAL and TRANS-ASIA filed a Motion for Reconsideration and Partial
Motion for Reconsideration thereon, respectively, which motions were denied by the Court of Appeals in the Resolution
dated 29 January 2002.

The Issues

Aggrieved, PRUDENTIAL filed before this Court a Petition for Review, docketed as G.R. No. 151890, relying on the
following grounds, viz:

I.

THE AWARD IS GROSSLY UNCONSCIONABLE.

II.

THE COURT OF APPEALS ERRED IN HOLDING THAT THERE WAS NO VIOLATION BY


TRANS-ASIA OF A MATERIAL WARRANTY, NAMELY, WARRANTY CLAUSE NO. 5, OF THE
INSURANCE POLICY.

III.

THE COURT OF APPEALS ERRED IN HOLDING THAT PRUDENTIAL, AS INSURER HAD THE
BURDEN OF PROVING THAT THE ASSURED, TRANS-ASIA, VIOLATED A MATERIAL
WARRANTY.

IV.

THE COURT OF APPEALS ERRED IN HOLDING THAT THE WARRANTY CLAUSE EMBODIED
IN THE INSURANCE POLICY CONTRACT WAS A MERE RIDER.

V.

THE COURT OF APPEALS ERRED IN HOLDING THAT THE ALLEGED RENEWALS OF THE
POLICY CONSTITUTED A WAIVER ON THE PART OF PRUDENTIAL OF THE BREACH OF THE
WARRANTY BY TRANS-ASIA.
VI.

THE COURT OF APPEALS ERRED IN HOLDING THAT THE LOAN AND TRUST RECEIPT
EXECUTED BY TRANS-ASIA IS AN ADVANCE ON THE POLICY, THUS CONSTITUTING
PARTIAL PAYMENT THEREOF.

VII.

THE COURT OF APPEALS ERRED IN HOLDING THAT THE ACCEPTANCE BY PRUDENTIAL


OF THE FINDINGS OF RICHARDS HOGG IS INDICATIVE OF A WAIVER ON THE PART OF
PRUDENTIAL OF ANY VIOLATION BY TRANS-ASIA OF THE WARRANTY.

VIII.

THE COURT OF APPEALS ERRRED (sic) IN REVERSING THE TRIAL COURT, IN FINDING
THAT PRUDENTIAL UNJUSTIFIABLY REFUSED TO PAY THE CLAIM AND IN ORDERING
PRUDENTIAL TO PAY TRANS-ASIA P8,395,072.26 PLUS DOUBLE INTEREST FROM 13
AUGUST 1996, UNTIL [THE] SAME IS FULLY PAID.[15]

Similarly, TRANS-ASIA, disagreeing in the ruling of the Court of Appeals filed a Petition for Review docketed as G.R.
No. 151991, raising the following grounds for the allowance of the petition, to wit:

I.
THE HONORABLE COURT OF APPEALS ERRED IN NOT AWARDING ATTORNEYS FEES TO
PETITIONER TRANS-ASIA ON THE GROUND THAT SUCH CAN ONLY BE AWARDED IN THE
CASES ENUMERATED IN ARTICLE 2208 OF THE CIVIL CODE, AND THERE BEING NO BAD
FAITH ON THE PART OF RESPONDENT PRUDENTIAL IN DENYING HEREIN PETITIONER
TRANS-ASIAS INSURANCE CLAIM.

II.

THE DOUBLE INTEREST REFERRED TO IN THE DECISION DATED 06 NOVEMBER


2001 SHOULD BE CONSTRUED TO MEAN DOUBLE INTEREST BASED ON THE LEGAL
INTEREST OF 12%, OR INTEREST AT THE RATE OF 24% PER ANNUM.[16]

In our Resolution of 2 December 2002, we granted TRANS-ASIAs Motion for Consolidation[17] of G.R. Nos. 151890 and
151991;[18] hence, the instant consolidated petitions.

In sum, for our main resolution are: (1) the liability, if any, of PRUDENTIAL to TRANS-ASIA arising from the subject
insurance contract; (2) the liability, if any, of TRANS-ASIA to PRUDENTIAL arising from the transaction between the
parties as evidenced by a document denominated as Loan and Trust Receipt, dated 29 May 1995; and (3) the amount of
interest to be imposed on the liability, if any, of either or both parties.

Ruling of the Court


Prefatorily, it must be emphasized that in a petition for review, only questions of law, and not questions of fact, may be
raised.[19] This rule may be disregarded only when the findings of fact of the Court of Appeals are contrary to the findings
and conclusions of the trial court, or are not supported by the evidence on record.[20] In the case at bar, we find an
incongruence between the findings of fact of the Court of Appeals and the court a quo, thus, in our determination of the
issues, we are constrained to assess the evidence adduced by the parties to make appropriate findings of facts as are
necessary.

I.

A. PRUDENTIAL failed to establish that TRANS-ASIA violated and breached the policy condition on WARRANTED
VESSEL CLASSED AND CLASS MAINTAINED, as contained in the subject insurance contract.

In resisting the claim of TRANS-ASIA, PRUDENTIAL posits that TRANS-ASIA violated an express and material
warranty in the subject insurance contract, i.e., Marine Insurance Policy No. MH93/1363, specifically Warranty Clause
No. 5 thereof, which stipulates that the insured vessel, M/V ASIA KOREA is required to be CLASSED AND CLASS
MAINTAINED. According to PRUDENTIAL, on 25 October 1993, or at the time of the occurrence of the fire, M/V
ASIA KOREA was in violation of the warranty as it was not CLASSED AND CLASS MAINTAINED. PRUDENTIAL
submits that Warranty Clause No. 5 was a condition precedent to the recovery of TRANS-ASIA under the policy, the
violation of which entitled PRUDENTIAL to rescind the contract under Sec. 74[21] of the Insurance Code.

The warranty condition CLASSED AND CLASS MAINTAINED was explained by PRUDENTIALs Senior Manager of
the Marine and Aviation Division, Lucio Fernandez. The pertinent portions of his testimony on direct examination is
reproduced hereunder, viz:

ATTY. LIM

Q Please tell the court, Mr. Witness, the result of the evaluation of this claim, what final action was
taken?

A It was eventually determined that there was a breach of the policy condition, and basically there is a
breach of policy warranty condition and on that basis the claim was denied.

Q To refer you (sic) the policy warranty condition, I am showing to you a policy here marked as Exhibits
1, 1-A series, please point to the warranty in the policy which you said was breached or violated
by the plaintiff which constituted your basis for denying the claim as you testified.

A Warranted Vessel Classed and Class Maintained.

ATTY. LIM

Witness pointing, Your Honor, to that portion in Exhibit 1-A which is the second page of the
policy below the printed words: Clauses, Endorsements, Special Conditions and Warranties,
below this are several typewritten clauses and the witness pointed out in particular the clause
reading: Warranted Vessel Classed and Class Maintained.

COURT
Q Will you explain that particular phrase?

A Yes, a warranty is a condition that has to be complied with by the insured. When we say a class
warranty, it must be entered in the classification society.

COURT

Slowly.

WITNESS
(continued)

A A classification society is an organization which sets certain standards for a vessel to maintain in order
to maintain their membership in the classification society. So, if they failed to meet that standard,
they are considered not members of that class, and thus breaching the warranty, that requires
them to maintain membership or to maintain their class on that classification society. And it is not
sufficient that the member of this classification society at the time of a loss, their membership
must be continuous for the whole length of the policy such that during the effectivity of the
policy, their classification is suspended, and then thereafter, they get reinstated, that again still a
breach of the warranty that they maintained their class (sic).Our maintaining team membership in
the classification society thereby maintaining the standards of the vessel (sic).

ATTY. LIM
Q Can you mention some classification societies that you know?
A Well we have the Bureau Veritas, American Bureau of Shipping, D&V Local Classification Society,
The Philippine Registration of Ships Society, China Classification, NKK and Company
Classification Society, and many others, we have among others, there are over 20 worldwide. [22]

At the outset, it must be emphasized that the party which alleges a fact as a matter of defense has the burden of
proving it. PRUDENTIAL, as the party which asserted the claim that TRANS-ASIA breached the warranty in the policy,
has the burden of evidence to establish the same. Hence, on the part of PRUDENTIAL lies the initiative to show proof in
support of its defense; otherwise, failing to establish the same, it remains self-serving. Clearly, if no evidence on the
alleged breach of TRANS-ASIA of the subject warranty is shown, a fortiori, TRANS-ASIA would be successful in
claiming on the policy. It follows that PRUDENTIAL bears the burden of evidence to establish the fact of breach.

In our rule on evidence, TRANS-ASIA, as the plaintiff below, necessarily has the burden of proof to show proof
of loss, and the coverage thereof, in the subject insurance policy. However, in the course of trial in a civil case, once
plaintiff makes out a prima facie case in his favor, the duty or the burden of evidence shifts to defendant to controvert
plaintiffs prima facie case, otherwise, a verdict must be returned in favor of plaintiff.[23] TRANS-ASIA was able to
establish proof of loss and the coverage of the loss, i.e., 25 October 1993: Fire on Board. Thereafter, the burden of
evidence shifted to PRUDENTIAL to counter TRANS-ASIAs case, and to prove its special and affirmative defense that
TRANS-ASIA was in violation of the particular condition on CLASSED AND CLASS MAINTAINED.

We sustain the findings of the Court of Appeals that PRUDENTIAL was not successful in discharging the burden
of evidence that TRANS-ASIA breached the subject policy condition on CLASSED AND CLASS MAINTAINED.
Foremost, PRUDENTIAL, through the Senior Manager of its Marine and Aviation Division, Lucio Fernandez,
made a categorical admission that at the time of the procurement of the insurance contract in July 1993, TRANS-ASIAs
vessel, M/V Asia Korea was properly classed by Bureau Veritas, thus:

Q Kindly examine the records particularly the policy, please tell us if you know whether M/V Asia Korea
was classed at the time (sic) policy was procured perthe (sic) insurance was procured that Exhibit
1 on 1st July 1993 (sic).

WITNESS

A I recall that they were classed.

ATTY. LIM

Q With what classification society?

A I believe with Bureau Veritas.[24]

As found by the Court of Appeals and as supported by the records, Bureau Veritas is a classification society
recognized in the marine industry. As it is undisputed that TRANS-ASIA was properly classed at the time the contract of
insurance was entered into, thus, it becomes incumbent upon PRUDENTIAL to show evidence that the status of TRANS-
ASIA as being properly CLASSED by Bureau Veritas had shifted in violation of the warranty. Unfortunately,
PRUDENTIAL failed to support the allegation.

We are in accord with the ruling of the Court of Appeals that the lack of a certification in PRUDENTIALs records
to the effect that TRANS-ASIAs M/V Asia Korea was CLASSED AND CLASS MAINTAINED at the time of the
occurrence of the fire cannot be tantamount to the conclusion that TRANS-ASIA in fact breached the warranty contained
in the policy. With more reason must we sustain the findings of the Court of Appeals on the ground that as admitted by
PRUDENTIAL, it was likewise the responsibility of the average adjuster, Richards Hogg International (Phils.), Inc., to
secure a copy of such certification, and the alleged breach of TRANS-ASIA cannot be gleaned from the average adjusters
survey report, or adjustment of particular average per M/V Asia Korea of the 25 October 1993 fire on board.

We are not unmindful of the clear language of Sec. 74 of the Insurance Code which provides that, the violation of
a material warranty, or other material provision of a policy on the part of either party thereto, entitles the other to
rescind. It is generally accepted that [a] warranty is a statement or promise set forth in the policy, or by reference
incorporated therein, the untruth or non-fulfillment of which in any respect, and without reference to whether the insurer
was in fact prejudiced by such untruth or non-fulfillment, renders the policy voidable by the insurer.[25] However, it is
similarly indubitable that for the breach of a warranty to avoid a policy, the same must be duly shown by the party
alleging the same. We cannot sustain an allegation that is unfounded. Consequently, PRUDENTIAL, not having shown
that TRANS-ASIA breached the warranty condition, CLASSED AND CLASS MAINTAINED, it remains that TRANS-
ASIA must be allowed to recover its rightful claims on the policy.
B. Assuming arguendo that TRANS-ASIA violated the policy condition on WARRANTED VESSEL CLASSED AND CLASS
MAINTAINED, PRUDENTIAL made a valid waiver of the same.
The Court of Appeals, in reversing the Judgment of the RTC which held that TRANS-ASIA breached the
warranty provision on CLASSED AND CLASS MAINTAINED, underscored that PRUDENTIAL can be deemed to have
made a valid waiver of TRANS-ASIAs breach of warranty as alleged, ratiocinating, thus:

Third, after the loss, Prudential renewed the insurance policy of Trans-Asia for two (2)
consecutive years, from noon of 01 July 1994 to noon of 01 July 1995, and then again until noonof 01
July 1996. This renewal is deemed a waiver of any breach of warranty.[26]

PRUDENTIAL finds fault with the ruling of the appellate court when it ruled that the renewal policies are
deemed a waiver of TRANS-ASIAs alleged breach, averring herein that the subsequent policies, designated as
MH94/1595 and MH95/1788 show that they were issued only on 1 July 1994 and 3 July 1995, respectively, prior to the
time it made a request to TRANS-ASIA that it be furnished a copy of the certification specifying that the insured vessel
M/V Asia Korea was CLASSED AND CLASS MAINTAINED.PRUDENTIAL posits that it came to know of the breach
by TRANS-ASIA of the subject warranty clause only on 21 April 1997. On even date, PRUDENTIAL sent TRANS-
ASIA a letter of denial, advising the latter that their claim is not compensable. In fine, PRUDENTIAL would have this
Court believe that the issuance of the renewal policies cannot be a waiver because they were issued without knowledge of
the alleged breach of warranty committed by TRANS-ASIA.[27]

We are not impressed. We do not find that the Court of Appeals was in error when it held that PRUDENTIAL, in
renewing TRANS-ASIAs insurance policy for two consecutive years after the loss covered by Policy No. MH93/1363,
was considered to have waived TRANS-ASIAs breach of the subject warranty, if any. Breach of a warranty or of a
condition renders the contract defeasible at the option of the insurer; but if he so elects, he may waive his privilege and
power to rescind by the mere expression of an intention so to do. In that event his liability under the policy continues as
before.[28] There can be no clearer intention of the waiver of the alleged breach than the renewal of the policy insurance
granted by PRUDENTIAL to TRANS-ASIA in MH94/1595 and MH95/1788, issued in the years 1994 and 1995,
respectively.

To our mind, the argument is made even more credulous by PRUDENTIALs lack of proof to support its
allegation that the renewals of the policies were taken only after a request was made to TRANS-ASIA to furnish them a
copy of the certificate attesting that M/V Asia Korea was CLASSED AND CLASS MAINTAINED. Notwithstanding
PRUDENTIALs claim that no certification was issued to that effect, it renewed the policy, thereby, evidencing an
intention to waive TRANS-ASIAs alleged breach. Clearly, by granting the renewal policies twice and successively after
the loss, the intent was to benefit the insured, TRANS-ASIA, as well as to waive compliance of the warranty.

The foregoing finding renders a determination of whether the subject warranty is a rider, moot, as raised by the
PRUDENTIAL in its assignment of errors. Whether it is a rider will not effectively alter the result for the reasons that: (1)
PRUDENTIAL was not able to discharge the burden of evidence to show that TRANS-ASIA committed a breach,
thereof; and (2) assuming arguendo the commission of a breach by TRANS-ASIA, the same was shown to have been
waived by PRUDENTIAL.
II.

A. The amount of P3,000,000.00 granted by PRUDENTIAL to TRANS- ASIA via a transaction between the parties
evidenced by a document denominated as Loan and Trust Receipt, dated 29 May 1995 constituted partial payment
on the policy.

It is undisputed that TRANS-ASIA received from PRUDENTIAL the amount of P3,000,000.00. The same was
evidenced by a transaction receipt denominated as a Loan and Trust Receipt, dated 29 May 1995, reproduced hereunder:

LOAN AND TRUST RECEIPT

Claim File No. MH-93-025 May 29, 1995


P3,000,000.00
Check No. PCIB066755

Received FROM PRUDENTIAL GUARANTEE AND ASSURANCE INC., the sum of PESOS THREE
MILLION ONLY (P3,000,000.00) as a loan without interest, under Policy No. MH93/1353, repayable
only in the event and to the extent that any net recovery is made by TRANS ASIA SHIPPING CORP.,
from any person or persons, corporation or corporations, or other parties, on account of loss by any
casualty for which they may be liable, occasioned by the 25 October 1993: Fire on Board.

As security for such repayment, we hereby pledge to PRUDENTIAL GUARANTEE AND


ASSURANCE INC. whatever recovery we may make and deliver to it all documents necessary to prove
our interest in said property. We also hereby agree to promptly prosecute suit against such persons,
corporation or corporations through whose negligence the aforesaid loss was caused or who may
otherwise be responsible therefore, with all due diligence, in our own name, but at the expense of and
under the exclusive direction and control of PRUDENTIAL GUARANTEE AND ASSURANCE INC.

TRANS-ASIA SHIPPING CORPORATION[29]

PRUDENTIAL largely contends that the Loan and Trust Receipt executed by the parties evidenced a loan
of P3,000,000.00 which it granted to TRANS-ASIA, and not an advance payment on the policy or a partial payment for
the loss. It further submits that it is a customary practice for insurance companies in this country to extend loans
gratuitously as part of good business dealing with their assured, in order to afford their assured the chance to continue
business without embarrassment while awaiting outcome of the settlement of their claims. [30] According to
PRUDENTIAL, the Trust and Loan Agreement did not subrogate to it whatever rights and/or actions TRANS-ASIA may
have against third persons, and it cannot by no means be taken that by virtue thereof, PRUDENTIAL was granted
irrevocable power of attorney by TRANS-ASIA, as the sole power to prosecute lies solely with the latter.

The Court of Appeals held that the real character of the transaction between the parties as evidenced by the Loan
and Trust Receipt is that of an advance payment by PRUDENTIAL of TRANS-ASIAs claim on the insurance, thus:

The Philippine Insurance Code (PD 1460 as amended) was derived from the old Insurance Law
Act No. 2427 of the Philippine Legislature during the American Regime. The Insurance Act was lifted
verbatim from the law of California, except Chapter V thereof, which was taken largely from the
insurance law of New York. Therefore, ruling case law in that jurisdiction is to Us persuasive in
interpreting provisions of our own Insurance Code. In addition, the application of the adopted statute
should correspond in fundamental points with the application in its country of origin x x x.

xxxx

Likewise, it is settled in that jurisdiction that the (sic) notwithstanding recitals in the Loan Receipt that the
money was intended as a loan does not detract from its real character as payment of claim, thus:

The receipt of money by the insured employers from a surety company for losses
on account of forgery of drafts by an employee where no provision or repayment of the
money was made except upon condition that it be recovered from other parties and
neither interest nor security for the asserted debts was provided for, the money
constituted the payment of a liability and not a mere loan, notwithstanding recitals in the
written receipt that the money was intended as a mere loan.

What is clear from the wordings of the so-called Loan and Trust Receipt Agreement is that
appellant is obligated to hand over to appellee whatever recovery (Trans Asia) may make and deliver
to (Prudential) all documents necessary to prove its interest in the said property. For all intents and
purposes therefore, the money receipted is payment under the policy, with Prudential having the right
of subrogation to whatever net recovery Trans-Asia may obtain from third parties resulting from the
fire. In the law on insurance, subrogation is an equitable assignment to the insurer of all remedies
which the insured may have against third person whose negligence or wrongful act caused the loss
covered by the insurance policy, which is created as the legal effect of payment by the insurer as an
assignee in equity. The loss in the first instance is that of the insured but after reimbursement or
compensation, it becomes the loss of the insurer. It has been referred to as the doctrine of substitution
and rests on the principle that substantial justice should be attained regardless of form, that is, its basis
is the doing of complete, essential, and perfect justice between all the parties without regard to
form.[31]

We agree. Notwithstanding its designation, the tenor of the Loan and Trust Receipt evidences that the real nature
of the transaction between the parties was that the amount of P3,000,000.00 was not intended as a loan whereby TRANS-
ASIA is obligated to pay PRUDENTIAL, but rather, the same was a partial payment or an advance on the policy of the
claims due to TRANS-ASIA.

First, the amount of P3,000,000.00 constitutes an advance payment to TRANS-ASIA by PRUDENTIAL,


subrogating the former to the extent of any net recovery made by TRANS ASIA SHIPPING CORP., from any person or
persons, corporation or corporations, or other parties, on account of loss by any casualty for which they may be liable,
occasioned by the 25 October 1993: Fire on Board.[32]

Second, we find that per the Loan and Trust Receipt, even as TRANS-ASIA agreed to promptly prosecute suit
against such persons, corporation or corporations through whose negligence the aforesaid loss was caused or who may
otherwise be responsible therefore, with all due diligence in its name, the prosecution of the claims against such third
persons are to be carried on at the expense of and under the exclusive direction and control of PRUDENTIAL
GUARANTEE AND ASSURANCE INC.[33] The clear import of the phrase at the expense of and under the exclusive
direction and control as used in the Loan and Trust Receipt grants solely to PRUDENTIAL the power to prosecute, even
as the same is carried in the name of TRANS-ASIA, thereby making TRANS-ASIA merely an agent of PRUDENTIAL,
the principal, in the prosecution of the suit against parties who may have occasioned the loss.
Third, per the subject Loan and Trust Receipt, the obligation of TRANS-ASIA to repay PRUDENTIAL is highly
speculative and contingent, i.e., only in the event and to the extent that any net recovery is made by TRANS-ASIA from
any person on account of loss occasioned by the fire of 25 October 1993. The transaction, therefore, was made to benefit
TRANS-ASIA, such that, if no recovery from third parties is made, PRUDENTIAL cannot be repaid the amount. Verily,
we do not think that this is constitutive of a loan.[34] The liberality in the tenor of the Loan and Trust Receipt in favor of
TRANS-ASIA leads to the conclusion that the amount of P3,000,000.00 was a form of an advance payment on TRANS-
ASIAs claim on MH93/1353.

III.

A. PRUDENTIAL is directed to pay TRANS-ASIA the amount of P8,395,072.26, representing the balance of the
loss suffered by TRANS-ASIA and covered by Marine Policy No. MH93/1363.

Our foregoing discussion supports the conclusion that TRANS-ASIA is entitled to the unpaid claims covered by
Marine Policy No. MH93/1363, or a total amount of P8,395,072.26.

B. Likewise, PRUDENTIAL is directed to pay TRANS-ASIA, damages in the form of attorneys fees equivalent to
10% of P8,395,072.26.

The Court of Appeals denied the grant of attorneys fees. It held that attorneys fees cannot be awarded absent a
showing of bad faith on the part of PRUDENTIAL in rejecting TRANS-ASIAs claim, notwithstanding that the rejection
was erroneous. According to the Court of Appeals, attorneys fees can be awarded only in the cases enumerated in Article
2208 of the Civil Code which finds no application in the instant case.

We disagree. Sec. 244 of the Insurance Code grants damages consisting of attorneys fees and other expenses
incurred by the insured after a finding by the Insurance Commissioner or the Court, as the case may be, of an
unreasonable denial or withholding of the payment of the claims due. Moreover, the law imposes an interest of twice the
ceiling prescribed by the Monetary Board on the amount of the claim due the insured from the date following the time
prescribed in Section 242[35] or in Section 243,[36] as the case may be, until the claim is fully satisfied. Finally, Section 244
considers the failure to pay the claims within the time prescribed in Sections 242 or 243, when applicable, as prima
facie evidence of unreasonable delay in payment.

To the mind of this Court, Section 244 does not require a showing of bad faith in order that attorneys fees be
granted. As earlier stated, under Section 244, a prima facie evidence of unreasonable delay in payment of the claim is
created by failure of the insurer to pay the claim within the time fixed in both Sections 242 and 243 of the Insurance
Code. As established in Section 244, by reason of the delay and the consequent filing of the suit by the insured, the
insurers shall be adjudged to pay damages which shall consist of attorneys fees and other expenses incurred by the
insured.[37]

Section 244 reads:


In case of any litigation for the enforcement of any policy or contract of insurance, it shall be the
duty of the Commissioner or the Court, as the case may be, to make a finding as to whether the payment
of the claim of the insured has been unreasonably denied or withheld; and in the affirmative case, the
insurance company shall be adjudged to pay damages which shall consist of attorneys fees and other
expenses incurred by the insured person by reason of such unreasonable denial or withholding of payment
plus interest of twice the ceiling prescribed by the Monetary Board of the amount of the claim due the
insured, from the date following the time prescribed in section two hundred forty-two or in section two
hundred forty-three, as the case may be, until the claim is fully satisfied; Provided, That the failure to pay
any such claim within the time prescribed in said sections shall be considered prima facie evidence of
unreasonable delay in payment.

Sections 243 and 244 of the Insurance Code apply when the court finds an unreasonable delay or refusal in the
payment of the insurance claims.

In the case at bar, the facts as found by the Court of Appeals, and confirmed by the records show that there was an
unreasonable delay by PRUDENTIAL in the payment of the unpaid balance of P8,395,072.26 to TRANS-ASIA. On 26
October 1993, a day after the occurrence of the fire in M/V Asia Korea, TRANS-ASIA filed its notice of claim. On13
August 1996, the adjuster, Richards Hogg International (Phils.), Inc., completed its survey report recommending the
amount of P11,395,072.26 as the total indemnity due to TRANS-ASIA.[38] On 21 April 1997, PRUDENTIAL, in a
letter[39] addressed to TRANS-ASIA denied the latters claim for the amount of P8,395,072.26 representing the balance of
the total indemnity. On 21 July 1997, PRUDENTIAL sent a second letter[40] to TRANS-ASIA seeking a return of the
amount of P3,000,000.00. On 13 August 1997, TRANS-ASIA was constrained to file a complaint for sum of money
against PRUDENTIAL praying, inter alia, for the sum of P8,395,072.26 representing the balance of the proceeds of the
insurance claim.

As can be gleaned from the foregoing, there was an unreasonable delay on the part of PRUDENTIAL to pay TRANS-
ASIA, as in fact, it refuted the latters right to the insurance claims, from the time proof of loss was shown and the
ascertainment of the loss was made by the insurance adjuster. Evidently, PRUDENTIALs unreasonable delay in satisfying
TRANS-ASIAs unpaid claims compelled the latter to file a suit for collection.

Succinctly, an award equivalent to ten percent (10%) of the unpaid proceeds of the policy as attorneys fees to TRANS-
ASIA is reasonable under the circumstances, or otherwise stated, ten percent (10%) of P8,395,072.26. In the case
of Cathay Insurance, Co., Inc. v. Court of Appeals,[41] where a finding of an unreasonable delay under Section 244 of the
Insurance Code was made by this Court, we grant an award of attorneys fees equivalent to ten percent (10%) of the total
proceeds. We find no reason to deviate from this judicial precedent in the case at bar.

C. Further, the aggregate amount (P8,395,072.26 plus 10% thereof as attorneys fees) shall be imposed double
interest in accordance with Section 244 of the Insurance Code.

Section 244 of the Insurance Code is categorical in imposing an interest twice the ceiling prescribed by the Monetary
Board due the insured, from the date following the time prescribed in Section 242 or in Section 243, as the case may be,
until the claim is fully satisfied. In the case at bar, we find Section 243 to be applicable as what is involved herein is a
marine insurance, clearly, a policy other than life insurance.

Section 243 is hereunder reproduced:

SEC. 243. The amount of any loss or damage for which an insurer may be liable, under any policy other
than life insurance policy, shall be paid within thirty days after proof of loss is received by the insurer and
ascertainment of the loss or damage is made either by agreement between the insured and the insurer or
by arbitration; but if such ascertainment is not had or made within sixty days after such receipt by the
insurer of the proof of loss, then the loss or damage shall be paid within ninety days after such
receipt. Refusal or failure to pay the loss or damage within the time prescribed herein will entitle the
assured to collect interest on the proceeds of the policy for the duration of the delay at the rate of twice
the ceiling prescribed by the Monetary Board, unless such failure or refusal to pay is based on the ground
that the claim is fraudulent.

As specified, the assured is entitled to interest on the proceeds for the duration of the delay at the rate of twice the
ceiling prescribed by the Monetary Board except when the failure or refusal of the insurer to pay was founded on the
ground that the claim is fraudulent.

D. The term double interest as used in the Decision of the Court of Appeals must be interpreted to mean 24% per
annum.

PRUDENTIAL assails the award of interest, granted by the Court of Appeals, in favor of TRANS-ASIA in the
assailed Decision of 6 November 2001. It is PRUDENTIALs stance that the award is extortionate and grossly
unsconscionable. In support thereto, PRUDENTIAL makes a reference to TRANS-ASIAs prayer in the Complaint filed
with the court a quo wherein the latter sought, interest double the prevailing rate of interest of 21% per annum now
obtaining in the banking business or plus 42% per annum pursuant to Article 243 of the Insurance Code x x x.[42]

The contention fails to persuade. It is settled that an award of double interest is lawful and justified under Sections
243 and 244 of the Insurance Code.[43] In Finman General Assurance Corporation v. Court of Appeals,[44] this Court held
that the payment of 24% interest per annum is authorized by the Insurance Code.[45] There is no gainsaying that the term
double interest as used in Sections 243 and 244 can only be interpreted to mean twice 12% per annum or 24% per annum
interest, thus:

The term ceiling prescribed by the Monetary Board means the legal rate of interest of twelve per centum
per annum (12%) as prescribed by the Monetary Board in C.B. Circular No. 416, pursuant to P.D. No.
116, amending the Usury Law; so that when Sections 242, 243 and 244 of the Insurance Code provide
that the insurer shall be liable to pay interest twice the ceiling prescribed by the Monetary Board, it means
twice 12% per annum or 24% per annum interest on the proceeds of the insurance.[46]

E. The payment of double interest should be counted from 13 September 1996.

The Court of Appeals, in imposing double interest for the duration of the delay of the payment of the unpaid
balance due TRANS-ASIA, computed the same from 13 August 1996 until such time when the amount is fully
paid. Although not raised by the parties, we find the computation of the duration of the delay made by the appellate court
to be patently erroneous.

To be sure, Section 243 imposes interest on the proceeds of the policy for the duration of the delay at the rate of
twice the ceiling prescribed by the Monetary Board.Significantly, Section 243 mandates the payment of any loss or
damage for which an insurer may be liable, under any policy other than life insurance policy, within thirty days after proof
of loss is received by the insurer and ascertainment of the loss or damage is made either by agreement between the insured
and the insurer or by arbitration. It is clear that under Section 243, the insurer has until the 30th day after proof of loss and
ascertainment of the loss or damage to pay its liability under the insurance, and only after such time can the insurer be
held to be in delay, thereby necessitating the imposition of double interest.

In the case at bar, it was not disputed that the survey report on the ascertainment of the loss was completed by the
adjuster, Richard Hoggs International (Phils.), Inc. on 13 August 1996. PRUDENTIAL had thirty days from 13 August
1996 within which to pay its liability to TRANS-ASIA under the insurance policy, or until 13 September 1996.Therefore,
the double interest can begin to run from 13 September 1996 only.

IV.

A. An interest of 12% per annum is similarly imposed on the TOTAL amount of liability adjudged in section III
herein, computed from the time of finality of judgment until the full satisfaction thereof in conformity with
this Courts ruling in Eastern Shipping Lines, Inc. v. Court of Appeals.

This Court in Eastern Shipping Lines, Inc. v. Court of Appeals,[47] inscribed the rule of thumb[48] in the application
of interest to be imposed on obligations, regardless of their source. Eastern emphasized beyond cavil that when the
judgment of the court awarding a sum of money becomes final and executory, the rate of legal interest, regardless of
whether the obligation involves a loan or forbearance of money, shall be 12% per annum from such finality until its
satisfaction, this interim period being deemed to be by then an equivalent to a forbearance[49] of credit.

We find application of the rule in the case at bar proper, thus, a rate of 12% per annum from the finality of
judgment until the full satisfaction thereof must be imposed on the total amount of liability adjudged to PRUDENTIAL. It
is clear that the interim period from the finality of judgment until the satisfaction of the same is deemed equivalent to a
forbearance of credit, hence, the imposition of the aforesaid interest.

Fallo

WHEREFORE, the Petition in G.R. No. 151890 is DENIED. However, the Petition in G.R. No. 151991 is GRANTED,
thus, we award the grant of attorneys fees and make a clarification that the term double interest as used in the 6 November
2001 Decision of the Court of Appeals in CA GR CV No. 68278 should be construed to mean interest at the rate of 24%
per annum, with a further clarification, that the same should be computed from 13 September 1996 until fully paid. The
Decision and Resolution of the Court of Appeals, in CA-G.R. CV No. 68278, dated 6 November 2001 and 29 January
2002, respectively, are, thus, MODIFIED in the following manner, to wit:
1. PRUDENTIAL is DIRECTED to PAY TRANS-ASIA the amount of P8,395,072.26,
representing the balance of the loss suffered by TRANS-ASIA and covered by Marine Policy No.
MH93/1363;

2. PRUDENTIAL is DIRECTED further to PAY TRANS-ASIA damages in the form of


attorneys fees equivalent to 10% of the amount of P8,395,072.26;

3. The aggregate amount (P8,395,072.26 plus 10% thereof as attorneys fees) shall be imposed
double interest at the rate of 24% per annum to be computed from 13 September 1996 until fully
paid; and

4. An interest of 12% per annum is similarly imposed on the TOTAL amount of liability adjudged as
abovestated in paragraphs (1), (2), and (3) herein, computed from the time of finality of judgment
until the full satisfaction thereof.

No costs.
G.R. No. L-51221 July 31, 1991

FIRST INTEGRATED BONDING & INSURANCE COMPANY, INC., petitioner,


vs.
HON. HAROLD M. HERNANDO, VICTORINO ADVINCULA, ROMANA ADVINCULA, SILVERIO BLANCO &
THE SHERIFF OF MANILA and his DEPUTY SHERIFFS, respondents.

Octavio M. Zavas for petitioner.

MEDIALDEA, J.:

This petition for certiorari under Rule 65 of the Revised Rules of Court, seeks the annulment of the amended
decision of respondent trial court in Civil Case No. 1104 for allegedly having been rendered in excess of
jurisdiction. The same decision was sought to be annulled in a petition for relief from judgment filed in the same
case but the petition was denied for having been filed out of time.

The narration of facts below was taken from the pleadings filed by the parties. As regards the proceedings
following the promulgation of the amended decision, the dates were supplied in the Comment and Answer filed
by respondent judge and which were not disputed by petitioner.

Silverio Blanco was the owner of a passenger jeepney which he insured against liabilities for death and injuries
to third persons with First Integrated Bonding and Insurance Company, Inc. (First Insurance) under Motor
Vehicle Policy No. V-0563751 with the face value of P30,000.00 (p. 15, Rollo).

On November 25, 1976, the said jeepney driven by Blanco himself bumped a five-year old child, Deogracias
Advincula, causing the latter's death.

A complaint (pp. 38-41, Rollo) for damages was brought by the child's parents, the Advincula spouses, against
Silverio Blanco. First Insurance was also impleaded in the complaint as the insurer. The complaint was
docketed as Civil Case No. 1104 of the Court of First Instance of Abra (now Regional Trial Court).

Summons were served on Silverio Blanco and First Insurance. However, only Blanco filed an answer. Upon
motion of the Advincula spouses, First Insurance was declared in default (p. 45, Rollo) on January 19, 1978.
Thereafter, a pre-trial conference was conducted where the Advincula spouses presented the following
documentary evidence:

Exhibit "A" Marriage Certificate, Exhibit B Birth Certificate, Exhibit B-1 The Certificate of the
Local Civil Registrar, Exhibit C Certificate of Death, Exhibit C-1 the official receipt of the burial
permit, Exhibit C-2 the autopsy report, Exhibit D filing fee under official receipt in the amount of
P80.00, Exhibit D-1 list of actual expenses in connection with the death and burial of the deceased
Advincula, Exhibit E Criminal Case No. 666 of the Municipal Court of Tayum, Abra entitled People of
the Philippines versus Silverio Blanco for Homicide thru Reckless Imprudence, Exhibit E-1 sworn
statement of Severino Balneg Exhibit F Tax Declaration No. 906 in the name of Maria Blanco
delivered by Silverio Blanco to the plaintiffs as pledge of Silverio Blanco to settle the civil aspect of this
case. (pp. 14-15, Rollo)

On the basis of the evidence presented by the Advincula spouses, judgment was rendered by the trial court on
March 1, 1978, the dispositive portion of which states:

WHEREFORE, for moral damages, this court adjudicates to the plaintiffs P5,000.00; for the life of
Deogracias Advincula P12,000.00, for funeral expenses, P3,663.50 and for attomey's fees, P3,000.00.
The satisfaction of these damages divulged (sic) independently now upon the defendant insurance
company and to pay the costs of the proceedings.

SO ORDERED. (p. 16, Rollo)

First Insurance received a copy of the decision on March 14, 1978. Upon motion of the Advincula spouses, the
decision was amended on March 27, 1978 (p. 17, Rollo), which, in addition to the damages granted in the
original decision, awarded damages in the amount of P6,336.50 to Silverio Blanco. The dispositive portion of
the amended decision is quoted, as follows:

WHEREFORE, for moral damages, this Court hereby adjudicates to the plaintiffs P5,000.00; for the life
of Deogracias Advincula P12,000.00; for funeral expenses P3,663.50 and for attorney's fees P3,000.00
or in the total amount of P23,663.50 which must be satisfied independently by the defendant First
Integrated Bonding and Insurance Company, Inc. in favor of the plaintiffs and the balance of P6,336.50
shall also be paid by said defendant Insurance Company to the defendant Silverio Blanco. The grand
total under the insurance policy, Exhibit H, is P30,000.00.

The defendant Insurance Company to pay the costs of the proceedings.

SO ORDERED. (p. 17, Rollo)

The amended decision was received by First Instance on April 11, 1978. On May 11, 1978, entry of judgment
was made, a copy of which was furnished First Insurance on June 27, 1978. Upon motion of the Advincula
spouses, an order granting execution was issued by the court on June 14, 1978, which was received by First
Insurance on August 1, 1978 (pp. 31-32, Rollo).

On September 5, 1978, First Insurance filed a petition for relief from judgment in the same case. The petition
was set for hearing on September 28, 1978. No appearance was entered by First Insurance on the said date.
On October 4, 1978, the trial court issued an order, denying the petition for relief from judgment (pp. 33-
34, Rollo), a copy of which was received by First Insurance on October 10, 1978 (p. 35, Rollo). The order
reads:

The records of this case show that on April 11, 1978, the defendant First Integrated Bonding and
Insurance Company, Inc. received a copy of the amended decision dated March 27, 1978 and found on
page 30 of the records of this case; on May 11, 1978, the Deputy Clerk of Court entered the
corresponding entry of judgment and the First Integrated Bonding and Insurance Company, Inc.
received a copy thereof on June 27, 1978, On June 13, 1978, the plaintiffs moved for execution of
judgment and the same was granted pursuant to an Order of this Court dated June 14, 1978 and found
on page 35 of the records of this case.

And now comes the petition for relief from the Order of execution and judgment with preliminary
injunction filed by First integrated Bonding and Insurance Co., Inc. and which was received by this
Court on September 5, 1978; on September 28, 1978, the plaintiffs filed their written opposition to the
petition for relief from judgment and preliminary injunction. The opposition is based on three grounds,
namely: 1. that the petition is filed out of time; 2. that there was gross and notorious negligence of the
Insurance Company; 3. that this case is within the jurisdiction of this Court and therefore the cause of
action of the plaintiffs deserves judicial consideration.

It was on April 11, 1978 that the First Integrated Bonding and Insurance Co., Inc. received the amended
decision and the petition for relief from Order of Execution and judgment with preliminary injunction was
filed on September 5, 1978 or a period of 191 days already expired, that is, more than 6 months
already as required by Section 3, Rule 38 of the Rules of Court. Consequently, the first ground invoked
by the opposition must be sustained. On the second ground, the records of this case show that the First
Integrated Bonding and Insurance Co., Inc. was duly summoned and served a copy of the complaint on
August 16, 1977 and it was received by the President of the Insurance Company as shown by the
certificate of Service of the Sheriff of Manila and found in page 12 and page 13 of the records of this
case; after the reglementary period to file an answer expired, the plaintiffs move to declare the
defendant insurance company in default and likewise asked the Court that they be allowed to present
their evidence on January 23, 1978 and which was granted by this Court pursuant to an order dated
January 19, 1978 and found on page 16 of the records of this case; after the reception of the evidence
for the plaintiffs this Court rendered a decision on March 1, 1978 and which is found on pages 23 to 26
of the records of this case; subsequently, on March 27, 1978, an amended decision was issued by this
Court and it is found on page 30 of the records of this case. Clearly, therefore, the First Integrated
Bonding and Insurance Co., Inc. was grossly and notoriously negligent in giving the proper attention to
this case. This kind of gross and notorious negligence can not be considered excusable. The last
ground is that this Court has jurisdiction over the plaintiffs' cause of action against the insurance
company. This ground is well-taken because according to Section 416 of the Philippine Insurance
Code, Presidential Decree No. 612, it provides that the authority to adjudicate granted to the
Commissioner of insurance shall be concurrent with that of the civil courts, but the filing of a complaint
with the commissioner shall preclude the civil courts from taking cognizance of a suit involving the
same subject matter. Furthermore, the plaintiffs did not intervene in the criminal aspect of this case,
instead, they filed a separate and independent civil action on July 26, 1977 and which is now the
present Civil Case No. 1104. It may be added, that the matter of exhaustion of administrative remedy
may be waived which has been so in the present case because the First Integrated Bonding and
Insurance Co., Inc. was declared in default.

In view of all the foregoing considerations, the petition for relief from the order of execution and
judgment with preliminary injunction, for lack of merit, is hereby denied.

SO ORDERED. (pp. 33-34, Rollo)

First Insurance filed a motion for reconsideration of the order denying the petition for relief on May 14, 1979.
The motion was set for hearing and again no appearance was entered by the movant First Insurance (p.
35, Rollo), prompting the trial court to deny the same.

On August 13, 1979, the herein petitioner First Insurance filed this petition for certiorari on the following
grounds:

1. The trial court erred in deciding for the respondent spouse(s) where there exists no cause of action
against the herein petitioner.

2. The trial court erred when it abbreviated the proceeding and rendered judgment based only on the
documentary evidence presented during the pre-trial conference.

3. The trial court erred in holding the petitioner liable in excess of the limits of liability as provided for in
the policy contract.

On August 20, 1979, this Court issued a temporary restraining order enjoining the respondents from enforcing
the Writ of Execution dated August 1, 1978 (p. 19, Rollo)

It is the contention of the petitioner that the Advincula spouses have no cause of action against it. As parents of
the victim, they may proceed against the driver, Silverio Blanco on the basis of the provisions of the New Civil
Code. However, they have no cause of action against First Insurance, because they are not parties to the
insurance contract.

It is settled that where the insurance contract provides for indemnity against liability to a third party, such third
party can directly sue the insurer (Caguia v. Fieldman's Insurance Co., Inc., G. R. No. 23276, November 29,
1968, 26 SCRA 178). The liability of the insurer to such third person is based on contract while the liability of
the insured to the third party is based on tort (Malayan Insurance Co., Inc. v. CA, L-36413, September 26,
1988, 165 SCRA 536). This rule was explained in the case of Shafer v. Judge, RTC of Olongapo City, Br. 75,
G.R. No. 78848, November 14, 1988:

The injured for whom the contract of insurance is intended can sue directly the insurer. The general
purpose of statutes enabling an injured person to proceed directly against the insurer is to protect
injured persons against the insolvency of the insured who causes such injury, and to give such injured
person a certain beneficial interest in the proceeds of the policy, and statutes are to be liberally
construed so that their intended purpose may be accomplished. It has even been held that such a
provision creates a contractual relation which inures to the benefit of any and every person who may be
negligently injured by the named insured as if such injured person were specifically named in the policy.

In the event that the injured fails or refuses to include the insurer as party defendant in his claim for
indemnity against the insured, the latter is not prevented by law to avail of the procedural rules intended
to avoid multiplicity of suits. Not even a "no action" clause under the policy which requires that a final
judgment be first obtained against the insured and that only thereafter can the person insured recover
on the policy can prevail over the Rules of Court provisions aimed at avoiding multiplicity of suits. (p.
391, 167 SCRA) (emphasis supplied)

First Insurance cannot evade its liability as insurer by hiding under the cloak of the insured. Its liability is
primary and not dependent on the recovery of judgment from the insured.

Compulsory Motor Vehicle Liability Insurance (third party liability, or TPL) is primarily intended to
provide compensation for the death or bodily injuries suffered by innocent third parties or passengers
as a result of a negligent operation and use of motor vehicles. The victims and/or their dependents are
assured of immediate financial assistance, regardless of the financial capacity of the motor vehicle
owners.

. . . the insurer's liability accrues immediately upon the occurrence of the injury or event upon which the
liability depends, and does not depend on the recovery of judgment by the injured party against the
insured (Shafer v. Judge, RTC of Olongapo, supra, p. 390).

It is true that Blanco denied that he was negligent when the incident occurred. However, during the pre-trial
conference, when respondent judge admitted all the exhibits of the plaintiffs to abbreviate the proceedings, no
objection was interposed by Blanco. When a decision was rendered based only on the exhibits of the plaintiffs,
Blanco likewise did not object. No motion for reconsideration was filed by either Blanco or First Insurance.
Hence, the decision became final and may no longer be attacked.

It should be noted also that First Insurance was declared in default because of its failure to file an answer. As
far as it was concerned, it failed to raise any triable issue. It lost its standing in court and judgment may be
rendered against it on the basis only of the evidence of the Advincula spouses.

Petitioner had been given its day in court. Despite its having been declared in default and its failure to file a
motion to lift the order of default, it was still notified of the subsequent proceedings in the trial court. But no
positive step was taken by it on time to vacate the order of default, the decision nor the amended decision.
Instead, it chose to file a petition for relief from judgment on September 1, 1978 almost five (5) months from its
receipt of a copy of the amended decision on April 11, 1978. Clearly, the said petition for relief from judgment
was filed out of time. The rules require that such petitions must be filed within sixty (60) days after the
petitioner learns of the judgment and not more than six (6) months after such judgment was entered (Rule 38,
Section 3). The period fixed by Rule 38 of the Rules of Court is non-extendible and never interrupted. It is not
subject to any condition or contingency, because it is itself devised to meet a condition or contingency. The
remedy allowed by Rule 38 is an act of grace, as it were, designed to give the aggrieved party another and last
chance. Being in the position of one who begs, such party's privilege is not to impose conditions, haggle or
dilly-dally, but to grab what is offered him. (Palomares, et al. v. Jimenez, et al., 90 Phil. 773, XVII, L.J., No. 3, p.
136, Rafanan v. Rafanan, 35 O.G. 228; Santos v. Manila Electric Co., G.R. L-7735, December 29, 1955; Gana
v. Abaya, G.R. No. L-3106, December 29, 1955, cited in Vicente J. Francisco, The Revised Rules of Court of
the Philippines, Annotated and Commented, Vol, 11, p. 580.
It appears that the award of damages in favor of Blanco has no basis. The complaint in Civil case 1104 was for
damages brought by the spouses against Blanco and First Insurance. Blanco did not put up any claim against
the latter. However, since the said decision had already become final and executory, it can no longer be
corrected or amended. In the same vein, the claim of petitioner that its liability to third parties under the
insurance policy is limited to P20,000.00 only can no longer be given consideration at this late stage, when the
decision of the trial court awarding damages had already become final and executory.

ACCORDINGLY, finding respondent judge to have acted within his jurisdiction in denying the petition for relief
from judgment, the petition is DISMISSED. The questioned decision of the trial court in Civil Case No. 1104
having become final and executory, is AFFIRMED. The temporary restraining order issued on August 20, 1979
is hereby lifted. Costs against petitioner.

G.R. No. L-20853 May 29, 1967


BONIFACIO BROS., INC., ET AL., plaintiffs-appellants,
vs.
ENRIQUE MORA, ET AL., defendants-appellees.

G. Magsaysay for plaintiffs-appellants.


Abad Santos and Pablo for defendant-appellee H. E. Reyes, Inc.
J. P. Santilla and A. D. Hidalgo, Jr. for other defendant-appellee.

CASTRO, J.:

This is an appeal from the decision of the Court of First Instance of Manila, Branch XV, in civil case 48823, affirming
the decision of the Municipal Court of Manila, declaring the H.S. Reyes, Inc. as having a better right than the
Bonifacio Bros., Inc. and the Ayala Auto Parts Company, appellants herein, to the proceeds of motor insurance
policy A-0615, in the sum of P2,002.73, issued by the State Bonding & Insurance Co. Inc., and directing payment of
the said amount to the H. Reyes, Inc.

Enrique Mora, owner of Oldsmobile sedan model 1956, bearing plate No. QC- mortgaged the same to the H.S.
Reyes, Inc., with the condition that the former would insure the automobile with the latter as beneficiary. The
automobile was thereafter insured on June 23, 1959 with the State Bonding & Insurance Co., Inc., and motor car
insurance policy A-0615 was issued to Enrique Mora, the pertinent provisions of which read:

1. The Company (referring to the State Bonding & Insurance Co., Inc.) will, subject to the Limits of Liability,
indemnify the Insured against loss of or damages to the Motor Vehicle and its accessories and spare parts
whilst thereon; (a) by accidental collision or overturning or collision or overturning consequent upon
mechanical breakdown or consequent upon wear and tear,

xxx xxx xxx

2. At its own option the Company may pay in cash the amount of the loss or damage or may repair,
reinstate, or replace the Motor Vehicle or any part thereof or its accessories or spare parts. The liability of
the Company shall not exceed the value of the parts whichever is the less. The Insured's estimate of value
stated in the schedule will be the maximum amount payable by the Company in respect of any claim for loss
or damage. 1w ph1.t

xxx xxx xxx

4. The Insured may authorize the repair of the Motor Vehicle necessitated by damage for which the
Company may be liable under this Policy provided that: (a) The estimated cost of such repair does not
exceed the Authorized Repair Limit, (b) A detailed estimate of the cost is forwarded to the Company without
delay, subject to the condition that "Loss, if any is payable to H.S. Reyes, Inc.," by virtue of the fact that said
Oldsmobile sedan was mortgaged in favor of the said H.S. Reyes, Inc. and that under a clause in said
insurance policy, any loss was made payable to the H.S. Reyes, Inc. as Mortgagee;

xxx xxx xxx

During the effectivity of the insurance contract, the car met with an accident. The insurance company then assigned
the accident to the Bayne Adjustment Co. for investigation and appraisal of the damage. Enrique Mora, without the
knowledge and consent of the H.S. Reyes, Inc., authorized the Bonifacio Bros. Inc. to furnish the labor and
materials, some of which were supplied by the Ayala Auto Parts Co. For the cost of labor and materials, Enrique
Mora was billed at P2,102.73 through the H.H. Bayne Adjustment Co. The insurance company after claiming a
franchise in the amount of P100, drew a check in the amount of P2,002.73, as proceeds of the insurance policy,
payable to the order of Enrique Mora or H.S. Reyes,. Inc., and entrusted the check to the H.H. Bayne Adjustment
Co. for disposition and delivery to the proper party. In the meantime, the car was delivered to Enrique Mora without
the consent of the H.S. Reyes, Inc., and without payment to the Bonifacio Bros. Inc. and the Ayala Auto Parts Co. of
the cost of repairs and materials.
Upon the theory that the insurance proceeds should be paid directly to them, the Bonifacio Bros. Inc. and the Ayala
Auto Parts Co. filed on May 8, 1961 a complaint with the Municipal Court of Manila against Enrique Mora and the
State Bonding & Insurance Co., Inc. for the collection of the sum of P2,002.73 The insurance company filed its
answer with a counterclaim for interpleader, requiring the Bonifacio Bros. Inc. and the H.S. Reyes, Inc. to interplead
in order to determine who has better right to the insurance proceeds in question. Enrique Mora was declared in
default for failure to appear at the hearing, and evidence against him was received ex parte. However, the counsel
for the Bonifacio Bros. Inc., Ayala Auto Parts Co. and State Bonding & Insurance Co. Inc. submitted a stipulation of
facts, on the basis of which are Municipal Court rendered a decision declaring the H.S. Reyes, Inc. as having a
better right to the disputed amount and ordering State Bonding & Insurance Co. Inc. to pay to the H. S. Reyes, Inc.
the said sum of P2,002.73. From this decision, the appellants elevated the case to the Court of First Instance of
Manila which the stipulation of facts was reproduced. On October 19, 1962 the latter court rendered a decision,
affirming the decision of the Municipal Court. The Bonifacio Bros. Inc. and the Ayala Auto Parts Co. moved for
reconsideration of the decision, but the trial court denied the motion. Hence, this appeal.

The main issue raised is whether there is privity of contract between the Bonifacio Bros. Inc. and the Ayala Auto
Parts Co. on the one hand and the insurance company on the other. The appellants argue that the insurance
company and Enrique Mora are parties to the repair of the car as well as the towage thereof performed. The
authority for this assertion is to be found, it is alleged, in paragraph 4 of the insurance contract which provides that
"the insured may authorize the repair of the Motor Vehicle necessitated by damage for which the company may be
liable under the policy provided that (a) the estimated cost of such repair does not exceed the Authorized Repair
Limit, and (b) a detailed estimate of the cost is forwarded to the company without delay." It is stressed that the H.H.
Bayne Adjustment Company's recommendation of payment of the appellants' bill for materials and repairs for which
the latter drew a check for P2,002.73 indicates that Mora and the H.H. Bayne Adjustment Co. acted for and in
representation of the insurance company.

This argument is, in our view, beside the point, because from the undisputed facts and from the pleadings it will be
seen that the appellants' alleged cause of action rests exclusively upon the terms of the insurance contract. The
appellants seek to recover the insurance proceeds, and for this purpose, they rely upon paragraph 4 of the
insurance contract document executed by and between the State Bonding & Insurance Company, Inc. and Enrique
Mora. The appellants are not mentioned in the contract as parties thereto nor is there any clause or provision
thereof from which we can infer that there is an obligation on the part of the insurance company to pay the cost of
repairs directly to them. It is fundamental that contracts take effect only between the parties thereto, except in some
specific instances provided by law where the contract contains some stipulation in favor of a third person.1 Such
stipulation is known as stipulation pour autrui or a provision in favor of a third person not a pay to the contract.
Under this doctrine, a third person is allowed to avail himself of a benefit granted to him by the terms of the contract,
provided that the contracting parties have clearly and deliberately conferred a favor upon such
person.2Consequently, a third person not a party to the contract has no action against the parties thereto, and
cannot generally demand the enforcement of the same.3 The question of whether a third person has an enforcible
interest in a contract, must be settled by determining whether the contracting parties intended to tender him such an
interest by deliberately inserting terms in their agreement with the avowed purpose of conferring a favor upon such
third person. In this connection, this Court has laid down the rule that the fairest test to determine whether the
interest of a third person in a contract is a stipulation pour autrui or merely an incidental interest, is to rely upon the
intention of the parties as disclosed by their contract.4 In the instant case the insurance contract does not contain
any words or clauses to disclose an intent to give any benefit to any repairmen or materialmen in case of repair of
the car in question. The parties to the insurance contract omitted such stipulation, which is a circumstance that
supports the said conclusion. On the other hand, the "loss payable" clause of the insurance policy stipulates that
"Loss, if any, is payable to H.S. Reyes, Inc." indicating that it was only the H.S. Reyes, Inc. which they intended to
benefit.

We likewise observe from the brief of the State Bonding & Insurance Company that it has vehemently opposed the
assertion or pretension of the appellants that they are privy to the contract. If it were the intention of the insurance
company to make itself liable to the repair shop or materialmen, it could have easily inserted in the contract a
stipulation to that effect. To hold now that the original parties to the insurance contract intended to confer upon the
appellants the benefit claimed by them would require us to ignore the indespensable requisite that a stipulation pour
autrui must be clearly expressed by the parties, which we cannot do.

As regards paragraph 4 of the insurance contract, a perusal thereof would show that instead of establishing privity
between the appellants and the insurance company, such stipulation merely establishes the procedure that the
insured has to follow in order to be entitled to indemnity for repair. This paragraph therefore should not be construed
as bringing into existence in favor of the appellants a right of action against the insurance company as such
intention can never be inferred therefrom.

Another cogent reason for not recognizing a right of action by the appellants against the insurance company is that
"a policy of insurance is a distinct and independent contract between the insured and insurer, and third persons
have no right either in a court of equity, or in a court of law, to the proceeds of it, unless there be some contract of
trust, expressed or implied between the insured and third person."5 In this case, no contract of trust, expressed or
implied exists. We, therefore, agree with the trial court that no cause of action exists in favor of the appellants in so
far as the proceeds of insurance are concerned. The appellants' claim, if at all, is merely equitable in nature and
must be made effective through Enrique Mora who entered into a contract with the Bonifacio Bros. Inc. This
conclusion is deducible not only from the principle governing the operation and effect of insurance contracts in
general, but is clearly covered by the express provisions of section 50 of the Insurance Act which read:

The insurance shall be applied exclusively to the proper interests of the person in whose name it is made
unless otherwise specified in the policy.

The policy in question has been so framed that "Loss, if any, is payable to H.S. Reyes, Inc.," which unmistakably
shows the intention of the parties.

The final contention of the appellants is that the right of the H.S. Reyes, Inc. to the insurance proceeds arises only if
there was loss and not where there is mere damage as in the instant case. Suffice it to say that any attempt to draw
a distinction between "loss" and "damage" is uncalled for, because the word "loss" in insurance law embraces injury
or damage.

Loss in insurance, defined. The injury or damage sustained by the insured in consequence of the
happening of one or more of the accidents or misfortune against which the insurer, in consideration of the
premium, has undertaken to indemnify the insured. (1 Bouv. Ins. No. 1215; Black's Law Dictionary;
Cyclopedic Law Dictionary, cited in Martin's Phil. Commercial Laws, Vol. 1, 1961 ed. p. 608).

Indeed, according to sec. 120 of the Insurance Act, a loss may be either total or partial.

Accordingly, the judgment appealed from is hereby affirmed, at appellants' cost.


G.R. No. 78848 November 14, 1988

SHERMAN SHAFER, petitioner,


vs.
HON. JUDGE, REGIONAL TRIAL COURT OF OLONGAPO CITY, BRANCH 75, and MAKATI INSURANCE
COMPANY, INC., respondents.

On 2 January 1985, petitioner Sherman Shafer obtained a private car policy, GA No. 0889, 2 over his Ford Laser car
with Plate No. CFN-361 from Makati Insurance Company, Inc., for third party liability (TPL). During the effectivity of
<re||an1w>

the policy, an information 3 for reckless imprudence resulting in damage to property and serious physical injuries
was filed against petitioner. The information reads as follows:

That on or about the seventeeth (17th) day of May 1985, in the City of Olongapo, Philippines, and
within the jurisdiction of this Honorable Court, the above-named accused, being then the driver and
in actual physical control of a Ford Laser car bearing Plate No. CFN-361, did then and there wilfully,
unlawfully and criminally drive, operate and manage the said Ford Laser car in a careless, reckless
and imprudent manner without exercising reasonable caution, diligence and due care to avoid
accident to persons and damage to property and in disregard of existing traffic rules and regulations,
causing by such carelessness, recklessness and imprudence the said Ford Laser car to hit and
bump a Volkswagen car bearing Plate No. NJE-338 owned and driven by Felino llano y Legaspi,
thereby causing damage in the total amount of P12,345.00 Pesos, Philippine Currency, and as a
result thereof one Jovencio Poblete, Sr. who was on board of the said Volkswagen car sustained
physical injuries, to wit:

1. 2 cm. laceration of left side of tongue.

2. 6 cm. laceration with partial transection of muscle (almost full thickness) left side of face.

3. Full thickness laceration of lower lip and adjacent skin.

which injuries causing [sic] deformity on the face. 4

The owner of the damaged Volkswagen car filed a separate civil action against petitioner for damages, while
Jovencio Poblete, Sr., who was a passenger in the Volkswagen car when allegedly hit and bumped by the car
driven by petitioner, did not reserve his right to file a separate civil action for damages. Instead, in the course of the
trial in the criminal case, Poblete, Sr. testified on his claim for damages for the serious physical injuries which he
claimed to have sustained as a result of the accident.

Upon motion, petitioner was granted leave by the former presiding judge of the trail court to file a third party
complaint against the herein private respondent, Makati Insurance Company, Inc. Said insurance company,
however, moved to vacate the order granting leave to petitioner to file a third party complaint against it and/or to
dismiss the same. 5

On 24 April 1987, the court a quo issued an order dismissing the third party complaint on the ground that it was
premature, based on the premise that unless the accused (herein petitioner) is found guilty and sentenced to pay
the offended party (Poblete Sr.) indemnity or damages, the third party complaint is without cause of action. The
court further stated that the better procedure is for the accused (petitioner) to wait for the outcome of the criminal
aspect of the case to determine whether or not the accused, also the third party plaintiff, has a cause of action
against the third party defendant for the enforcement of its third party liability (TPL) under the insurance
contract.6Petitioner moved for reconsideration of said order, but the motion was denied; 7 hence, this petition.

It is the contention of herein petitioner that the dismissal of the third party complaint amounts to a denial or
curtailment of his right to defend himself in the civil aspect of the case. Petitioner further raises the legal question of
whether the accused in a criminal action for reckless imprudence, where the civil action is jointly prosecuted, can
legally implead the insurance company as third party defendant under its private car insurance policy, as one of his
modes of defense in the civil aspect of said proceedings.

On the other hand, the insurance company submits that a third party complaint is, under the rules, available only if
the defendant has a right to demand contribution, indemnity, subrogation or any other relief in respect of plaintiff's
claim, to minimize the number of lawsuits and avoid the necessity of bringing two (2) or more suits involving the
same subject matter. The insurance company further contends that the contract of motor vehicle insurance, the
damages and attorney's fees claimed by accused/third party plaintiff are matters entirely different from his criminal
liability in the reckless imprudence case, and that petitioner has no cause of action against the insurer until
petitioner's liability shall have been determined by final judgment, as stipulated in the contract of insurance. 8

Compulsory Motor Vehicle Liability Insurance (third party liability, or TPL) is primarily intended to provide
compensation for the death or bodily injuries suffered by innocent third parties or passengers as a result of a
negligent operation and use of motor vehicles.9 The victims and/or their dependents are assured of immediate
financial assistance, regardless of the financial capacity of motor vehicle owners.

The liability of the insurance company under the Compulsory Motor Vehicle Liability Insurance is for loss or damage.
Where an insurance policy insures directly against liability, the insurer's liability accrues immediately upon the
occurrence of the injury or event upon which the liability depends, and does not depend on the recovery of judgment
by the injured party against the insured. 10

The injured for whom the contract of insurance is intended can sue directly the insurer. The general purpose of
statutes enabling an injured person to proceed directly against the insurer is to protect injured persons against the
insolvency of the insured who causes such injury, and to give such injured person a certain beneficial interest in the
proceeds of the policy, and statutes are to be liberally construed so that their intended purpose may be
accomplished. It has even been held that such a provision creates a contractual relation which inures to the benefit
of any and every person who may be negligently injured by the named insured as if such injured person were
specifically named in the policy. 11

In the event that the injured fails or refuses to include the insurer as party defendant in his claim for indemnity
against the insured, the latter is not prevented by law to avail of the procedural rules intended to avoid multiplicity of
suits. Not even a "no action" clause under the policy-which requires that a final judgment be first obtained against
the insured and that only thereafter can the person insured recover on the policy can prevail over the Rules of Court
provisions aimed at avoiding multiplicity of suits. 12

In the instant case, the court a quo erred in dismissing petitioner's third party complaint on the ground that petitioner
had no cause of action yet against the insurance company (third party defendant). There is no need on the part of
the insured to wait for the decision of the trial court finding him guilty of reckless imprudence. The occurrence of the
injury to the third party immediately gave rise to the liability of the insurer under its policy.

A third party complaint is a device allowed by the rules of procedure by which the defendant can bring into the
original suit a party against whom he will have a claim for indemnity or remuneration as a result of a liability
established against him in the original suit.13 Third party complaints are allowed to minimize the number of lawsuits
and avoid the necessity of bringing two (2) or more actions involving the same subject matter. They are predicated
on the need for expediency and the avoidance of unnecessary lawsuits. If it appears probable that a second action
will result if the plaintiff prevails, and that this result can be avoided by allowing the third party complaint to remain,
then the motion to dismiss the third party complaint should be denied. 14

Respondent insurance company's contention that the third party complaint involves extraneous matter which will
only clutter, complicate and delay the criminal case is without merit. An offense causes two (2) classes of injuries
the first is the social injury produced by the criminal act which is sought to be repaired thru the imposition of the
corresponding penalty, and the second is the personal injury caused to the victim of the crime, which injury is sought
to be compensated thru indemnity, which is civil in nature. 15

In the instant case, the civil aspect of the offense charged, i.e., serious physical injuries allegedly suffered by
Jovencio Poblete, Sr., was impliedly instituted with the criminal case. Petitioner may thus raise all defenses
available to him insofar as the criminal and civil aspects of the case are concerned. The claim of petitioner for
payment of indemnity to the injured third party, under the insurance policy, for the alleged bodily injuries caused to
said third party, arose from the offense charged in the criminal case, from which the injured (Jovencio Poblete, Sr.)
has sought to recover civil damages. Hence, such claim of petitioner against the insurance company cannot be
regarded as not related to the criminal action.

WHEREFORE, the instant petition is GRANTED. The questioned order dated 24 April 1987 is SET ASIDE and a
new one entered admitting petitioner's third party complaint against the private respondent Makati Insurance
Company, Inc.
G.R. No. 48049 June 29, 1989

EMILIO TAN, JUANITO TAN, ALBERTO TAN and ARTURO TAN, petitioners,
vs.
THE COURT OF APPEALS and THE PHILIPPINE AMERICAN LIFE INSURANCE COMPANY, respondents.

O.F. Santos & P.C. Nolasco for petitioners.

Ferry, De la Rosa and Associates for private respondent.

GUTIERREZ, JR., J.:

This is a petition for review on certiorari of the Court of Appeals' decision affirming the decision of the Insurance
Commissioner which dismissed the petitioners' complaint against respondent Philippine American Life Insurance
Company for the recovery of the proceeds from their late father's policy. The facts of the case as found by the Court
of Appeals are:

Petitioners appeal from the Decision of the Insurance Commissioner dismissing herein petitioners'
complaint against respondent Philippine American Life Insurance Company for the recovery of the
proceeds of Policy No. 1082467 in the amount of P 80,000.00.

On September 23,1973, Tan Lee Siong, father of herein petitioners, applied for life insurance in the
amount of P 80,000.00 with respondent company. Said application was approved and Policy No.
1082467 was issued effective November 6,1973, with petitioners the beneficiaries thereof (Exhibit
A).

On April 26,1975, Tan Lee Siong died of hepatoma (Exhibit B). Petitioners then filed with respondent
company their claim for the proceeds of the life insurance policy. However, in a letter dated
September 11, 1975, respondent company denied petitioners' claim and rescinded the policy by
reason of the alleged misrepresentation and concealment of material facts made by the deceased
Tan Lee Siong in his application for insurance (Exhibit 3). The premiums paid on the policy were
thereupon refunded .

Alleging that respondent company's refusal to pay them the proceeds of the policy was unjustified
and unreasonable, petitioners filed on November 27, 1975, a complaint against the former with the
Office of the Insurance Commissioner, docketed as I.C. Case No. 218.

After hearing the evidence of both parties, the Insurance Commissioner rendered judgment on
August 9, 1977, dismissing petitioners' complaint. (Rollo, pp. 91-92)

The Court of Appeals dismissed ' the petitioners' appeal from the Insurance Commissioner's decision for lack of
merit

Hence, this petition.

The petitioners raise the following issues in their assignment of errors, to wit:

A. The conclusion in law of respondent Court that respondent insurer has the right to rescind the
policy contract when insured is already dead is not in accordance with existing law and applicable
jurisprudence.

B. The conclusion in law of respondent Court that respondent insurer may be allowed to avoid the
policy on grounds of concealment by the deceased assured, is contrary to the provisions of the
policy contract itself, as well as, of applicable legal provisions and established jurisprudence.
C. The inference of respondent Court that respondent insurer was misled in issuing the policy are
manifestly mistaken and contrary to admitted evidence. (Rollo, p. 7)

The petitioners contend that the respondent company no longer had the right to rescind the contract of insurance as
rescission must allegedly be done during the lifetime of the insured within two years and prior to the commencement
of action.

The contention is without merit.

The pertinent section in the Insurance Code provides:

Section 48. Whenever a right to rescind a contract of insurance is given to the insurer by any
provision of this chapter, such right must be exercised previous to the commencement of an action
on the contract.

After a policy of life insurance made payable on the death of the insured shall have been in force
during the lifetime of the insured for a period of two years from the date of its issue or of its last
reinstatement, the insurer cannot prove that the policy is void ab initio or is rescindable by reason of
the fraudulent concealment or misrepresentation of the insured or his agent.

According to the petitioners, the Insurance Law was amended and the second paragraph of Section 48 added to
prevent the insurance company from exercising a right to rescind after the death of the insured.

The so-called "incontestability clause" precludes the insurer from raising the defenses of false representations or
concealment of material facts insofar as health and previous diseases are concerned if the insurance has been in
force for at least two years during the insured's lifetime. The phrase "during the lifetime" found in Section 48 simply
means that the policy is no longer considered in force after the insured has died. The key phrase in the second
paragraph of Section 48 is "for a period of two years."

As noted by the Court of Appeals, to wit:

The policy was issued on November 6,1973 and the insured died on April 26,1975. The policy was
thus in force for a period of only one year and five months. Considering that the insured died before
the two-year period had lapsed, respondent company is not, therefore, barred from proving that the
policy is void ab initio by reason of the insured's fraudulent concealment or misrepresentation.
Moreover, respondent company rescinded the contract of insurance and refunded the premiums
paid on September 11, 1975, previous to the commencement of this action on November 27,1975.
(Rollo, pp. 99-100)

xxx xxx xxx

The petitioners contend that there could have been no concealment or misrepresentation by their late father
because Tan Lee Siong did not have to buy insurance. He was only pressured by insistent salesmen to do so. The
petitioners state:

Here then is a case of an assured whose application was submitted because of repeated visits and
solicitations by the insurer's agent. Assured did not knock at the door of the insurer to buy insurance.
He was the object of solicitations and visits.

Assured was a man of means. He could have obtained a bigger insurance, not just P 80,000.00. If
his purpose were to misrepresent and to conceal his ailments in anticipation of death during the two-
year period, he certainly could have gotten a bigger insurance. He did not.

Insurer Philamlife could have presented as witness its Medical Examiner Dr. Urbano Guinto. It was
he who accomplished the application, Part II, medical. Philamlife did not.
Philamlife could have put to the witness stand its Agent Bienvenido S. Guinto, a relative to Dr.
Guinto, Again Philamlife did not. (pp. 138139, Rollo)

xxx xxx xxx

This Honorable Supreme Court has had occasion to denounce the pressure and practice indulged in
by agents in selling insurance. At one time or another most of us have been subjected to that
pressure, that practice. This court took judicial cognizance of the whirlwind pressure of insurance
selling-especially of the agent's practice of 'supplying the information, preparing and answering the
application, submitting the application to their companies, concluding the transactions and
otherwise smoothing out all difficulties.

We call attention to what this Honorable Court said in Insular Life v. Feliciano, et al., 73 Phil. 201; at page 205:

It is of common knowledge that the selling of insurance today is subjected to the


whirlwind pressure of modern salesmanship.

Insurance companies send detailed instructions to their agents to solicit and procure applications.

These agents are to be found all over the length and breadth of the land. They are stimulated to
more active efforts by contests and by the keen competition offered by the other rival insurance
companies.

They supply all the information, prepare and answer the applications, submit the applications to their
companies, conclude the transactions, and otherwise smooth out all difficulties.

The agents in short do what the company set them out to do.

The Insular Life case was decided some forty years ago when the pressure of insurance
salesmanship was not overwhelming as it is now; when the population of this country was less than
one-fourth of what it is now; when the insurance companies competing with one another could be
counted by the fingers. (pp. 140-142, Rollo)

xxx xxx xxx

In the face of all the above, it would be unjust if, having been subjected to the whirlwind pressure of
insurance salesmanship this Court itself has long denounced, the assured who dies within the two-
year period, should stand charged of fraudulent concealment and misrepresentation." (p. 142, Rollo)

The legislative answer to the arguments posed by the petitioners is the "incontestability clause" added by the
second paragraph of Section 48.

The insurer has two years from the date of issuance of the insurance contract or of its last reinstatement within
which to contest the policy, whether or not, the insured still lives within such period. After two years, the defenses of
concealment or misrepresentation, no matter how patent or well founded, no longer lie. Congress felt this was a
sufficient answer to the various tactics employed by insurance companies to avoid liability. The petitioners'
interpretation would give rise to the incongruous situation where the beneficiaries of an insured who dies right after
taking out and paying for a life insurance policy, would be allowed to collect on the policy even if the insured
fraudulently concealed material facts.

The petitioners argue that no evidence was presented to show that the medical terms were explained in a layman's
language to the insured. They state that the insurer should have presented its two medical field examiners as
witnesses. Moreover, the petitioners allege that the policy intends that the medical examination must be conducted
before its issuance otherwise the insurer "waives whatever imperfection by ratification."

We agree with the Court of Appeals which ruled:


On the other hand, petitioners argue that no evidence was presented by respondent company to
show that the questions appearing in Part II of the application for insurance were asked, explained to
and understood by the deceased so as to prove concealment on his part. The same is not well
taken. The deceased, by affixing his signature on the application form, affirmed the correctness of all
the entries and answers appearing therein. It is but to be expected that he, a businessman, would
not have affixed his signature on the application form unless he clearly understood its significance.
For, the presumption is that a person intends the ordinary consequence of his voluntary act and
takes ordinary care of his concerns. [Sec. 5(c) and (d), Rule 131, Rules of Court].

The evidence for respondent company shows that on September 19,1972, the deceased was
examined by Dr. Victoriano Lim and was found to be diabetic and hypertensive; that by January,
1973, the deceased was complaining of progressive weight loss and abdominal pain and was
diagnosed to be suffering from hepatoma, (t.s.n. August 23, 1976, pp. 8-10; Exhibit 2). Another
physician, Dr. Wenceslao Vitug, testified that the deceased came to see him on December 14, 1973
for consolation and claimed to have been diabetic for five years. (t.s.n., Aug. 23,1976, p. 5; Exhibit 6)
Because of the concealment made by the deceased of his consultations and treatments for
hypertension, diabetes and liver disorders, respondent company was thus misled into accepting the
risk and approving his application as medically standard (Exhibit 5- C) and dispensing with further
medical investigation and examination (Exhibit 5-A). For as long as no adverse medical history is
revealed in the application form, an applicant for insurance is presumed to be healthy and physically
fit and no further medical investigation or examination is conducted by respondent company. (t.s.n.,
April 8,1976, pp. 6-8). (Rollo, pp. 96-98)

There is no strong showing that we should apply the "fine print" or "contract of adhesion" rule in this case. (Sweet
Lines, Inc. v. Teves, 83 SCRA 361 [1978]). The petitioners cite:

It is a matter of common knowledge that large amounts of money are collected from ignorant
persons by companies and associations which adopt high sounding titles and print the amount of
benefits they agree to pay in large black-faced type, following such undertakings by fine print
conditions which destroy the substance of the promise. All provisions, conditions, or exceptions
which in any way tend to work a forfeiture of the policy should be construed most strongly against
those for whose benefit they are inserted, and most favorably toward those against whom they are
meant to operate. (Trinidad v. Orient Protective Assurance Assn., 67 Phil. 184)

There is no showing that the questions in the application form for insurance regarding the insured's medical history
are in smaller print than the rest of the printed form or that they are designed in such a way as to conceal from the
applicant their importance. If a warning in bold red letters or a boxed warning similar to that required for cigarette
advertisements by the Surgeon General of the United States is necessary, that is for Congress or the Insurance
Commission to provide as protection against high pressure insurance salesmanship. We are limited in this petition
to ascertaining whether or not the respondent Court of Appeals committed reversible error. It is the petitioners'
burden to show that the factual findings of the respondent court are not based on substantial evidence or that its
conclusions are contrary to applicable law and jurisprudence. They have failed to discharge that burden.

WHEREFORE, the petition is hereby DENIED for lack of merit. The questioned decision of the Court of Appeals is
AFFIRMED.
G.R. No. 71360 July 16, 1986

DEVELOPMENT INSURANCE CORPORATION, petitioner,


vs.
INTERMEDIATE APPELLATE COURT, and PHILIPPINE UNION REALTY DEVELOPMENT
CORPORATION, respondents.

Balgos & Perez Law Offices for petitioner.

Agustin M. Sundiam for private respondent.

CRUZ, J.:

A fire occurred in the building of the private respondent and it sued for recovery of damages from the petitioner on
the basis of an insurance contract between them. The petitioner allegedly failed to answer on time and was declared
in default by the trial court. A judgment of default was subsequently rendered on the strength of the evidence
submitted ex parte by the private respondent, which was allowed full recovery of its claimed damages. On learning
of this decision, the petitioner moved to lift the order of default, invoking excusable neglect, and to vacate the
judgment by default. Its motion was denied. It then went to the respondent court, which affirmed the decision of the
trial court in toto. The petitioner is now before us, hoping presumably that it will fare better here than before the trial
court and the Intermediate Appellate Court. We shall see.

On the question of default, the record argues mightily against it. It is indisputable that summons was served on it,
through its senior vice-president, on June 19,1980. On July 14, 1980, ten days after the expiration of the original 15-
day period to answer (excluding July 4), its counsel filed an ex parte motion for an extension of five days within
which to file its answer. On July 18, 1980, the last day of the requested extension-which at the time had not yet been
granted-the same counsel filed a second motion for another 5-day extension, fourteen days after the expiry of the
original period to file its answer. The trial court nevertheless gave it five days from July 14, 1980, or until July 19,
1980, within which to file its answer. But it did not. It did so only on July 26, 1980, after the expiry of the original and
extended periods, or twenty-one days after the July 5, deadline. As a consequence, the trial court, on motion of the
private respondent filed on July 28, 1980, declared the petitioner in default. This was done almost one month later,
on August 25, 1980. Even so, the petitioner made no move at all for two months thereafter. It was only on October
27, 1980, more than one month after the judgment of default was rendered by the trial court on September 26, 1980,
that it filed a motion to lift the order of default and vacate the judgment by default.1

The pattern of inexcusable neglect, if not deliberate delay, is all too clear. The petitioner has slumbered on its right
and awakened too late. While it is true that in Trajano v. Cruz,2 which it cites, this Court declared "that judgments by default are generally
looked upon with disfavor," the default judgment in that case was set aside precisely because there was excusable neglect, Summons in that case was served
through "an employee in petitioners' office and not the person in-charge," whereas in the present case summons was served on the vice-president of the petitioner
who however refused to accept it. Furthermore, as Justice Guerrero noted, there was no evidence showing that the petitioners in Trajano intended to unduly delay
the case.

Besides, the petitioners in Trajano had a valid defense against the complaint filed against them, and this justified a
relaxation of the procedural rules to allow full hearing on the substantive issues raised. In the instant case, by
contrast, the petitioner must just the same fail on the merits even if the default orders were to be lifted. As the
respondent Court observed, "Nothing would be gained by having the order of default set aside considering the
appellant has no valid defense in its favor." 3

The petitioner's claim that the insurance covered only the building and not the elevators is absurd, to say the least.
This Court has little patience with puerile arguments that affront common sense, let alone basic legal principles with
which even law students are familiar. The circumstance that the building insured is seven stories high and so had to
be provided with elevators-a legal requirement known to the petitioner as an insurance company-makes its
contention all the more ridiculous.

No less preposterous is the petitioner's claim that the elevators were insured after the occurrence of the fire, a case
of shutting the barn door after the horse had escaped, so to speak.4 This pretense merits scant attention. Equally undeserving of
serious consideration is its submission that the elevators were not damaged by the fire, against the report of The arson investigators of the INP5 and, indeed, its
own expressed admission in its answer6 where it affirmed that the fire "damaged or destroyed a portion of the 7th floor of the insured building and more particularly
a Hitachi elevator control panel." 7

There is no reason to disturb the factual findings of the lower court, as affirmed by the Intermediate Appellate Court,
that the heat and moisture caused by the fire damaged although they did not actually burn the elevators. Neither is
this Court justified in reversing their determination, also factual, of the value of the loss sustained by the private
respondent in the amount of P508,867.00.

The only remaining question to be settled is the amount of the indemnity due to the private respondent under its
insurance contract with the petitioner. This will require an examination of this contract, Policy No. RY/F-082, as
renewed, by virtue of which the petitioner insured the private respondent's building against fire for P2,500,000.00. 8

The petitioner argues that since at the time of the fire the building insured was worth P5,800,000.00, the private
respondent should be considered its own insurer for the difference between that amount and the face value of the
policy and should share pro rata in the loss sustained. Accordingly, the private respondent is entitled to an indemnity
of only P67,629.31, the rest of the loss to be shouldered by it alone. In support of this contention, the petitioner cites
Condition 17 of the policy, which provides:

If the property hereby insured shall, at the breaking out of any fire, be collectively of greater value
than the sum insured thereon then the insured shall be considered as being his own insurer for the
difference, and shall bear a ratable proportion of the loss accordingly. Every item, if more than one,
of the policy shall be separately subject to this condition.

However, there is no evidence on record that the building was worth P5,800,000.00 at the time of the loss; only the
petitioner says so and it does not back up its self-serving estimate with any independent corroboration. On the
contrary, the building was insured at P2,500,000.00, and this must be considered, by agreement of the insurer and
the insured, the actual value of the property insured on the day the fire occurred. This valuation becomes even more
believable if it is remembered that at the time the building was burned it was still under construction and not yet
completed.

The Court notes that Policy RY/F-082 is an open policy and is subject to the express condition that:

Open Policy

This is an open policy as defined in Section 57 of the Insurance Act. In the event of loss, whether
total or partial, it is understood that the amount of the loss shall be subject to appraisal and the
liability of the company, if established, shall be limited to the actual loss, subject to the applicable
terms, conditions, warranties and clauses of this Policy, and in no case shall exceed the amount of
the policy.

As defined in the aforestated provision, which is now Section 60 of the Insurance Code, "an open policy is one in
which the value of the thing insured is not agreed upon but is left to be ascertained in case of loss. " This means that
the actual loss, as determined, will represent the total indemnity due the insured from the insurer except only that
the total indemnity shall not exceed the face value of the policy.

The actual loss has been ascertained in this case and, to repeat, this Court will respect such factual determination in
the absence of proof that it was arrived at arbitrarily. There is no such showing. Hence, applying the open policy
clause as expressly agreed upon by the parties in their contract, we hold that the private respondent is entitled to
the payment of indemnity under the said contract in the total amount of P508,867.00.

The refusal of its vice-president to receive the private respondent's complaint, as reported in the sheriff's return, was
the first indication of the petitioner's intention to prolong this case and postpone the discharge of its obligation to the
private respondent under this agreement. That intention was revealed further in its subsequent acts-or inaction-
which indeed enabled it to avoid payment for more than five years from the filing of the claim against it in 1980. The
petitioner has temporized long enough to avoid its legitimate responsibility; the delay must and does end now.

WHEREFORE, the appealed decision is affirmed in full, with costs against the petitioner.
G.R. No. 92383 July 17, 1992

SUN INSURANCE OFFICE, LTD., petitioner,


vs.
THE HON. COURT OF APPEALS and NERISSA LIM, respondents.

CRUZ, J.:

The petitioner issued Personal Accident Policy No. 05687 to Felix Lim, Jr. with a face value of P200,000.00. Two
months later, he was dead with a bullet wound in his head. As beneficiary, his wife Nerissa Lim sought payment on
the policy but her claim was rejected. The petitioner agreed that there was no suicide. It argued, however that there
was no accident either.

Pilar Nalagon, Lim's secretary, was the only eyewitness to his death. It happened on October 6, 1982, at about 10
o'clock in the evening, after his mother's birthday party. According to Nalagon, Lim was in a happy mood (but not
drunk) and was playing with his handgun, from which he had previously removed the magazine. As she watched
television, he stood in front of her and pointed the gun at her. She pushed it aside and said it might he loaded. He
assured her it was not and then pointed it to his temple. The next moment there was an explosion and Lim slumped
to the floor. He was dead before he fell. 1

The widow sued the petitioner in the Regional Trial Court of Zamboanga City and was sustained. 2 The petitioner
was sentenced to pay her P200,000.00, representing the face value of the policy, with interest at the legal rate;
P10,000.00 as moral damages; P5,000.00 as exemplary damages; P5,000.00 as actual and compensatory
damages; and P5,000.00 as attorney's fees, plus the costs of the suit. This decision was affirmed on appeal, and
the motion for reconsideration was denied. 3 The petitioner then came to this Court to fault the Court of Appeals for
approving the payment of the claim and the award of damages.

The term "accident" has been defined as follows:

The words "accident" and "accidental" have never acquired any technical signification in law, and when used in an
insurance contract are to be construed and considered according to the ordinary understanding and common usage
and speech of people generally. In-substance, the courts are practically agreed that the words "accident" and
"accidental" mean that which happens by chance or fortuitously, without intention or design, and which is
unexpected, unusual, and unforeseen. The definition that has usually been adopted by the courts is that an accident
is an event that takes place without one's foresight or expectation an event that proceeds from an unknown
cause, or is an unusual effect of a known case, and therefore not expected. 4

An accident is an event which happens without any human agency or, if happening through human agency, an
event which, under the circumstances, is unusual to and not expected by the person to whom it happens. It has also
been defined as an injury which happens by reason of some violence or casualty to the injured without his design,
consent, or voluntary co-operation. 5

In light of these definitions, the Court is convinced that the incident that resulted in Lim's death was indeed an
accident. The petitioner, invoking the case of De la Cruz v. Capital Insurance, 6 says that "there is no accident when
a deliberate act is performed unless some additional, unexpected, independent and unforeseen happening occurs
which produces or brings about their injury or death." There was such a happening. This was the firing of the gun,
which was the additional unexpected and independent and unforeseen occurrence that led to the insured person's
death.

The petitioner also cites one of the four exceptions provided for in the insurance contract and contends that the
private petitioner's claim is barred by such provision. It is there stated:

Exceptions

The company shall not be liable in respect of


1. Bodily injury

xxx xxx xxx

b. consequent upon

i) The insured person attempting to commit suicide or willfully exposing himself to needless peril
except in an attempt to save human life.

To repeat, the parties agree that Lim did not commit suicide. Nevertheless, the petitioner contends that the insured
willfully exposed himself to needless peril and thus removed himself from the coverage of the insurance policy.

It should be noted at the outset that suicide and willful exposure to needless peril are in pari materia because they
both signify a disregard for one's life. The only difference is in degree, as suicide imports a positive act of ending
such life whereas the second act indicates a reckless risking of it that is almost suicidal in intent. To illustrate, a
person who walks a tightrope one thousand meters above the ground and without any safety device may not
actually be intending to commit suicide, but his act is nonetheless suicidal. He would thus be considered as "willfully
exposing himself to needless peril" within the meaning of the exception in question.

The petitioner maintains that by the mere act of pointing the gun to hip temple, Lim had willfully exposed himself to
needless peril and so came under the exception. The theory is that a gun is per se dangerous and should therefore
be handled cautiously in every case.

That posture is arguable. But what is not is that, as the secretary testified, Lim had removed the magazine from the
gun and believed it was no longer dangerous. He expressly assured her that the gun was not loaded. It is submitted
that Lim did not willfully expose himself to needless peril when he pointed the gun to his temple because the fact is
that he thought it was not unsafe to do so. The act was precisely intended to assure Nalagon that the gun was
indeed harmless.

The contrary view is expressed by the petitioner thus:

Accident insurance policies were never intended to reward the insured for his tendency to show off
or for his miscalculations. They were intended to provide for contingencies. Hence, when I
miscalculate and jump from the Quezon Bridge into the Pasig River in the belief that I can overcome
the current, I have wilfully exposed myself to peril and must accept the consequences of my act. If I
drown I cannot go to the insurance company to ask them to compensate me for my failure to swim
as well as I thought I could. The insured in the case at bar deliberately put the gun to his head and
pulled the trigger. He wilfully exposed himself to peril.

The Court certainly agrees that a drowned man cannot go to the insurance company to ask for compensation. That
might frighten the insurance people to death. We also agree that under the circumstances narrated, his beneficiary
would not be able to collect on the insurance policy for it is clear that when he braved the currents below,
he deliberately exposed himself to a known peril.

The private respondent maintains that Lim did not. That is where she says the analogy fails. The petitioner's
hypothetical swimmer knew when he dived off the Quezon Bridge that the currents below were dangerous. By
contrast, Lim did not know that the gun he put to his head was loaded.

Lim was unquestionably negligent and that negligence cost him his own life. But it should not prevent his widow
from recovering from the insurance policy he obtained precisely against accident. There is nothing in the policy that
relieves the insurer of the responsibility to pay the indemnity agreed upon if the insured is shown to have contributed
to his own accident. Indeed, most accidents are caused by negligence. There are only four exceptions expressly
made in the contract to relieve the insurer from liability, and none of these exceptions is applicable in the case at
bar. **

It bears noting that insurance contracts are as a rule supposed to be interpreted liberally in favor of the assured.
There is no reason to deviate from this rule, especially in view of the circumstances of this case as above analyzed.
On the second assigned error, however, the Court must rule in favor of the petitioner. The basic issue raised in this
case is, as the petitioner correctly observed, one of first impression. It is evident that the petitioner was acting in
good faith then it resisted the private respondent's claim on the ground that the death of the insured was covered by
the exception. The issue was indeed debatable and was clearly not raised only for the purpose of evading a
legitimate obligation. We hold therefore that the award of moral and exemplary damages and of attorney's fees is
unjust and so must be disapproved.

In order that a person may be made liable to the payment of moral damages, the law requires that
his act be wrongful. The adverse result of an action does not per se make the act wrongful and
subject the act or to the payment of moral damages. The law could not have meant to impose a
penalty on the right to litigate; such right is so precious that moral damages may not be charged on
those who may exercise it erroneously. For these the law taxes costs. 7

The fact that the results of the trial were adverse to Barreto did not alone make his act in bringing the
action wrongful because in most cases one party will lose; we would be imposing an unjust condition
or limitation on the right to litigate. We hold that the award of moral damages in the case at bar is not
justified by the facts had circumstances as well as the law.

If a party wins, he cannot, as a rule, recover attorney's fees and litigation expenses, since it is not
the fact of winning alone that entitles him to recover such damages of the exceptional circumstances
enumerated in Art. 2208. Otherwise, every time a defendant wins, automatically the plaintiff must
pay attorney's fees thereby putting a premium on the right to litigate which should not be so. For
those expenses, the law deems the award of costs as sufficient. 8
G.R. No. 103883 November 14, 1996

JACQUELINE JIMENEZ VDA. DE GABRIEL, petitioner,


vs.
HON. COURT OF APPEALS and FORTUNE INSURANCE & SURETY COMPANY, INC., respondents.

VITUG, J.:

The petition for review on certiorari in this case seeks the reversal of the decision 1 of the Court of Appeals setting
aside the judgment of the Regional Trial Court of Manila, Branch 55, which has ordered private respondent Fortune
Insurance & Surety Company, Inc., to pay petitioner Jacqueline Jimenez vda. de Gabriel, the surviving spouse and
beneficiary in an accident (group) insurance of her deceased husband, the amount of P100,000.00, plus legal
interest.

Marcelino Gabriel, the insured, was employed by Emerald Construction & Development Corporation ("ECDC") at its
construction project in Iraq. He was covered by a personal accident insurance in the amount of P100,000.00 under a
group policy 2 procured from private respondent by ECDC for its overseas workers. The insured risk was for "(b)odily
injury caused by violent accidental external and visible means which injury (would) solely and independently of any
other cause" 3 result in death or disability.

On 22 May 1982, within the life of the policy, Gabriel died in Iraq. A year later, or on 12 July 1983, ECDC reported
Gabriel's death to private respondent by telephone. 4 Among the documents thereafter submitted to private
respondent were a copy of the death certificate 5 issued by the Ministry of Health of the Republic of Iraq which
stated

REASON OF DEATH: UNDER EXAMINATION NOW NOT YET KNOWN 6

and an autopsy report 7 of the National Bureau of Investigation ("NBI") to the effect that "(d)ue to advanced
state of postmortem decomposition, cause of death (could) not be determined." 8 Private respondent referred
the insurance claim to Mission Adjustment Service, Inc.

Following a series of communications between petitioner and private respondent, the latter, on 22 September 1983,
ultimately denied the claim of ECDC on the ground of prescription. 9 Petitioner went to the Regional Trial Court of
Manila. In her complaint against ECDC and private respondent, she averred that her husband died of electrocution
while in the performance of his work and prayed for the recovery of P100,000.00 for insurance indemnification and
of various other sums by way of actual, moral, and exemplary damages, plus attorney's fees and costs of suit.

Private respondent filed its answer, which was not verified, admitting the genuineness and due execution of the
insurance policy; it alleged, however, that since both the death certificate issued by the Iraqi Ministry of Health and
the autopsy report of the NBI failed to disclose the cause of Gabriel's death, it denied liability under the policy. In
addition, private respondent raised the defense of "prescription," invoking Section 384 10 of the Insurance Code.
Later, private respondent filed an amended answer, still unverified, reiterating its original defenses but, this time,
additionally putting up a counterclaim and a crossclaim.

The trial court dismissed the case against ECDC for the failure of petitioner to take steps to cause the service of the
fourth alias summons on ECDC. The dismissal was without prejudice.

The case proceeded against private respondent alone. On 28 May 1987, the trial court rendered its decision 11 in
favor (partly) of petitioner's claim. In arriving at its conclusion, the trial court held that private respondent was
deemed to have waived the defense, i.e., that the cause of Gabriel's death was not covered by the policy, when the
latter failed to impugn by evidence petitioner's averment on the matter. With regard to the defense of prescription,
the court considered the complaint to have been timely filed or within one (1) year from private respondent's denial
of the claim.
Petitioner and private respondent both appealed to the Court of Appeals. Petitioner contended that the lower court
should have awarded all the claims she had asked for. Private respondent asserted, on its part, that the lower court
erred in ruling (a) that the insurer had waived the defense that Gabriel's death was not caused by the insured peril
("violent accidental external and visible means") specified in the policy and (b) that the cause of action had not
prescribed.

The Court of Appeals, on 18 September 1991, reversed the decision of the lower court. The appellate court held that
petitioner had failed to substantiate her allegation that her husband's death was caused by a risk insured against.
The appellate court observed that the only evidence presented by petitioner, in her attempt to show the
circumstances that led to the death of the insured, were her own affidavit and a letter allegedly written by a co-
worker of the deceased in Iraq which, unfortunately for her, were held to be both
hearsay. 12

The motion for reconsideration was denied. 13

Petitioner's recourse to this Court must also fail.

On the issue of "prescription," private respondent correctly invoked Section 384 of the Insurance Code; viz:

Sec. 384. Any person having any claim upon the policy issued pursuant to this chapter shall, without
any unnecessary delay, present to the insurance company concerned a written notice of claim
setting forth the nature, extent and duration of the injuries sustained as certified by a duly licensed
physician. Notice of claim must be filed within six months from date of the accident, otherwise, the
claim shall be deemed waived. Action or suit for recovery of damage due to loss or injury must be
brought, in proper cases, with the Commissioner or the Courts within one year from denial of the
claim, otherwise, the claimant's right of action shall prescribe.

The notice of death was given to private respondent, concededly, more than a year after the death of
petitioner's husband. Private respondent, in invoking prescription, was not referring to the one-year period
from the denial of the claim within which to file an action against an insurer but obviously to the written notice
of claim that had to be submitted within six months from the time of the accident.

Petitioner argues that private respondent must be deemed to have waived its right to controvert the claim, that is, to
show that the cause of death is an excepted peril, by failing to have its answers (to the Request for Admission sent
by petitioner) duly verified. It is true that a matter of which a written request for admission is made shall be deemed
impliedly admitted "unless, within a period designated in the request, which shall not be less than ten (10) days after
service thereof, or within such further time as the court may allow on motion and notice, the party to whom the
request is directed serves upon the party requesting the admission a sworn statement either denying specifically the
matters of which an admission is requested or setting forth in detail the reasons why he cannot truthfully either admit
or deny those matters;" 14 however, the verification, like in most cases required by the rules of procedure, is a formal,
not jurisdictional, requirement, and mainly intended to secure an assurance that matters which are alleged are done
in good faith or are true and correct and not of mere speculation. When circumstances warrant, the court may simply
order the correction of unverified pleadings or act on it and waive strict compliance with the rules in order that the
ends of justice may thereby be served. 15 In the case of answers to written requests for admission particularly, the
court can allow the party making the admission, whether made expressly or deemed to have been made impliedly,
"to withdraw or amend it upon such terms as may be just." 16

The appellate court acted neither erroneously nor with grave abuse of discretion when it seconded the court a
quoand ruled:

As to the allegation of the plaintiff-appellant that the matters requested by her to be admitted by the
defendant-appellant under the Request for Admission were already deemed admitted by the latter
for its failure to answer it under oath, has already been properly laid to rest when the lower court in
its Order of May 28, 1987 correctly ruled:

At the outset, it must be stressed that the defendant indeed filed a written answer to
the request for admission, sans verification. The case of Motor Service
Co., Inc. vs. Yellow Taxicab Co., Inc., et al. may not therefore be controlling, or
actually opposite. In said case, there was an absolute failure on the part of the
defendant to answer the request for admission, and thus the court was justified in
rendering a summary judgment. Here, however, as clearly intimated elsewhere
above, the defendant answered in writing practically every question posed in the
request for admission. The Court believes, under the peculiar circumstance, that the
more controlling jurisprudence on the mater would be those cited by the defendant in
its memorandum, particularly the case of Quimpo vs. de la Victoria, 46 SCRA 139.

Prescinding from the foregoing, there is absolutely no basis in fact and in law for the lower court to
hold that the appellant insurance company was deemed to have waived the defense, that the death
of plaintiff-appellant's husband was not caused by violent accidental external and visible means' as
contemplated in the insurance policy. The Death Certificate (Exh. 9) and the Autopsy Report (Exh.
10), more than controverted the allegation of the plaintiff-appellant as to the cause of death of her
husband. 17

The insurance policy expressly provided that to be compensable, the injury or death should be caused by "violent
accidental external and visible means." In attempting to prove the cause of her husband's death, all that petitioner
could submit were a letter sent to her by her husband's co-worker, stating that Gabriel died when he tried to haul
water out of a tank while its submerged motor was still functioning, 18 and petitioner's sinumpaang
salaysay 19 which merely confirmed the receipt and stated contents of the letter. Said the appellate court in this
regard:

. . . . It must be noted that the only evidence presented by her to prove the circumstances
surrounding her husband's death were her purported affidavit and the letter allegedly written by the
deceased co-worker in Iraq. The said affidavit however suffers from procedural infirmity as it was not
even testified to or identified by the affiant (plaintiff-appellant) herself. This self-serving affidavit
therefore is a mere hearsay under the rules, . . . .

xxx xxx xxx

In like manner, the letter allegedly written by the deceased's co-worker which was never identified to
in court by the supposed author, suffers from the same defect as the affidavit of the plaintiff-
appellant. 20

Not one of the other documents submitted, to wit, the POEA decision, dated 06 June 1984, 21 the death
certificate issued by the Ministry of Health of Iraq and the NBI autopsy report, 22 could give any probative
value to petitioner's claim. The POEA decision did not make any categorical holding on the specific cause of
Gabriel's death. Neither did the death certificate issued by the health authorities in Iraq nor the NBI autopsy
report provide any clue on the cause of death. All that appeared to be clear was the fact of Gabriel's demise
on 22 May 1982 in Iraq.

Evidence, in fine, is utterly wanting to establish that the insured suffered from an accidental death, the risk covered
by the policy. In an accident insurance, the insured's beneficiary has the burden of proof in demonstrating that the
cause of death is due to the covered peril. Once that fact is established, the burden then shifts to the insurer to show
any excepted peril that may have been stipulated by the parties. An "accident insurance" is not thus to be likened to
an ordinary life insurance where the insured's death, regardless of the cause thereof, would normally be
compensable. The latter is akin in property insurance to an "all risk" coverage where the insured, on the aspect of
burden of proof, has merely to show the condition of the property insured when the policy attaches and the fact of
loss or damage during the period of the policy and where, thereafter, the burden would be on the insurer to show
any "excluded peril." When, however, the insured risk is specified, like in the case before us, it lies with the claimant
of the insurance proceeds to initially prove that the loss is caused by the covered peril.

While petitioner did fail in substantiating her allegation that the death of her husband was due to an accident,
considering, however, the uncertainty on the real cause of death, private respondent might find its way clear into still
taking a second look on the matter and perhaps help ease the load of petitioner's loss.

WHEREFORE, the decision appealed from is AFFIRMED. No costs.


G.R. No. L-67835 October 12, 1987

MALAYAN INSURANCE CO., INC. (MICO), petitioner,


vs.
GREGORIA CRUZ ARNALDO, in her capacity as the INSURANCE COMMISSIONER, and CORONACION
PINCA, respondents.

CRUZ, J.:

When a person's house is razed, the fire usually burns down the efforts of a lifetime and forecloses hope for the
suddenly somber future. The vanished abode becomes a charred and painful memory. Where once stood a home,
there is now, in the sighing wisps of smoke, only a gray desolation. The dying embers leave ashes in the heart.

For peace of mind and as a hedge against possible loss, many people now secure fire insurance. This is an aleatory
contract. By such insurance, the insured in effect wagers that his house will be burned, with the insurer assuring him
against the loss, for a fee. If the house does burn, the insured, while losing his house, wins the wagers. The prize is
the recompense to be given by the insurer to make good the loss the insured has sustained.

It would be a pity then if, having lost his house, the insured were also to lose the payment he expects to recover for
such loss. Sometimes it is his fault that he cannot collect, as where there is a defect imputable to him in the
insurance contract. Conversely, the reason may be an unjust refusal of the insurer to acknowledge a just obligation,
as has happened many times.

In the instant case the private respondent has been sustained by the Insurance Commission in her claim for
compensation for her burned property. The petitioner is now before us to dispute the decision, 1 on the ground that there
was no valid insurance contract at the time of the loss.

The chronology of the relevant antecedent facts is as follows:

On June 7, 1981, the petitioner (hereinafter called (MICO) issued to the private respondent, Coronacion Pinca, Fire
Insurance Policy No. F-001-17212 on her property for the amount of P14,000.00 effective July 22, 1981, until July
22, 1982. 2

On October 15,1981, MICO allegedly cancelled the policy for non-payment, of the premium and sent the
corresponding notice to Pinca. 3

On December 24, 1981, payment of the premium for Pinca was received by DomingoAdora, agent of MICO. 4

On January 15, 1982, Adora remitted this payment to MICO,together with other payments. 5

On January 18, 1982, Pinca's property was completely burned. 6

On February 5, 1982, Pinca's payment was returned by MICO to Adora on the ground that her policy had been
cancelled earlier. But Adora refused to accept it. 7

In due time, Pinca made the requisite demands for payment, which MICO rejected. She then went to the Insurance
Commission. It is because she was ultimately sustained by the public respondent that the petitioner has come to us
for relief.

From the procedural viewpoint alone, the petition must be rejected. It is stillborn.

The records show that notice of the decision of the public respondent dated April 5, 1982, was received by MICO on
April 10, 1982. 8 On April 25, 1982, it filed a motion for reconsideration, which was denied on June 4, 1982. 9 Notice
of this denial was received by MICO on June 13, 1982, as evidenced by Annex "1" duly authenticated by the
Insurance Commission. 10 The instant petition was filed with this Court on July 2, 1982. 11
The position of the petition is that the petition is governed by Section 416 0f the Insurance Code giving it thirty days
wthin which to appeal by certiorari to this Court. Alternatively, it also invokes Rule 45 of the Rules of Court. For their
part, the public and private respondents insist that the applicable law is B.P. 129, which they say governs not only
courts of justice but also quasi-judicial bodies like the Insurance Commission. The period for appeal under this law is
also fifteen days, as under Rule 45.

The pivotal date is the date the notice of the denial of the motion for reconsideration was received by MICO.

MICO avers this was June 18, 1982, and offers in evidence its Annex "B," 12 which is a copy of the Order of June 14, 1982, with a
signed rubber-stamped notation on the upper left-hand corner that it was received on June 18, 1982, by its legal department. It does not indicate from whom. At
the bottom, significantly, there is another signature under which are the ciphers "6-13-82," for which no explanation has been given.

Against this document, the private respodent points in her Annex "1," 13 the authenticated copy of the same Order with a rubber-
stamped notation at the bottom thereof indicating that it was received for the Malayan Insurance Co., Inc. by J. Gotladera on "6-13-82." The signature may or may
not habe been written by the same person who signed at the bottom of the petitioner's Annex "B."

Between the two dates, the court chooses to believe June 13, 1982, not only because the numbers "6-13-82"
appear on both annexes but also because it is the date authenticated by the administrative division of the Insurance
Commission. Annex "B" is at worst self-serving; at best, it might only indicate that it was received on June 18, 1982,
by the legal department of MICO, after it had been received earlier by some other of its personnel on June 13, 1982.
Whatever the reason for the delay in transmitting it to the legal department need not detain us here.

Under Section 416 of the Insurance Code, the period for appeal is thirty days from notice of the decision of the
Insurance Commission. The petitioner filed its motion for reconsideration on April 25, 1981, or fifteen days such
notice, and the reglementary period began to run again after June 13, 1981, date of its receipt of notice of the denial
of the said motion for reconsideration. As the herein petition was filed on July 2, 1981, or nineteen days later, there
is no question that it is tardy by four days.

Counted from June 13, the fifteen-day period prescribed under Rule 45, assuming it is applicable, would end on
June 28, 1982, or also four days from July 2, when the petition was filed.

If it was filed under B.P. 129, then, considering that the motion for reconsideration was filed on the fifteenth day after
MICO received notice of the decision, only one more day would have remained for it to appeal, to wit, June 14,
1982. That would make the petition eighteen days late by July 2.

Indeed, even if the applicable law were still R.A. 5434, governing appeals from administrative bodies, the petition
would still be tardy. The law provides for a fixed period of ten days from notice of the denial of a seasonable motion
for reconsideration within which to appeal from the decision. Accordingly, that ten-day period, counted from June 13,
1982, would have ended on June 23, 1982, making the petition filed on July 2, 1982, nine days late.

Whichever law is applicable, therefore, the petition can and should be dismissed for late filing.

On the merits, it must also fail. MICO's arguments that there was no payment of premium and that the policy had
been cancelled before the occurence of the loss are not acceptable. Its contention that the claim was allowed
without proof of loss is also untenable.

The petitioner relies heavily on Section 77 of the Insurance Code providing that:

SEC. 77. An insurer is entitled to payment of the premium as soon as the thing is exposed to the
peril insured against. Notwithstanding any agreement to the contrary, no policy or contract of
insurance issued by an insurance company is valid and binding unless and until the premium thereof
has been paid, except in the case of a life or an industrial life policy whenever the grace period
provision applies.

The above provision is not applicable because payment of the premium was in fact eventually made in this case.
Notably, the premium invoice issued to Pinca at the time of the delivery of the policy on June 7, 1981 was stamped
"Payment Received" of the amoung of P930.60 on "12-24-81" by Domingo Adora. 14 This is important because it suggests an
understanding between MICO and the insured that such payment could be made later, as agent Adora had assured Pinca. In any event, it is not denied that this
payment was actually made by Pinca to Adora, who remitted the same to MICO.
The payment was made on December 24, 1981, and the fire occured on January 18, 1982. One wonders: suppose
the payment had been made and accepted in, say, August 1981, would the commencement date of the policy have
been changed to the date of the payment, or would the payment have retroacted to July 22, 1981? If MICO
accepted the payment in December 1981 and the insured property had not been burned, would that policy not have
expired just the same on July 22, 1982, pursuant to its original terms, and not on December 24, 1982?

It would seem from MICO's own theory, that the policy would have become effective only upon payment, if accepted
and so would have been valid only from December 24, 1981m but only up to July 22, 1981, according to the original
terms. In others words, the policy would have run for only eight months although the premium paid was for one
whole year.

It is not disputed that the preium was actually paid by Pinca to Adora on December 24, 1981, who received it on
behalf of MICO, to which it was remitted on January 15, 1982. What is questioned is the validity of Pinca's payment
and of Adora's authority to receive it.

MICO's acknowledgment of Adora as its agent defeats its contention that he was not authorized to receive the
premium payment on its behalf. It is clearly provided in Section 306 of the Insurance Code that:

SEC. 306. xxx xxx xxx

Any insurance company which delivers to an insurance agant or insurance broker a policy or
contract of insurance shall be demmed to have authorized such agent or broker to receive on its
behalf payment of any premium which is due on such policy or contract of insurance at the time of its
issuance or delivery or which becomes due thereon.

And it is a well-known principle under the law of agency that:

Payment to an agent having authority to receive or collect payment is equivalent to payment to the
principal himself; such payment is complete when the money delivered is into the agent's hands and
is a discharge of the indebtedness owing to the principal. 15

There is the petitioner's argument, however, that Adora was not authorized to accept the premium payment because six months had elapsed since the issuance
by the policy itself. It is argued that this prohibition was binding upon Pinca, who made the payment to Adora at her own riskl as she was bound to first check his
authority to receive it. 16

MICO is taking an inconsistent stand. While contending that acceptance of the premium payment was prohibited by the policy, it at the same time insists that the
policy never came into force because the premium had not been paid. One surely, cannot have his cake and eat it too.

We do not share MICO's view that there was no existing insurance at the time of the loss sustained by Pinca
because her policy never became effective for non-payment of premium. Payment was in fact made, rendering the
policy operative as of June 22, 1981, and removing it from the provisions of Article 77, Thereafter, the policy could
be cancelled on any of the supervening grounds enumerated in Article 64 (except "nonpayment of premium")
provided the cancellation was made in accordance therewith and with Article 65.

Section 64 reads as follows:

SEC. 64. No policy of insurance other than life shall be cancelled by the insurer except upon prior
notice thereof to the insured, and no notice of cancellation shall be effective unless it is based on the
occurrence, after the effective date of the policy, of one or more of the following:

(a) non-payment of premium;

(b) conviction of a crime arising out of acts increasing the hazard insured against;

(c) discovery of fraud or material misrepresentation;

(d) discovery of willful, or reckless acts or commissions increasing the hazard insured against;
(e) physical changes in the property insured which result in the property becoming uninsurable;or

(f) a determination by the Commissioner that the continuation of the policy would violate or would
place the insurer in violation of this Code.

As for the method of cancellation, Section 65 provides as follows:

SEC. 65. All notices of cancellation mentioned in the preceding section shall be in writing, mailed or
delivered to the named insured at the address shown in the policy, and shall state (a) which of the
grounds set forth in section sixty-four is relied upon and (b) that, upon written request of the named
insured, the insurer will furnish the facts on which the cancellation is based.

A valid cancellation must, therefore, require concurrence of the following conditions:

(1) There must be prior notice of cancellation to the insured; 17

(2) The notice must be based on the occurrence, after the effective date of the policy, of one or more of the grounds mentioned;18

(3) The notice must be (a) in writing, (b) mailed, or delivered to the named insured, (c) at the address shown in the
policy; 19

(4) It must state (a) which of the grounds mentioned in Section 64 is relied upon and (b) that upon written request of the insured, the insurer will furnish the facts on
which the cancellation is based. 20

MICO's claims it cancelled the policy in question on October 15, 1981, for non-payment of premium. To support this
assertion, it presented one of its employees, who testified that "the original of the endorsement and credit memo"
presumably meaning the alleged cancellation "were sent the assured by mail through our mailing
section" 21However, there is no proof that the notice, assuming it complied with the other requisites mentioned
above, was actually mailed to and received by Pinca. All MICO's offers to show that the cancellation was
communicated to the insured is its employee's testimony that the said cancellation was sent "by mail through our
mailing section." without more. The petitioner then says that its "stand is enervated (sic) by the legal presumption of
regularity and due performance of duty." 22 (not realizing perhaps that "enervated" means "debilitated" not
"strengthened").

On the other hand, there is the flat denial of Pinca, who says she never received the claimed cancellation and who,
of course, did not have to prove such denial Considering the strict language of Section 64 that no insurance policy
shall be cancelled except upon prior notice, it behooved MICO's to make sure that the cancellation was actually sent
to and received by the insured. The presumption cited is unavailing against the positive duty enjoined by Section 64
upon MICO and the flat denial made by the private respondent that she had received notice of the claimed
cancellation.

It stands to reason that if Pinca had really received the said notice, she would not have made payment on the
original policy on December 24, 1981. Instead, she would have asked for a new insurance, effective on that date
and until one year later, and so taken advantage of the extended period. The Court finds that if she did pay on that
date, it was because she honestly believed that the policy issued on June 7, 1981, was still in effect and she was
willing to make her payment retroact to July 22, 1981, its stipulated commencement date. After all, agent Adora was
very accomodating and had earlier told her "to call him up any time" she was ready with her payment on the policy
earlier issued. She was obviously only reciprocating in kind when she paid her premium for the period beginning
July 22, 1981, and not December 24, 1981.

MICO's suggests that Pinca knew the policy had already been cancelled and that when she paid the premium on
December 24, 1981, her purpose was "to renew it." As this could not be done by the agent alone under the terms of
the original policy, the renewal thereof did not legally bind MICO. which had not ratified it. To support this argument,
MICO's cites the following exchange:

Q: Now, Madam Witness, on December 25th you made the alleged payment. Now,
my question is that, did it not come to your mind that after the lapse of six (6) months,
your policy was cancelled?
A: I have thought of that but the agent told me to call him up at anytime.

Q: So if you thought that your policy was already intended to revive cancelled policy?

A: Misleading, Your Honor.

Hearing Officer: The testimony of witness is that, she thought of that.

Q: I will revise the question. Now, Mrs. Witness, you stated that you thought the
policy was cancelled. Now, when you made the payment of December 24, 1981,
your intention was to revive the policy if it was already cancelled?

A: Yes, to renew it. 23

A close study of the above transcript will show that Pinca meant to renew the policy if it had really been already
cancelled but not if it was stffl effective. It was all conditional. As it has not been shown that there was a valid
cancellation of the policy, there was consequently no need to renew it but to pay the premium thereon. Payment
was thus legally made on the original transaction and it could be, and was, validly received on behalf of the insurer
by its agent Adora. Adora. incidentally, had not been informed of the cancellation either and saw no reason not to
accept the said payment.

The last point raised by the petitioner should not pose much difficulty. The valuation fixed in fire insurance policy is
conclusive in case of total loss in the absence of fraud, 24 which is not shown here. Loss and its amount may be
determined on the basis of such proof as may be offered by the insured, which need not be of such persuasiveness
as is required in judicial proceedings. 25 If, as in this case, the insured files notice and preliminary proof of loss and
the insurer fails to specify to the former all the defects thereof and without unnecessary delay, all objections to
notice and proof of loss are deemed waived under Section 90 of the Insurance Code.

The certification 26 issued by the Integrated National Police, Lao-ang, Samar, as to the extent of Pinca's loss should
be considered sufficient. Notably,MICO submitted no evidence to the contrary nor did it even question the extent of
the loss in its answer before the Insurance Commission. It is also worth observing that Pinca's property was not the
only building bumed in the fire that razed the commercial district of Lao-ang, Samar, on January 18, 1982. 27

There is nothing in the Insurance Code that makes the participation of an adjuster in the assessment of the loss
imperative or indespensable, as MICO suggests. Section 325, which it cites, simply speaks of the licensing and
duties of adjusters.

We see in this cases an obvious design to evade or at least delay the discharge of a just obligation through efforts
bordering on bad faith if not plain duplicity, We note that the motion for reconsideration was filed on the fifteenth day
from notice of the decision of the Insurance Commission and that there was a feeble attempt to show that the notice
of denial of the said motion was not received on June 13, 1982, to further hinder the proceedings and justify the
filing of the petition with this Court fourteen days after June 18, 1982. We also look askance at the alleged
cancellation, of which the insured and MICO's agent himself had no knowledge, and the curious fact that although
Pinca's payment was remitted to MICO's by its agent on January 15, 1982, MICO sought to return it to Adora only
on February 5, 1982, after it presumably had learned of the occurrence of the loss insured against on January 18,
1982. These circumstances make the motives of the petitioner highly suspect, to say the least, and cast serious
doubts upon its candor and bona fides.

WHEREFORE, the petition is DENIED. The decision of the Insurance Commission dated April 10, 1981, and its
Order of June 4, 1981, are AFFIRMED in full, with costs against the petitioner. This decision is immediately
executory.
G.R. No. 95546 November 6, 1992

MAKATI TUSCANY CONDOMINIUM CORPORATION, petitioner,


vs.
THE COURT OF APPEALS, AMERICAN HOME ASSURANCE CO., represented by American International
Underwriters (Phils.), Inc., respondent.

BELLOSILLO, J.:

This case involves a purely legal question: whether payment by installment of the premiums due on an insurance
policy invalidates the contract of insurance, in view of Sec. 77 of P.D. 612, otherwise known as the Insurance Code,
as amended, which provides:

Sec. 77. An insurer is entitled to the payment of the premium as soon as the thing is exposed to the
peril insured against. Notwithstanding any agreement to the contrary, no policy or contract of
insurance issued by an insurance company is valid and binding unless and until the premium thereof
has been paid, except in the case of a life or an industrial life policy whenever the grace period
provision applies.

Sometime in early 1982, private respondent American Home Assurance Co. (AHAC), represented by American
International Underwriters (Phils.), Inc., issued in favor of petitioner Makati Tuscany Condominium Corporation
(TUSCANY) Insurance Policy No. AH-CPP-9210452 on the latter's building and premises, for a period beginning 1
March 1982 and ending 1 March 1983, with a total premium of P466,103.05. The premium was paid on installments
on 12 March 1982, 20 May 1982, 21 June 1982 and 16 November 1982, all of which were accepted by private
respondent.

On 10 February 1983, private respondent issued to petitioner Insurance Policy No. AH-CPP-9210596, which
replaced and renewed the previous policy, for a term covering 1 March 1983 to 1 March 1984. The premium in the
amount of P466,103.05 was again paid on installments on 13 April 1983, 13 July 1983, 3 August 1983, 9 September
1983, and 21 November 1983. All payments were likewise accepted by private respondent.

On 20 January 1984, the policy was again renewed and private respondent issued to petitioner Insurance Policy No.
AH-CPP-9210651 for the period 1 March 1984 to 1 March 1985. On this renewed policy, petitioner made two
installment payments, both accepted by private respondent, the first on 6 February 1984 for P52,000.00 and the
second, on 6 June 1984 for P100,000.00. Thereafter, petitioner refused to pay the balance of the premium.

Consequently, private respondent filed an action to recover the unpaid balance of P314,103.05 for Insurance Policy
No. AH-CPP-9210651.

In its answer with counterclaim, petitioner admitted the issuance of Insurance Policy No. AH-CPP-9210651. It
explained that it discontinued the payment of premiums because the policy did not contain a credit clause in its favor
and the receipts for the installment payments covering the policy for 1984-85, as well as the two (2) previous
policies, stated the following reservations:

2. Acceptance of this payment shall not waive any of the company rights to deny liability on any
claim under the policy arising before such payments or after the expiration of the credit clause of the
policy; and

3. Subject to no loss prior to premium payment. If there be any loss such is not covered.

Petitioner further claimed that the policy was never binding and valid, and no risk attached to the policy. It then
pleaded a counterclaim for P152,000.00 for the premiums already paid for 1984-85, and in its answer with amended
counterclaim, sought the refund of P924,206.10 representing the premium payments for 1982-85.

After some incidents, petitioner and private respondent moved for summary judgment.
On 8 October 1987, the trial court dismissed the complaint and the counterclaim upon the following findings:

While it is true that the receipts issued to the defendant contained the aforementioned reservations,
it is equally true that payment of the premiums of the three aforementioned policies (being sought to
be refunded) were made during the lifetime or term of said policies, hence, it could not be said,
inspite of the reservations, that no risk attached under the policies. Consequently, defendant's
counterclaim for refund is not justified.

As regards the unpaid premiums on Insurance Policy No. AH-CPP-9210651, in view of the
reservation in the receipts ordinarily issued by the plaintiff on premium payments the only plausible
conclusion is that plaintiff has no right to demand their payment after the lapse of the term of said
policy on March 1, 1985. Therefore, the defendant was justified in refusing to pay the same. 1

Both parties appealed from the judgment of the trial court. Thereafter, the Court of Appeals rendered a
decision 2modifying that of the trial court by ordering herein petitioner to pay the balance of the premiums due on
Policy No. AH-CPP-921-651, or P314,103.05 plus legal interest until fully paid, and affirming the denial of the
counterclaim. The appellate court thus explained

The obligation to pay premiums when due is ordinarily as indivisible obligation to pay the entire
premium. Here, the parties herein agreed to make the premiums payable in installments, and there
is no pretense that the parties never envisioned to make the insurance contract binding between
them. It was renewed for two succeeding years, the second and third policies being a
renewal/replacement for the previous one. And the insured never informed the insurer that it was
terminating the policy because the terms were unacceptable.

While it may be true that under Section 77 of the Insurance Code, the parties may not agree to make
the insurance contract valid and binding without payment of premiums, there is nothing in said
section which suggests that the parties may not agree to allow payment of the premiums in
installment, or to consider the contract as valid and binding upon payment of the first premium.
Otherwise, we would allow the insurer to renege on its liability under the contract, had a loss
incurred (sic) before completion of payment of the entire premium, despite its voluntary acceptance
of partial payments, a result eschewed by a basic considerations of fairness and equity.

To our mind, the insurance contract became valid and binding upon payment of the first premium,
and the plaintiff could not have denied liability on the ground that payment was not made in full, for
the reason that it agreed to accept installment payment. . . . 3

Petitioner now asserts that its payment by installment of the premiums for the insurance policies for 1982, 1983 and
1984 invalidated said policies because of the provisions of Sec. 77 of the Insurance Code, as amended, and by the
conditions stipulated by the insurer in its receipts, disclaiming liability for loss for occurring before payment of
premiums.

It argues that where the premiums is not actually paid in full, the policy would only be effective if there is an
acknowledgment in the policy of the receipt of premium pursuant to Sec. 78 of the Insurance Code. The absence of
an express acknowledgment in the policies of such receipt of the corresponding premium payments, and petitioner's
failure to pay said premiums on or before the effective dates of said policies rendered them invalid. Petitioner thus
concludes that there cannot be a perfected contract of insurance upon mere partial payment of the premiums
because under Sec. 77 of the Insurance Code, no contract of insurance is valid and binding unless the premium
thereof has been paid, notwithstanding any agreement to the contrary. As a consequence, petitioner seeks a refund
of all premium payments made on the alleged invalid insurance policies.

We hold that the subject policies are valid even if the premiums were paid on installments. The records clearly show
that petitioner and private respondent intended subject insurance policies to be binding and effective
notwithstanding the staggered payment of the premiums. The initial insurance contract entered into in 1982 was
renewed in 1983, then in 1984. In those three (3) years, the insurer accepted all the installment payments. Such
acceptance of payments speaks loudly of the insurer's intention to honor the policies it issued to petitioner.
Certainly, basic principles of equity and fairness would not allow the insurer to continue collecting and accepting the
premiums, although paid on installments, and later deny liability on the lame excuse that the premiums were not
prepared in full.

We therefore sustain the Court of Appeals. We quote with approval the well-reasoned findings and conclusion of the
appellate court contained in its Resolution denying the motion to reconsider its Decision

While the import of Section 77 is that prepayment of premiums is strictly required as a condition to
the validity of the contract, We are not prepared to rule that the request to make installment
payments duly approved by the insurer, would prevent the entire contract of insurance from going
into effect despite payment and acceptance of the initial premium or first installment. Section 78 of
the Insurance Code in effect allows waiver by the insurer of the condition of prepayment by making
an acknowledgment in the insurance policy of receipt of premium as conclusive evidence of payment
so far as to make the policy binding despite the fact that premium is actually unpaid. Section 77
merely precludes the parties from stipulating that the policy is valid even if premiums are not paid,
but does not expressly prohibit an agreement granting credit extension, and such an agreement is
not contrary to morals, good customs, public order or public policy (De Leon, the Insurance Code, at
p. 175). So is an understanding to allow insured to pay premiums in installments not so proscribed.
At the very least, both parties should be deemed in estoppel to question the arrangement they have
voluntarily accepted. 4

The reliance by petitioner on Arce vs. Capital Surety and Insurance


Co. 5 is unavailing because the facts therein are substantially different from those in the case at bar. In Arce, no
payment was made by the insured at all despite the grace period given. In the case before Us, petitioner paid the
initial installment and thereafter made staggered payments resulting in full payment of the 1982 and 1983 insurance
policies. For the 1984 policy, petitioner paid two (2) installments although it refused to pay the balance.

It appearing from the peculiar circumstances that the parties actually intended to make three (3) insurance contracts
valid, effective and binding, petitioner may not be allowed to renege on its obligation to pay the balance of the
premium after the expiration of the whole term of the third policy (No. AH-CPP-9210651) in March 1985. Moreover,
as correctly observed by the appellate court, where the risk is entire and the contract is indivisible, the insured is not
entitled to a refund of the premiums paid if the insurer was exposed to the risk insured for any period, however brief
or momentary.

WHEREFORE, finding no reversible error in the judgment appealed from, the same is AFFIRMED. Costs against
petitioner.
[G.R. No. 119655. May 24, 1996]

SPS. ANTONIO A. TIBAY and VIOLETA R. TIBAY and OFELIA M. RORALDO, VICTORINA M. RORALDO,
VIRGILIO M. RORALDO, MYRNA M. RORALDO and ROSABELLA M. RORALDO, petitioners,
vs. COURT OF APPEALS and FORTUNE LIFE AND GENERAL INSURANCE CO.,
INC., respondents.

D E C I S I O N*
BELLOSILLO, J.:

May a fire insurance policy be valid, binding and enforceable upon mere partial payment of premium?
On 22 January 1987 private respondent Fortune Life and General Insurance Co., Inc. (FORTUNE) issued
Fire Insurance Policy No. 136171 in favor of Violeta R. Tibay and/or Nicolas Roraldo on their two-storey
residential building located at 5855 Zobel Street, Makati City, together with all their personal effects therein.
The insurance was for P600,000.00 covering the period from 23 January 1987 to 23 January 1988. On 23
January 1987, of the total premium of P2,983.50, petitioner Violeta Tibay only paid P600.00 thus leaving a
considerable balance unpaid.
On 8 March 1987 the insured building was completely destroyed by fire. Two days later or on 10 March
1987 Violeta Tibay paid the balance of the premium. On the same day, she filed with FORTUNE a claim on the
fire insurance policy. Her claim was accordingly referred to its adjuster, Goodwill Adjustment Services, Inc.
(GASI), which immediately wrote Violeta requesting her to furnish it with the necessary documents for the
investigation and processing of her claim. Petitioner forthwith complied. On 28 March 1987 she signed a non-
waiver agreement with GASI to the effect that any action taken by the companies or their representatives in
investigating the claim made by the claimant for his loss which occurred at 5855 Zobel Roxas, Makati on
March 8, 1987, or in the investigating or ascertainment of the amount of actual cash value and loss, shall not
waive or invalidate any condition of the policies of such companies held by said claimant, nor the rights of
either or any of the parties to this agreement, and such action shall not be, or be claimed to be, an admission
of liability on the part of said companies or any of them.[1]
In a letter dated 11 June 1987 FORTUNE denied the claim of Violeta for violation of Policy Condition No. 2
and of Sec. 77 of the Insurance Code. Efforts to settle the case before the Insurance Commission proved
futile. On 3 March 1988 Violeta and the other petitioners sued FORTUNE for damages in the amount of
P600,000.00 representing the total coverage of the fire insurance policy plus 12% interest per annum, P
100,000.00 moral damages, and attorneys fees equivalent to 20% of the total claim.
On 19 July 1990 the trial court ruled for petitioners and adjudged FORTUNE liable for the total value of the
insured building and personal properties in the amount of P600,000.00 plus interest at the legal rate of 6% per
annum from the filing of the complaint until full payment, and attorneys fees equivalent to 20% of the total
amount claimed plus costs of suit.[2]
On 24 March 1995 the Court of Appeals reversed the court a quo by declaring FORTUNE not to be liable
to plaintiff-appellees therein but ordering defendant-appellant to return to the former the premium of P2,983.50
plus 12% interest from 10 March 1987 until full payment.[3]
Hence this petition for review with petitioners contending mainly that contrary to the conclusion of the
appellate court, FORTUNE remains liable under the subject fire insurance policy inspite of the failure of
petitioners to pay their premium in full.
We find no merit in the petition; hence, we affirm the Court of Appeals.

Insurance is a contract whereby one undertakes for a consideration to indemnify another against loss, damage or liability
arising from an unknown or contingent event.[4] The consideration is the premium, which must be paid at the time and in
the way and manner specified in the policy, and if not so paid, the policy will lapse and be forfeited by its own terms.[5]

The pertinent provisions in the Policy on premium read


THIS POLICY OF INSURANCE WITNESSETH, THAT only after payment to the Company in accordance with Policy
Condition No. 2 of the total premiums by the insured as stipulated above for the period aforementioned for insuring
against Loss or Damage by Fire or Lightning as herein appears, the Property herein described x x x

2. This policy including any renewal thereof and/or any endorsement thereon is not in force until the premium has been
fully paid to and duly receipted by the Company in the manner provided herein.

Any supplementary agreement seeking to amend this condition prepared by agent, broker or Company official, shall be
deemed invalid and of no effect.

xxx xxx xxx

Except only in those specific cases where corresponding rules and regulations which are or may hereafter be in force
provide for the payment of the stipulated premiums in periodic installments at fixed percentage, it is hereby declared,
agreed and warranted that this policy shall be deemed effective, valid and binding upon the Company only when the
premiums therefor have actually been paid in full and duly acknowledged in a receipt signed by any authorized official or
representative/agent of the Company in such manner as provided herein, (Italics supplied).[6]

Clearly the Policy provides for payment of premium in full. Accordingly, where the premium has only been
partially paid and the balance paid only after the peril insured against has occurred, the insurance contract did
not take effect and the insured cannot collect at all on the policy. This is fully supported by Sec. 77 of the
Insurance Code which provides

SEC. 77. An insurer is entitled to payment of the premium as soon as the thing insured is exposed to the peril insured
against. Notwithstanding any agreement to the contrary, no policy or contract of insurance issued by an insurance
company is valid and binding unless and until the premium thereof has been paid, except in the case of a life or an
industrial life policy whenever the grace period provision applies (Italics supplied).

Apparently the crux of the controversy lies in the phrase unless and until the premium thereof has been
paid. This leads us to the manner of payment envisioned by the law to make the insurance policy operative
and binding. For whatever judicial construction may be accorded the disputed phrase must ultimately yield to
the clear mandate of the law. The principle that where the law does not distinguish the court should neither
distinguish assumes that the legislature made no qualification on the use of a general word or expression.
In Escosura v. San Miguel Brewery, inc.,[7] the Court through Mr. Justice Jesus G. Barrera, interpreting the
phrase with pay used in connection with leaves of absence with pay granted to employees, ruled -

x x x the legislative practice seems to be that when the intention is to distinguish between full and partial payment, the
modifying term is used x x x

Citing C. A. No. 647 governing maternity leaves of married women in government, R. A. No. 679 regulating
employment of women and children, R.A. No. 843 granting vacation and sick leaves to judges of municipal
courts and justices of the peace, and finally, Art. 1695 of the New Civil Code providing that every househelp
shall be allowed four (4) days vacation each month, which laws simply stated with pay, the Court concluded
that it was undisputed that in all these laws the phrase with pay used without any qualifying adjective meant
that the employee was entitled to full compensation during his leave of absence.
Petitioners maintain otherwise. Insisting that FORTUNE is liable on the policy despite partial payment of
the premium due and the express stipulation thereof to the contrary, petitioners rely heavily on the 1967 case
of Philippine Phoenix and Insurance Co., Inc. v. Woodworks, Inc.[8] where the Court through Mr. Justice
Arsenio P. Dizon sustained the ruling of the trial court that partial payment of the premium made the policy
effective during the whole period of the policy. In that case, the insurance company commenced action against
the insured for the unpaid balance on a fire insurance policy. In its defense the insured claimed that
nonpayment of premium produced the cancellation of the insurance contract. Ruling otherwise the Court held

It is clear x x x that on April 1, 1960, Fire Insurance Policy No. 9652 was issued by appellee and delivered to appellant,
and that on September 22 of the same year, the latter paid to the former the sum of P3,000.00 on account of the total
premium of P6,051.95 due thereon. There is, consequently, no doubt at all that, as between the insurer and the insured,
there was not only a perfected contract of insurance but a partially performed one as far as the payment of the agreed
premium was concerned. Thereafter the obligation of the insurer to pay the insured the amount, for which the policy was
issued in case the conditions therefor had been complied with, arose and became binding upon it, while the obligation of
the insured to pay the remainder of the total amount of the premium due became demandable.

The 1967 Phoenix case is not persuasive; neither is it decisive of the instant dispute. For one, the factual scenario is
different. In Phoenix it was the insurance company that sued for the balance of the premium, i.e., it recognized and
admitted the existence of an insurance contract with the insured. In the case before us, there is, quite unlike in Phoenix, a
specific stipulation that (t)his policy xxx is not in force until the premium has been fully paid and duly receipted by the
Company x x x. Resultantly, it is correct to say that in Phoenix a contract was perfected upon partial payment of the
premium since the parties had not otherwise stipulated that prepayment of the premium in full was a condition precedent
to the existence of a contract.

In Phoenix, by accepting the initial payment of P3,000.00 and then later demanding the remainder of the premium without
any other precondition to its enforceability as in the instant case, the insurer in effect had shown its intention to continue
with the existing contract of insurance, as in fact it was enforcing its right to collect premium, or exact specific
performance from the insured. This is not so here. By express agreement of the parties, no vinculum juris or bond of law
was to be established until full payment was effected prior to the occurrence of the risk insured against.

In Makati Tuscany Condominium Corp. v. Court of Appeals[9] the parties mutually agreed that the
premiums could be paid in installments, which in fact they did for three (3) years, hence, this Court refused to
invalidate the insurance policy. In giving effect to the policy, the Court quoted with approval the Court of
Appeals

The obligation to pay premiums when due is ordinarily an indivisible obligation to pay the entire premium. Here, the
parties x x x agreed to make the premiums payable in installments, and there is no pretense that the parties never
envisioned to make the insurance contract binding between them. It was renewed for two succeeding years, the second
and third policies being a renewal/replacement for the previous one. And the insured never informed the insurer that it
was terminating the policy because the terms were unacceptable.

While it maybe true that under Section 77 of the Insurance Code, the parties may not agree to make the insurance contract
valid and binding without payment of premiums, there is nothing in said section which suggests that the parties may not
agree to allow payment of the premiums in installment, or to consider the contract as valid and binding upon payment of
the first premium. Otherwise we would allow the insurer to renege on its liability under the contract, had a loss incurred
(sic) before completion of payment of the entire premium, despite its voluntary acceptance of partial payments, a result
eschewed by basic considerations of fairness and equity x x x.

These two (2) cases, Phoenix and Tuscany, adequately demonstrate the waiver, either express or implied,
of prepayment in full by the insurer: impliedly, by suing for the balance of the premium as inPhoenix, and
expressly, by agreeing to make premiums payable in installments as in Tuscany. But contrary to the stance
taken by petitioners, there is no waiver express or implied in the case at bench. Precisely, the insurer and the
insured expressly stipulated that (t)his policy including any renewal thereof and/or any indorsement thereon is
not in force until the premium has been fully paid to and duly receipted by the Company x x x and that this
policy shall be deemed effective, valid and binding upon the Company only when the premiums therefor have
actually been paid in full and duly acknowledged.
Conformably with the aforesaid stipulations explicitly worded and taken in conjunction with Sec. 77 of the
Insurance Code the payment of partial premium by the assured in this particular instance should not be
considered the payment required by the law and the stipulation of the parties. Rather, it must be taken in the
concept of a deposit to be held in trust by the insurer until such time that the full amount has been tendered
and duly receipted for. In other words, as expressly agreed upon in the contract, full payment must be made
before the risk occurs for the policy to be considered effective and in force.
Thus, no vinculum juris whereby the insurer bound itself to indemnify the assured according to law ever
resulted from the fractional payment of premium. The insurance contract itself expressly provided that the
policy would be effective only when the premium was paid in full. It would have been altogether different were it
not so stipulated. Ergo, petitioners had absolute freedom of choice whether or not to be insured by FORTUNE
under the terms of its policy and they freely opted to adhere thereto.
Indeed, and far more importantly, the cardinal polestar in the construction of an insurance contract is the
intention of the parties as expressed in the policy.[10] Courts have no other function but to enforce the
same. The rule that contracts of insurance will be construed in favor of the insured and most strongly against
the insurer should not be permitted to have the effect of making a plain agreement ambiguous and then
construe it in favor of the insured.[11] Verily, it is elemental law that the payment of premium is requisite to keep
the policy of insurance in force. If the premium is not paid in the manner prescribed in the policy as intended by
the parties the policy is ineffective. Partial payment even when accepted as a partial payment will not keep the
policy alive even for such fractional part of the year as the part payment bears to the whole payment. [12]
Applying further the rules of statutory construction, the position maintained by petitioners becomes even
more untenable. The case of South Sea Surety and Insurance Company, Inc. v. Court of Appeals,[13] speaks
only of two (2) statutory exceptions to the requirement of payment of the entire premium as a prerequisite to
the validity of the insurance contract. These exceptions are: (a) in case the insurance coverage relates to life or
industrial life (health) insurance when a grace period applies, and (b) when the insurer makes a written
acknowledgment of the receipt of premium, this acknowledgment being declared by law to, be then conclusive
evidence of the premium payment.[14]
A maxim of recognized practicality is the rule that the expressed exception or exemption excludes
others. Exceptio firm at regulim in casibus non exceptis. The express mention of exceptions operates to
exclude other exceptions; conversely, those which are not within the enumerated exceptions are deemed
included in the general rule. Thus, under Sec. 77, as well as Sec. 78, until the premium is paid, and the law
has not expressly excepted partial payments, there is no valid and binding contract. Hence, in the absence of
clear waiver of prepayment in full by the insurer, the insured cannot collect on the proceeds of the policy.
In the desire to safeguard the interest of the assured, itmust not be ignored that the contract of insurance
is primarily a risk-distributing device, a mechanism by which all members of a group exposed to a particular
risk contribute premiums to an insurer. From these contributory funds are paid whatever losses occur due to
exposure to the peril insured against. Each party therefore takes a risk: the insurer, that of being compelled
upon the happening of the contingency to pay the entire sum agreed upon, and the insured, that of parting with
the amount required as premium, without receiving anything therefor in case the contingency does not
happen. To ensure payment for these losses, the law mandates all insurance companies to maintain a legal
reserve fund in favor of those claiming under their policies.[15] It should be understood that the integrity of this
fund cannot be secured and maintained if by judicial fiat partial offerings of premiums were to be construed as
a legal nexus between the applicant and the insurer despite an express agreement to the contrary. For what
could prevent the insurance applicant from deliberately or wilfully holding back full premium payment and wait
for the risk insured against to transpire and then conveniently pass on the balance of the premium to be
deducted from the proceeds of the insurance? Worse, what if the insured makes an initial payment of only
10%, or even 1%, of the required premium, and when the risk occurs simply points to the proceeds from where
to source the balance? Can an insurance company then exist and survive upon the payment of 1%, or even
10%, of the premium stipulated in the policy on the basis that, after all, the insurer can deduct from the
proceeds of the insurance should the risk insured against occur?
Interpreting the contract of insurance stringently against the insurer but liberally in favor of the insured
despite clearly defined obligations of the parties to the policy can be carried out to extremes that there is the
danger that we may, so to speak, kill the goose that lays the golden egg. We are well aware of insurance
companies falling into the despicable habit of collecting premiums promptly yet resorting to all kinds of excuses
to deny or delay payment of just insurance claims. But, in this case, the law is manifestly on the side of the
insurer. For as long as the current Insurance Code remains unchanged and partial payment of premiums is not
mentioned at all as among the exceptions provided in Secs. 77 and 78, no policy of insurance can ever
pretend to be efficacious or effective until premium has been fully paid.
And so it must be. For it cannot be disputed that premium is the elixir vitae of the insurance business
because by law the insurer must maintain a legal reserve fund to meet its contingent obligations to the public,
hence, the imperative need for its prompt payment and full satisfaction. [16] It must be emphasized here that all
actuarial calculations and various tabulations of probabilities of losses under the risks insured against are
based on the sound hypothesis of prompt payment of premiums. Upon this bedrock insurance firms are
enabled to offer the assurance of security to the public at favorable rates. But once payment of premium is left
to the whim and caprice of the insured, as when the courts tolerate the payment of a mere P600.00 as partial
undertaking out of the stipulated total premium of P2,983.50 and the balance to be paid even after the risk
insured against has occurred, as petitioners have done in this case, on the principle that the strength of
the vinculumjuris is not measured by any specific amount of premium payment, we will surely wreak havoc on
the business and set to naught what has taken actuarians centuries to devise to arrive at a fair and equitable
distribution of risks and benefits between the insurer and the insured.
The terms of the insurance policy constitute the measure of the insurers liability. In the absence of
statutory prohibition to the contrary, insurance companies have the same rights as individuals to limit their
liability and to impose whatever conditions they deem best upon their obligations not inconsistent with public
policy.[17] The validity of these limitations is by law passed upon by the Insurance Commissioner who is
empowered to approve all forms of policies, certificates or contracts of insurance which insurers intend to issue
or deliver. That the policy contract in the case at bench was approved and allowed issuance simply reaffirms
the validity of such policy, particularly the provision in question.
WHEREFORE, the petition is DENIED and the assailed Decision of the Court of Appeals dated 24 March
1995 is AFFIRMED.
SO ORDERED.

G.R. No. 102253 June 2, 1995

SOUTH SEA SURETY AND INSURANCE COMPANY, INC., petitioner,


vs.
HON. COURT OF APPEALS and VALENZUELA HARDWOOD AND INDUSTRIAL SUPPLY, INC., respondents.

RESOLUTION
VITUG, J.:

Two issues on the subject of insurance are raised in this petition, that assails the decision, that assails the decision
of the Court of Appeals. (in CA-G.R. NO. CV-20156), the first dealing on the requirement of premium payment and
the second relating to the agency relationship of parties under that contract.

The court litigation started when Valenzuela Hardwood and Industrial Supply, Inc. ("Hardwood"), filed with the
Regional, Trial Court of the National Capital Judicial Region, Branch l71 in Valenzuela, Metro Manila, a complaint for
the recovery of the value of lost logs and freight charges from Seven Brothers Shipping Corporation or, to the extent
of its alleged insurance cover, from South Sea Surety and insurance Company.

The factual backdrop is described briefly by the appellate court thusly:

It appears that on 16 January 1984, plaintiff [Valenzuela Hardwood and Industrial Supply, Inc.]
entered into an agreement with the defendant Seven Brothers whereby the latter undertook to load
on board its vessel M/V Seven Ambassador the former's lauan round logs numbering 940 at the port
of Maconacon, Isabela for shipment to Manila.

On 20 January 1984, plaintiff insured the logs, against loss and/or, damage with defendant South
Sea Surety and Insurance Co., Inc. for P2,000,000.00 end the latter issued its Marine Cargo
Insurance Policy No. 84/24229 for P2,000,000.00 on said date.

On 24 January 1984, the plaintiff gave the check in payment of the premium on the insurance policy
to Mr. Victorio Chua.

In the meantime, the said vessel M/V Seven Ambassador sank on 25 January 1984 resulting in the
loss of the plaintiffs insured logs.

On 30 January 1984, a check for P5,625.00 (Exh. "E") to cover payment of the premium and
documentary stamps due on the policy was tendered to the insurer but was not accepted. Instead,
the South Sea Surety and Insurance Co., Inc. cancelled the insurance policy it issued as of the date
of inception for non-payment of the premium due in accordance with Section 77 of the Insurance
Code.

On 2 February 1984, plaintiff demanded from defendant South Sea Surety and Insurance Co., Inc.
the payment of the proceeds of the policy but the latter denied liability under the policy. Plaintiff
likewise filed a formal claim with defendant Seven Brothers Shipping Corporation for the value of the
lost logs but the latter denied the claim.1

In its decision, dated 11 May 1988, the trial court rendered judgment in favor of plaintiff Hardwood.

On appeal perfected by both the shipping firm and the insurance company, the Court of Appeals affirmed the
judgment of the court a quo only against the insurance corporation; in absolving the shipping entity from liability, the
appellate court ratiocinated:

The primary issue to be resolved before us is whether defendants shipping corporation and the
surety company are liable to the plaintiff for the latter's lost logs.

It appears that there is a stipulation in the charter party that the ship owner would be exempted from
liability in case of loss.

The court a quo erred in applying the provisions of the Civil Code on common carriers to establish
the liability of the shipping corporation. The provisions on common carriers should not be applied
where the carrier is not acting as such but as a private carrier.

Under American jurisprudence, a common carrier undertaking to carry a special or chartered to a


special person only, becomes a private carrier.
As a private carrier, a stipulation exempting the owner from liability even for the negligence of its
agent is valid (Home Insurance Company, Inc. vs. American Steamship Agencies, Inc., 23 SCRA
24).

The shipping corporation should not therefore be held liable for the loss of the logs.2

In this petition for review on certiorari brought by South Sea Surety and Insurance Co., Inc., petitioner argues that it
likewise should have been freed from any liability to Hardwood. It faults the appellate court (a) for having
Supposedly disregarded Section 77 of the insurance Code and (b) for holding Victorio Chua to have been an
authorized representative of the insurer.

Section 77 of the Insurance Code provides:

Sec. 77. An insurer is entitled to payment of the premium as soon as the thing insured is exposed to
the peril insured against. Notwithstanding any agreement to the contrary, no policy or contract of
insurance issued by an insurance company is valid and binding unless and until the premium thereof
has been paid, except in the case of a life or an industrial life policy whenever the grace period
provision applies.

Undoubtedly, the payment of the premium is a condition precedent to, and essential for, the efficaciousness of the
contract. The only two statutorily provided exceptions are (a) in case the insurance coverage relates to life or
industrial life (health) insurance when a grace period applies and (b) when the insurer makes a written
acknowledgment of the receipt of premium, this acknowledgment being declared by law to be then conclusive
evidence of the premium payment (Secs. 77-78, Insurance Code). The appellate court, contrary to what the petition
suggests, did not make any pronouncement to the contrary. Indeed, it has said:

Concerning the issue as to whether there is a valid contract of insurance between plaintiff-appellee
and defendant-appellant South Sea Surety and Insurance Co., Inc., Section 77 of the Insurance
Code explicitly provides that notwithstanding any agreement to the contrary, no policy issued by an
insurance company is valid and binding unless and until premium thereof has been paid. It is
therefore important to determine whether at the time of the loss, the premium was already paid.3

No attempt becloud the issues can disguise the fact that the sole question raised in the instant petition is really
evidentiary in nature, i.e., whether or not Victorio Chua, in receiving the check for the insurance premium prior to the
occurrence of the risk insured against has so acted as an agent of petitioner. The appellate court, like the trial court,
has found in the affirmative. Said the appellate court:

In the instant case, the Marine Cargo Insurance Policy No. 84/24229 was issued by defendant
insurance company on 20 January 1984. At the time the vessel sank on 25 January 1984 resulting in
the loss of the insured logs, the insured had already delivered to Victorio Chua the check in payment
of premium. But, as Victorio Chua testified, it was only in the morning of 30 January 1984 or 5 days
after the vessel sank when his messenger tendered the check to defendant South Sea Surety and
Insurance Co., Inc. (TSN, pp. 3-27, 16-17, 22 October 1985).

The pivotal issue to be resolved to determine the liability, of the surety corporation is whether Mr.
Chua acted as an agent of the surety company or of the insured when he received the check for
insurance premiums.

Appellant surety company insists that Mr. Chua is an administrative assistant for the past ten years
and an agent for less than ten years of the Columbia Insurance Brokers, Ltd. He is paid a salary as a
administrative assistant and a commission as agent based on the premiums he turns over to the
broker. Appellant therefore argues that Mr. Chua, having received the insurance premiums as an
agent of the Columbia Insurance Broker, acted as an agent of the insured under Section 301 of the
Insurance Code which provides as follows:

Sec. 301. Any person who for any compensation, commission or other thing of value,
acts, or aids in soliciting, negotiating or procuring the making of any insurance
contract or in placing risk or taking out insurance, on behalf of an insured other than
himself, shall be an insurance broker within the intent of this Code, and shall thereby
become liable to all the duties requirements, liabilities and penalties to which an
insurance broker is subject.

The appellees, upon the other hand, claim that the second paragraph of Section 306 of the
Insurance Code provide as follows:

Sec. 306. . . . Any insurance company which delivers to an insurance agent or


insurance broker a policy or contract of insurance shall be deemed to have
authorized such agent or broker to receive on its behalf payment of any premium
which is due on such policy of contract of insurance at the time of its issuance or
delivery or which becomes due thereon.

On cross-examination in behalf of South Sea Surety and Insurance Co., Inc. Mr. Chua testified that
the marine cargo insurance policy for the plaintiff's logs was delivered to him on 21 January 1984 at
his office to be delivered to the plaintiff.

When the appellant South Sea Surety and Insurance Co., Inc. delivered to Mr. Chua the marine
cargo insurance policy for the plaintiffs logs, he is deemed to have been authorized by the South
Sea Surety and Insurance Co., Inc. to receive the premium which is due on its behalf.

When therefore the insured logs were lost, the insured had already paid the premium to an agent of
the South Sea Surety and Insurance Co., Inc., which is consequently liable to pay the insurance
proceeds under the policy it issued to the insured.4

We see no valid reason to discard the factual conclusions of the appellate court. Just as so correctly pointed out by
private respondent, it is not the function of this Court to assess and evaluate all over again the evidence, testimonial
and documentary, adduced by the parties particularly where, such as here, the findings of both the trial court and
the appellate court on the matter coincide.

WHEREFORE, the resolution, dated 01 February 1993, granting due course to the petition is RECALLED, and the
petition is DENIED. Costs against petitioner.
[G.R. No. 137172. April 4, 2001]

UCPB GENERAL INSURANCE CO. INC., petitioner, vs. MASAGANA TELAMART,


INC., respondent.

RESOLUTION
DAVIDE, JR., C.J.:

In our decision of 15 June 1999 in this case, we reversed and set aside the assailed decision[1] of the Court
of Appeals, which affirmed with modification the judgment of the trial court (a) allowing Respondent to
consign the sum of P225,753.95 as full payment of the premiums for the renewal of the five insurance policies
on Respondents properties; (b) declaring the replacement-renewal policies effective and binding from 22 May
1992 until 22 May 1993; and (c) ordering Petitioner to pay Respondent P18,645,000.00 as indemnity for the
burned properties covered by the renewal-replacement policies.The modification consisted in the (1) deletion of
the trial courts declaration that three of the policies were in force from August 1991 to August 1992; and (2)
reduction of the award of the attorneys fees from 25% to 10% of the total amount due the Respondent.
The material operative facts upon which the appealed judgment was based are summarized by the Court of
Appeals in its assailed decision as follows:

Plaintiff [herein Respondent] obtained from defendant [herein Petitioner] five (5) insurance policies
(Exhibits "A" to "E", Record, pp. 158-175) on its properties [in Pasay City and Manila].

All five (5) policies reflect on their face the effectivity term: "from 4:00 P.M. of 22 May 1991 to
4:00 P.M. of 22 May 1992." On June 13, 1992, plaintiff's properties located at 2410-2432 and
2442-2450 Taft Avenue, Pasay City were razed by fire. On July 13, 1992, plaintiff tendered, and
defendant accepted, five (5) Equitable Bank Manager's Checks in the total amount of P225,753.45
as renewal premium payments for which Official Receipt Direct Premium No. 62926 (Exhibit "Q",
Record, p. 191) was issued by defendant. On July 14, 1992, Masagana made its formal demand for
indemnification for the burned insured properties. On the same day, defendant returned the five (5)
manager's checks stating in its letter (Exhibit "R"/"8", Record, p. 192) that it was rejecting
Masagana's claim on the following grounds:

"a) Said policies expired last May 22, 1992 and were not renewed for another term;
b) Defendant had put plaintiff and its alleged broker on notice of non-renewal earlier; and
c) The properties covered by the said policies were burned in a fire that took place last June 13, 1992, or before
tender of premium payment."
(Record, p. 5)

Hence Masagana filed this case.

The Court of Appeals disagreed with Petitioners stand that Respondents tender of payment of the premiums
on 13 July 1992 did not result in the renewal of the policies, having been made beyond the effective date of
renewal as provided under Policy Condition No. 26, which states:

26. Renewal Clause. -- Unless the company at least forty five days in advance of the end of the
policy period mails or delivers to the assured at the address shown in the policy notice of its
intention not to renew the policy or to condition its renewal upon reduction of limits or elimination
of coverages, the assured shall be entitled to renew the policy upon payment of the premium due on
the effective date of renewal.

Both the Court of Appeals and the trial court found that sufficient proof exists that Respondent, which had
procured insurance coverage from Petitioner for a number of years, had been granted a 60 to 90-day credit term
for the renewal of the policies. Such a practice had existed up to the time the claims were filed. Thus:

Fire Insurance Policy No. 34658 covering May 22, 1990 to May 22, 1991 was issued on May 7,
1990 but premium was paid more than 90 days later on August 31, 1990 under O.R. No. 4771
(Exhs. "T" and "T-1"). Fire Insurance Policy No. 34660 for Insurance Risk Coverage from May 22,
1990 to May 22, 1991 was issued by UCPB on May 4, 1990 but premium was collected by UCPB
only on July 13, 1990 or more than 60 days later under O.R. No. 46487 (Exhs. "V" and "V-
1"). And so were as other policies: Fire Insurance Policy No. 34657 covering risks from May 22,
1990 to May 22, 1991 was issued on May 7, 1990 but premium therefor was paid only on July 19,
1990 under O.R. No. 46583 (Exhs. "W" and "W-1"). Fire Insurance Policy No. 34661 covering
risks from May 22, 1990 to May 22, 1991 was issued on May 3, 1990 but premium was paid only
on July 19, 1990 under O.R. No. 46582 (Exhs. "X' and "X-1"). Fire Insurance Policy No. 34688 for
insurance coverage from May 22, 1990 to May 22, 1991 was issued on May 7, 1990 but premium
was paid only on July 19, 1990 under O.R. No. 46585 (Exhs. "Y" and "Y-1"). Fire Insurance Policy
No. 29126 to cover insurance risks from May 22, 1989 to May 22, 1990 was issued on May 22,
1989 but premium therefor was collected only on July 25, 1990[sic] under O.R. No. 40799 (Exhs.
"AA" and "AA-1"). Fire Insurance Policy No. HO/F-26408 covering risks from January 12, 1989 to
January 12, 1990 was issued to Intratrade Phils. (Masagana's sister company) dated December 10,
1988 but premium therefor was paid only on February 15, 1989 under O.R. No. 38075 (Exhs. "BB"
and "BB-1"). Fire Insurance Policy No. 29128 was issued on May 22, 1989 but premium was paid
only on July 25, 1989 under O.R. No. 40800 for insurance coverage from May 22, 1989 to May 22,
1990 (Exhs. "CC" and "CC-1"). Fire Insurance Policy No. 29127 was issued on May 22, 1989 but
premium was paid only on July 17, 1989 under O.R. No. 40682 for insurance risk coverage from
May 22, 1989 to May 22, 1990 (Exhs. "DD" and "DD-1"). Fire Insurance Policy No. HO/F-29362
was issued on June 15, 1989 but premium was paid only on February 13, 1990 under O.R. No.
39233 for insurance coverage from May 22, 1989 to May 22, 1990 (Exhs. "EE" and "EE-1"). Fire
Insurance Policy No. 26303 was issued on November 22, 1988 but premium therefor was collected
only on March 15, 1989 under O.R. NO. 38573 for insurance risks coverage from December 15,
1988 to December 15, 1989 (Exhs. "FF" and "FF-1").

Moreover, according to the Court of Appeals the following circumstances constitute preponderant proof
that no timely notice of non-renewal was made by Petitioner:

(1) Defendant-appellant received the confirmation (Exhibit 11, Record, p. 350) from Ultramar
Reinsurance Brokers that plaintiffs reinsurance facility had been confirmed up to 67.5% only
on April 15, 1992 as indicated on Exhibit 11. Apparently, the notice of non-renewal (Exhibit 7,
Record, p. 320) was sent not earlier than said date, or within 45 days from the expiry dates of
the policies as provided under Policy Condition No. 26; (2) Defendant insurer unconditionally
accepted, and issued an official receipt for, the premium payment on July 1[3], 1992 which
indicates defendant's willingness to assume the risk despite only a 67.5% reinsurance
cover[age]; and (3) Defendant insurer appointed Esteban Adjusters and Valuers to investigate
plaintiffs claim as shown by the letter dated July 17, 1992 (Exhibit 11, Record, p. 254).

In our decision of 15 June 1999, we defined the main issue to be whether the fire insurance policies issued
by petitioner to the respondent covering the period from May 22, 1991 to May 22, 1992 had been extended or
renewed by an implied credit arrangement though actual payment of premium was tendered on a later date and
after the occurrence of the (fire) risk insured against. We resolved this issue in the negative in view of Section
77 of the Insurance Code and our decisions in Valenzuela v. Court of Appeals[2]; South Sea Surety and Insurance
Co., Inc. v. Court of Appeals[3]; and Tibay v. Court of Appeals.[4] Accordingly, we reversed and set aside the
decision of the Court of Appeals.
Respondent seasonably filed a motion for the reconsideration of the adverse verdict. It alleges in the motion
that we had made in the decision our own findings of facts, which are not in accord with those of the trial court
and the Court of Appeals. The courts below correctly found that no notice of non-renewal was made within 45
days before 22 May 1992, or before the expiration date of the fire insurance policies. Thus, the policies in
question were renewed by operation of law and were effective and valid on 30 June 1992 when the fire
occurred, since the premiums were paid within the 60- to 90-day credit term.
Respondent likewise disagrees with our ruling that parties may neither agree expressly or impliedly on the
extension of credit or time to pay the premium nor consider a policy binding before actual payment. It urges the
Court to take judicial notice of the fact that despite the express provision of Section 77 of the Insurance Code,
extension of credit terms in premium payment has been the prevalent practice in the insurance industry. Most
insurance companies, including Petitioner, extend credit terms because Section 77 of the Insurance Code is not
a prohibitive injunction but is merely designed for the protection of the parties to an insurance contract. The
Code itself, in Section 78, authorizes the validity of a policy notwithstanding non-payment of premiums.
Respondent also asserts that the principle of estoppel applies to Petitioner. Despite its awareness of Section
77 Petitioner persuaded and induced Respondent to believe that payment of premium on the 60- to 90-day credit
term was perfectly alright; in fact it accepted payments within 60 to 90 days after the due dates. By extending
credit and habitually accepting payments 60 to 90 days from the effective dates of the policies, it has implicitly
agreed to modify the tenor of the insurance policy and in effect waived the provision therein that it would pay
only for the loss or damage in case the same occurred after payment of the premium.
Petitioner filed an opposition to the Respondents motion for reconsideration. It argues that both the trial
court and the Court of Appeals overlooked the fact that on 6 April 1992 Petitioner sent by ordinary mail to
Respondent a notice of non-renewal and sent by personal delivery a copy thereof to Respondents broker,
Zuellig. Both courts likewise ignored the fact that Respondent was fully aware of the notice of non-renewal. A
reading of Section 66 of the Insurance Code readily shows that in order for an insured to be entitled to a renewal
of a non-life policy, payment of the premium due on the effective date of renewal should first be
made. Respondents argument that Section 77 is not a prohibitive provision finds no authoritative support.
Upon a meticulous review of the records and reevaluation of the issues raised in the motion for
reconsideration and the pleadings filed thereafter by the parties, we resolved to grant the motion for
reconsideration. The following facts, as found by the trial court and the Court of Appeals, are indeed duly
established:
1. For years, Petitioner had been issuing fire policies to the Respondent, and these policies were annually
renewed.
2. Petitioner had been granting Respondent a 60- to 90-day credit term within which to pay the premiums on the
renewed policies.
3. There was no valid notice of non-renewal of the policies in question, as there is no proof at all that the notice
sent by ordinary mail was received by Respondent, and the copy thereof allegedly sent to Zuellig was ever
transmitted to Respondent.
4. The premiums for the policies in question in the aggregate amount of P225,753.95 were paid by Respondent
within the 60- to 90-day credit term and were duly accepted and received by Petitioners cashier.
The instant case has to rise or fall on the core issue of whether Section 77 of the Insurance Code of 1978
(P.D. No. 1460) must be strictly applied to Petitioners advantage despite its practice of granting a 60- to 90-day
credit term for the payment of premiums.
Section 77 of the Insurance Code of 1978 provides:

SEC. 77. An insurer is entitled to payment of the premium as soon as the thing insured is exposed
to the peril insured against. Notwithstanding any agreement to the contrary, no policy or contract of
insurance issued by an insurance company is valid and binding unless and until the premium
thereof has been paid, except in the case of a life or an industrial life policy whenever the grace
period provision applies.

This Section is a reproduction of Section 77 of P.D. No. 612 (The Insurance Code) promulgated on 18
December 1974. In turn, this Section has its source in Section 72 of Act No. 2427 otherwise known as the
Insurance Act as amended by R.A. No. 3540, approved on 21 June 1963, which read:

SEC. 72. An insurer is entitled to payment of premium as soon as the thing insured is exposed to
the peril insured against, unless there is clear agreement to grant the insured credit extension of the
premium due. No policy issued by an insurance company is valid and binding unless and until the
premium thereof has been paid. (Underscoring supplied)

It can be seen at once that Section 77 does not restate the portion of Section 72 expressly permitting an
agreement to extend the period to pay the premium. But are there exceptions to Section 77?
The answer is in the affirmative.
The first exception is provided by Section 77 itself, and that is, in case of a life or industrial life policy
whenever the grace period provision applies.
The second is that covered by Section 78 of the Insurance Code, which provides:

SEC. 78. Any acknowledgment in a policy or contract of insurance of the receipt of premium is
conclusive evidence of its payment, so far as to make the policy binding, notwithstanding any
stipulation therein that it shall not be binding until premium is actually paid.

A third exception was laid down in Makati Tuscany Condominium Corporation vs. Court of
Appeals,[5] wherein we ruled that Section 77 may not apply if the parties have agreed to the payment in
installments of the premium and partial payment has been made at the time of loss. We said therein, thus:

We hold that the subject policies are valid even if the premiums were paid on installments. The
records clearly show that the petitioners and private respondent intended subject insurance policies
to be binding and effective notwithstanding the staggered payment of the premiums. The initial
insurance contract entered into in 1982 was renewed in 1983, then in 1984. In those three years, the
insurer accepted all the installment payments. Such acceptance of payments speaks loudly of the
insurers intention to honor the policies it issued to petitioner. Certainly, basic principles of equity
and fairness would not allow the insurer to continue collecting and accepting the premiums,
although paid on installments, and later deny liability on the lame excuse that the premiums were
not prepaid in full.
Not only that. In Tuscany, we also quoted with approval the following pronouncement of the Court of
Appeals in its Resolution denying the motion for reconsideration of its decision:

While the import of Section 77 is that prepayment of premiums is strictly required as a condition to
the validity of the contract, We are not prepared to rule that the request to make installment
payments duly approved by the insurer would prevent the entire contract of insurance from going
into effect despite payment and acceptance of the initial premium or first installment. Section 78 of
the Insurance Code in effect allows waiver by the insurer of the condition of prepayment by making
an acknowledgment in the insurance policy of receipt of premium as conclusive evidence of
payment so far as to make the policy binding despite the fact that premium is actually
unpaid. Section 77 merely precludes the parties from stipulating that the policy is valid even if
premiums are not paid, but does not expressly prohibit an agreement granting credit extension, and
such an agreement is not contrary to morals, good customs, public order or public policy (De Leon,
The Insurance Code, p. 175). So is an understanding to allow insured to pay premiums in
installments not so prescribed. At the very least, both parties should be deemed in estoppel to
question the arrangement they have voluntarily accepted.

By the approval of the aforequoted findings and conclusion of the Court of Appeals, Tuscany has provided
a fourth exception to Section 77, namely, that the insurer may grant credit extension for the payment of the
premium. This simply means that if the insurer has granted the insured a credit term for the payment of the
premium and loss occurs before the expiration of the term, recovery on the policy should be allowed even
though the premium is paid after the loss but within the credit term.
Moreover, there is nothing in Section 77 which prohibits the parties in an insurance contract to provide a
credit term within which to pay the premiums. That agreement is not against the law, morals, good customs,
public order or public policy. The agreement binds the parties. Article 1306 of the Civil Code provides:

ART. 1306. The contracting parties may establish such stipulations clauses, terms and conditions as
they may deem convenient, provided they are not contrary to law, morals, good customs, public
order, or public policy.

Finally in the instant case, it would be unjust and inequitable if recovery on the policy would not be
permitted against Petitioner, which had consistently granted a 60- to 90-day credit term for the payment of
premiums despite its full awareness of Section 77. Estoppel bars it from taking refuge under said Section, since
Respondent relied in good faith on such practice. Estoppel then is the fifth exception to Section 77.
WHEREFORE, the Decision in this case of 15 June 1999 is RECONSIDERED and SET ASIDE, and a
new one is hereby entered DENYING the instant petition for failure of Petitioner to sufficiently show that a
reversible error was committed by the Court of Appeals in its challenged decision, which is
hereby AFFIRMED in toto.
No pronouncement as to cost.
[G.R. No. 130421. June 28, 1999]

AMERICAN HOME ASSURANCE COMPANY, petitioner, vs. ANTONIO


CHUA, respondent.

DECISION
DAVIDE, JR. C.J.:

In this petition for review on certiorari under Rule 45 of the 1997 Rules of Civil Procedure, petitioner
seeks the reversal of the decision[1] of the Court of Appeals in CA-G.R. CV No. 40751, which affirmed in
toto the decision of the Regional Trial Court, Makati City, Branch 150 (hereafter trial court), in Civil Case No.
91-1009.
Petitioner is a domestic corporation engaged in the insurance business. Sometime in 1990, respondent
obtained from petitioner a fire insurance covering the stock-in-trade of his business, Moonlight Enterprises,
located at Valencia, Bukidnon. The insurance was due to expire on 25 March 1990.
On 5 April 1990 respondent issued PCIBank Check No. 352123 in the amount of P2,983.50 to petitioners
agent, James Uy, as payment for the renewal of the policy. In turn, the latter delivered Renewal Certificate No.
00099047 to respondent. The check was drawn against a Manila bank and deposited in petitioners bank account
in Cagayan de Oro City. The corresponding official receipt was issued on 10 April. Subsequently, a new
insurance policy, Policy No. 206-4234498-7, was issued, whereby petitioner undertook to indemnify respondent
for any damage or loss arising from fire up to P200,000 for the period 25 March 1990 to 25 March 1991.
On 6 April 1990 Moonlight Enterprises was completely razed by fire. Total loss was estimated
between P4,000,000 and P5,000,000. Respondent filed an insurance claim with petitioner and four other co-
insurers, namely, Pioneer Insurance and Surety Corporation, Prudential Guarantee and Assurance, Inc., Filipino
Merchants Insurance Co. and Domestic Insurance Company of the Philippines. Petitioner refused to honor the
claim notwithstanding several demands by respondent, thus, the latter filed an action against petitioner before
the trial court.
In its defense, petitioner claimed there was no existing insurance contract when the fire occurred since
respondent did not pay the premium. It also alleged that even assuming there was a contract, respondent
violated several conditions of the policy, particularly: (1) his submission of fraudulent income tax return and
financial statements; (2) his failure to establish the actual loss, which petitioner assessed at P70,000; and (3) his
failure to notify to petitioner of any insurance already effected to cover the insured goods. These violations,
petitioner insisted, justified the denial of the claim.
The trial court ruled in favor of respondent. It found that respondent paid by way of check a day before the
fire occurred. The check, which was deposited in petitioners bank account, was even acknowledged in the
renewal certificate issued by petitioners agent. It declared that the alleged fraudulent documents were limited to
the disparity between the official receipts issued by the Bureau of Internal Revenue (BIR) and the income tax
returns for the years 1987 to 1989. All the other documents were found to be genuine. Nonetheless, it gave
credence to the BIR certification that respondent paid the corresponding taxes due for the questioned years.
As to respondents failure to notify petitioner of the other insurance contracts covering the same goods, the
trial court held that petitioner failed to show that such omission was intentional and fraudulent.Finally, it noted
that petitioners investigation of respondent's claim was done in collaboration with the representatives of other
insurance companies who found no irregularity therein. In fact, Pioneer Insurance and Surety Corporation and
Prudential Guarantee and Assurance, Inc. promptly paid the claims filed by respondent.
The trial court decreed as follows:
WHEREFORE, judgment is hereby rendered in favor of [respondent] and against the [petitioner]
ordering the latter to pay the former the following:

1. P200,000.00, representing the amount of the insurance, plus legal interest from the date of filing of this case;
2. P200,000.00 as moral damages;
3. P200,000.00 as loss of profit;
4. P100,000.00 as exemplary damages;
5. P50,000.00 as attorneys fees; and
6. Cost of suit.
On appeal, the assailed decision was affirmed in toto by the Court of Appeals. The Court of Appeals found
that respondents claim was substantially proved and petitioners unjustified refusal to pay the claim entitled
respondent to the award of damages.
Its motion for reconsideration of the judgment having been denied, petitioner filed the petition in this
case. Petitioner reiterates its stand that there was no existing insurance contract between the parties. It invokes
Section 77 of the Insurance Code, which provides:

An insurer is entitled to payment of the premium as soon as the thing insured is exposed to the peril
insured against. Notwithstanding any agreement to the contrary, no policy or contract of insurance
issued by an insurance company is valid and binding unless and until the premium thereof has been
paid, except in the case of life or an industrial life policy whenever the grace period provision
applies.

and cites the case of Arce v. Capital Insurance & Surety Co., Inc.,[2] where we ruled that unless and until the
premium is paid there is no insurance.
Petitioner emphasizes that when the fire occurred on 6 April 1990 the insurance contract was not yet
subsisting pursuant to Article 1249[3] of the Civil Code, which recognizes that a check can only effect payment
once it has been cashed. Although respondent testified that he gave the check on 5 April to a certain James Uy,
the check, drawn against a Manila bank and deposited in a Cagayan de Oro City bank, could not have been
cleared by 6 April, the date of the fire. In fact, the official receipt issued for respondents check payment was
dated 10 April 1990, four days after the fire occurred.
Citing jurisprudence,[4] petitioner also contends that respondents non-disclosure of the other insurance
contracts rendered the policy void. It underscores the trial courts neglect in considering the Commission on
Audits certification that the BIR receipts submitted by respondent were, in effect, fake since they were issued to
other persons. Finally, petitioner argues that the award of damages was excessive and unreasonable considering
that it did not act in bad faith in denying respondents claim.
Respondent counters that the issue of non-payment of premium is a question of fact which can no longer be
assailed. The trial courts finding on the matter, which was affirmed by the Court of Appeals, is conclusive.
Respondent refutes the reason for petitioners denial of his claim. As found by the trial court, petitioners loss
adjuster admitted prior knowledge of respondents existing insurance contracts with the other insurance
companies. Nonetheless, the loss adjuster recommended the denial of the claim, not because of the said
contracts, but because he was suspicious of the authenticity of certain documents which respondent submitted in
filing his claim.
To bolster his argument, respondent cites Section 66 of the Insurance Code,[5] which requires the insurer to
give a notice to the insured of its intention to terminate the policy forty-five days before the policy period
ends. In the instant case, petitioner opted not to terminate the policy. Instead, it renewed the policy by sending
its agent to respondent, who was issued a renewal certificate upon delivery of his check payment for the
renewal of premium. At this precise moment the contract of insurance was executed and already in
effect. Respondent also claims that it is standard operating procedure in the provinces to pay insurance
premiums by check when collected by insurance agents.
On the issue of damages, respondent maintains that the amounts awarded were reasonable. He cites
numerous trips he had to make from Cagayan de Oro City to Manila to follow up his rightful claim. He imputes
bad faith on petitioner who made enforcement of his claim difficult in the hope that he would eventually
abandon it. He further emphasizes that the adjusters of the other insurance companies recommended payment of
his claim, and they complied therewith.
In its reply, petitioner alleges that the petition questions the conclusions of law made by the trial court and
the Court of Appeals.
Petitioner invokes respondents admission that his check for the renewal of the policy was received only on
10 April 1990, taking into account that the policy period was 25 March 1990 to 25 March 1991.The official
receipt was dated 10 April 1990. Anent respondents testimony that the check was given to petitioners agent, a
certain James Uy, the latter points out that even respondent was not sure if Uy was indeed its agent. It faults
respondent for not producing Uy as his witness and not taking any receipt from him upon presentment of the
check. Even assuming that the check was received a day before the occurrence of the fire, there still could not
have been any payment until the check was cleared.
Moreover, petitioner denies respondents allegation that it intended a renewal of the contract for the renewal
certificate clearly specified the following conditions:

Subject to the payment by the assured of the amount due prior to renewal date, the policy shall be
renewed for the period stated.

Any payment tendered other than in cash is received subject to actual cash collection.

Subject to no loss prior to premium payment. If there be any loss, and is not covered [sic].

Petitioner asserts that an insurance contract can only be enforced upon the payment of the premium, which
should have been made before the renewal period.
Finally, in assailing the excessive damages awarded to respondent petitioner stresses that the policy in issue
was limited to a liability of P200,000; but the trial court granted the following monetary awards: P200,000 as
actual damages; P200,000 as moral damages; P100,000 as exemplary damages; and P50,000 as attorneys fees.
The following issues must be resolved: first, whether there was a valid payment of premium, considering
that respondents check was cashed after the occurrence of the fire; second, whether respondent violated the
policy by his submission of fraudulent documents and non-disclosure of the other existing insurance contracts;
and finally, whether respondent is entitled to the award of damages.
The general rule in insurance laws is that unless the premium is paid the insurance policy is not valid and
binding. The only exceptions are life and industrial life insurance.[6] Whether payment was indeed made is a
question of fact which is best determined by the trial court. The trial court found, as affirmed by the Court of
Appeals, that there was a valid check payment by respondent to petitioner. Well-settled is the rule that the
factual findings and conclusions of the trial court and the Court of Appeals are entitled to great weight and
respect, and will not be disturbed on appeal in the absence of any clear showing that the trial court overlooked
certain facts or circumstances which would substantially affect the disposition of the case.[7] We see no reason to
depart from this ruling.
According to the trial court the renewal certificate issued to respondent contained the acknowledgment that
premium had been paid. It is not disputed that the check drawn by respondent in favor of petitioner and
delivered to its agent was honored when presented and petitioner forthwith issued its official receipt to
respondent on 10 April 1990. Section 306 of the Insurance Code provides that any insurance company which
delivers a policy or contract of insurance to an insurance agent or insurance broker shall be deemed to have
authorized such agent or broker to receive on its behalf payment of any premium which is due on such policy or
contract of insurance at the time of its issuance or delivery or which becomes due thereon. [8] In the instant case,
the best evidence of such authority is the fact that petitioner accepted the check and issued the official receipt
for the payment. It is, as well, bound by its agents acknowledgment of receipt of payment.
Section 78 of the Insurance Code explicitly provides:

An acknowledgment in a policy or contract of insurance of the receipt of premium is conclusive


evidence of its payment, so far as to make the policy binding, notwithstanding any stipulation
therein that it shall not be binding until the premium is actually paid.

This Section establishes a legal fiction of payment and should be interpreted as an exception to Section 77.[9]
Is respondent guilty of the policy violations imputed against him? We are not convinced by petitioners
arguments. The submission of the alleged fraudulent documents pertained to respondents income tax returns for
1987 to 1989. Respondent, however, presented a BIR certification that he had paid the proper taxes for the said
years. The trial court and the Court of Appeals gave credence to the certification and it being a question of fact,
we hold that said finding is conclusive.
Ordinarily, where the insurance policy specifies as a condition the disclosure of existing co-insurers, non-
disclosure thereof is a violation that entitles the insurer to avoid the policy. This condition is common in fire
insurance policies and is known as the other insurance clause. The purpose for the inclusion of this clause is to
prevent an increase in the moral hazard. We have ruled on its validity and the case of Geagonia v. Court of
Appeals[10] clearly illustrates such principle. However, we see an exception in the instant case.
Citing Section 29[11] of the Insurance Code, the trial court reasoned that respondents failure to disclose was
not intentional and fraudulent. The application of Section 29 is misplaced. Section 29 concerns concealment
which is intentional. The relevant provision is Section 75, which provides that:

A policy may declare that a violation of specified provisions thereof shall avoid it, otherwise the
breach of an immaterial provision does not avoid the policy.

To constitute a violation the other existing insurance contracts must be upon the same subject matter and
with the same interest and risk.[12] Indeed, respondent acquired several co-insurers and he failed to disclose this
information to petitioner. Nonetheless, petitioner is estopped from invoking this argument. The trial court cited
the testimony of petitioners loss adjuster who admitted previous knowledge of the co-insurers. Thus,

COURT:

Q The matter of additional insurance of other companies, was that ever discussed in your
investigation?

A Yes, sir.

Q In other words, from the start, you were aware the insured was insured with other companies
like Pioneer and so on?

A Yes, Your Honor.


Q But in your report you never recommended the denial of the claim simply because of the non-
disclosure of other insurance? [sic]

A Yes, Your Honor.

Q In other words, to be emphatic about this, the only reason you recommended the denial of the
claim, you found three documents to be spurious. That is your only basis?

A Yes, Your Honor.[13] [Emphasis supplied]

Indubitably, it cannot be said that petitioner was deceived by respondent by the latters non-disclosure of the
other insurance contracts when petitioner actually had prior knowledge thereof. Petitioners loss adjuster had
known all along of the other existing insurance contracts, yet, he did not use that as basis for his
recommendation of denial. The loss adjuster, being an employee of petitioner, is deemed a representative of the
latter whose awareness of the other insurance contracts binds petitioner. We, therefore, hold that there was no
violation of the other insurance clause by respondent.
Petitioner is liable to pay its share of the loss. The trial court and the Court of Appeals were correct in
awarding P200,000 for this. There is, however, merit in petitioners grievance against the damages and attorneys
fees awarded.
There is no legal and factual basis for the award of P200,000 for loss of profit. It cannot be denied that the
fire totally gutted respondents business; thus, respondent no longer had any business to operate.His loss of profit
cannot be shouldered by petitioner whose obligation is limited to the object of insurance, which was the stock-
in-trade, and not the expected loss in income or profit.
Neither can we approve the award of moral and exemplary damages. At the core of this case is petitioners
alleged breach of its obligation under a contract of insurance. Under Article 2220 of the Civil Code, moral
damages may be awarded in breaches of contracts where the defendant acted fraudulently or in bad faith. We
find no such fraud or bad faith. It must again be stressed that moral damages are emphatically not intended to
enrich a plaintiff at the expense of the defendant. Such damages are awarded only to enable the injured party to
obtain means, diversion or amusements that will serve to obviate the moral suffering he has undergone, by
reason of the defendants culpable action. Its award is aimed at the restoration, within the limits of the possible,
of the spiritual status quo ante, and it must be proportional to the suffering inflicted.[14] When awarded, moral
damages must not be palpably and scandalously excessive as to indicate that it was the result of passion,
prejudice or corruption on the part of the trial court judge.[15]
The law[16] is likewise clear that in contracts and quasi-contracts the court may award exemplary damages if
the defendant acted in a wanton, fraudulent, reckless, oppressive, or malevolent manner.Nothing thereof can be
attributed to petitioner which merely tried to resist what it claimed to be an unfounded claim for enforcement of
the fire insurance policy.
As to attorneys fees, the general rule is that attorneys fees cannot be recovered as part of damages because
of the policy that no premium should be placed on the right to litigate.[17] In short, the grant of attorneys fees as
part of damages is the exception rather than the rule; counsels fees are not awarded every time a party prevails
in a suit. It can be awarded only in the cases enumerated in Article 2208 of the Civil Code, and in all cases it
must be reasonable.[18] Thereunder, the trial court may award attorneys fees where it deems just and equitable
that it be so granted. While we respect the trial courts exercise of its discretion in this case, the award
of P50,000 is unreasonable and excessive. It should be reduced to P10,000.
WHEREFORE, the instant petition is partly GRANTED. The challenged decision of the Court of Appeals
in CA-G.R. No. 40751 is hereby MODIFIED by a) deleting the awards of P200,000 for loss of profit, P200,000
as moral damages and P100,000 as exemplary damages, and b) reducing the award of attorneys fees
from P50,000 to P10,000.

S-ar putea să vă placă și